You are on page 1of 65

INSURANCE LAW CASE DIGESTS

I. GENERAL PROVISIONS (Section 1) A fire broke out in the motor launch ISLAMA. As a
A. Origin of Insurance consequence of which, Francisco del Rosario and 33
B. Laws Governing Insurance in the others were forced to jump off the launch. This
Philippines resulted in the death of Francisco and his beneficiary
i. Insurance Code of 1978 Remedios Jayme.
ii. Civil Code (Articles 739, 2012, 2011,
2207) Equitable insurance paid Simeon del Rosario, father of
iii. Special Laws Francisco Php1000 pursuant to Sec.1 of Part 1 of the
C. Insurance Contract (Section 2) policy. On the day of receipt, Atty. Francisco wrote
i. Definition Equitable acknowledging the receipt of Simeon of the
ii. Elements amount of Php1000 but informed the company that the
iii. Characteristics amount is incorrect as Simeon was entitled to
iv. Interpretation of Insurance Contracts Php1,500, under Sec.2 Part 1 of the policy.

Simeon del Rosario vs. The Equitable Insurance Equitable referred the matter to the Insurance
and Casualty Co Inc. (1963) Commissioner who opined that the liability of the
company is only Php1000. Thus, Equitable refused to
Facts: pay. Subsequently, Atty. Francisco asked for Php3000
from Equitable. The company refused to pay. Hence a
On February 7, 1957, Equitable Insurance and complaint for the recovery of the balance was
Casualty Co., Inc., issued Personal Accident Policy No. instituted.
7136 on the life of Francisco del Rosario, alias Paquito
Bolero, son of Simeon, binding itself to pay the sum of Issue:
P1,000.00 to P3,000.00, as indemnity for the death of
the insured. How much should the indemnity be?

The provisions of the insurance policy pertinent to the Ruling:


case are as follows:
The CFI ruled that:
Part I. Indemnity For Death
If the insured sustains any bodily injury which is On the face of the policy Exhibit "A" itself, death by
effected solely through violent, external, visible and drowning is a ground for recovery apart from the
accidental means, and which shall result, bodily injury because death by bodily injury is covered
independently of all other causes and within sixty (60) by Part I of the policy while death by drowning is
days from the occurrence thereof, in the Death of the covered by Part VI thereof. But while the policy
Insured, the Company shall pay the amount set mentions specific amounts that may be recovered for
opposite such injury: death for bodily injury, yet, there is not specific
amount mentioned in the policy for death thru
Section 1. Injury sustained other than those specified drowning although the latter is, under Part VI of the
below unless excepted hereinafter. . . . . . . . policy, a ground for recovery thereunder. Since the
P1,000.00 defendant has bound itself to pay P1000.00 to
Section 2. Injury sustained by the wrecking or P3,000.00 as indemnity for the death of the insured
disablement of a railroad passenger car or street but the policy does not positively state any definite
railway car in or on which the Insured is travelling as a amount that may be recovered in case of death by
farepaying passenger. . . . . . . P1,500.00 drowning, there is an ambiguity in this respect in the
policy, which ambiguity must be interpreted in favor of
Part VI. Exceptions the insured and strictly against the insurer so as to
This policy shall not cover disappearance of the allow greater indemnity. Thus, del Rosario is entitled to
Insured nor shall it cover Death, Disability, Hospital Php3000. Since Equitable has already paid Php1000, a
fees, or Loss of Time, caused to the insured: balance of Php2000 remains to be paid.
. . . (h) By drowning except as a consequence of the
wrecking or disablement in the Philippine waters of a SC upheld the ruling of the CFI for it is supported by
passenger steam or motor vessel in which the Insured the generally accepted principles of insurance, which
is travelling as a farepaying passenger; . . . . enunciate that where there is an ambiguity with
respect to the terms and conditions of the policy, the
A rider to the Policy contained the following: same will be resolved against the one responsible
thereof.
IV. DROWNING
It is hereby declared and agreed that exemption clause It should be recalled in this connection, that generally,
Letter (h) embodied in PART VI of the policy is hereby the insured, has little, if any, participation in the
waived by the company, and to form a part of the preparation of the policy, together with the drafting of
provision covered by the policy. its terms and Conditions. The interpretation of obscure
stipulations in a contract should not favor the party

EH 403 2010-2011 2011-2012


Page 1
INSURANCE LAW CASE DIGESTS
who cause the obscurity (Art. 1377, N.C.C.), which, in The Court of Appeals rendered a decision in favor of
the case at bar, is the insurance company. the claimants. It held that where inequitable conduct is
shown by an insurance firm, it is estopped from
. . . . And so it has been generally held that the "terms enforcing forfeitures in its favor, in order to forestall
in an insurance policy, which are ambiguous, equivocal fraud or imposition on the insured. After Fieldmen's
or uncertain . . . are to be construed strictly against, Insurance Co. had led the insured Songco to believe
the insurer, and liberally in favor of the insured so as that he could qualify under the common carrier liability
to effect the dominant purpose of indemnity or insurance policy, it could not, thereafter, be permitted
payment to the insured, especially where a forfeiture is to change its stand to the detriment of the heirs of the
involved," (29 Am. Jur. 181) and the reason for this insured. The failure to apply the Doctrine of Estoppel in
rule is that the "insured usually has no voice in the this case would result in a gross travesty of justice.
selection or arrangement of the words employed and
that the language of the contract is selected with great ISSUE:
care and deliberation by expert and legal advisers
employed by, and acting exclusively in the interest of, Whether or not the insurance claim is proper?
the insurance company" (44 C.J.S. 1174). Calanoc v.
Court of Appeals, et al., G.R. No. L-8151, Dec. 16, RULING:
1955.
The fact that the insured owned a private vehicle, not
. . . . Where two interpretations, equally fair, of a common carrier, was something which the company
languages used in an insurance policy may be made, knew all along. In fact, it exerted the utmost pressure
that which allows the greater indemnity will prevail. on the insured, a man of scant education, to enter into
(L'Engel v. Scotish Union & Nat. F. Ins. Co., 48 Fla. 82, the contract of insurance. The Court of Appeals also
37 So. 462, 67 LRA 581 111 Am. St. Rep. 70, 5 Ann. held that since some of the conditions contained in the
Cas. 749). policy were impossible to comply with under the
existing conditions at the time, the insurer is estopped
At any event, the policy under consideration, covers from asserting breach of such conditions.
death or disability by accidental means, and the
appellant insurance company agreed to pay P1,000.00 The Supreme Court, in affirming the decision of the
to P3,000.00. is indemnity for death of the insured. Court of Appeals, took judicial notice of the fact that
nowadays, monopolies, cartels and concentration of
capital, endowed with overwhelming economic power,
FIELDMENS INSURANCE CO. vs. VDA. DE manage to impose upon parties dealing with them
SONGCO cunningly prepared agreements that the weaker
party may not change one whit, his participation in the
FACTS: agreement being reduced to the alternative of take it
or leave it labelled since Raymond Saleilles as
Federico Songco owned a private jeepney. On contracts by adherence (contrats d'adhesion), in
September 15, 1960, he was induced by Fieldmen's contrast to those entered into by parties bargaining on
insurance agent Benjamin Sambat to apply for a an equal footing, such contracts (i.e. insurance policies
Common Carrier's Liability Insurance Policy covering & international bills of lading) obviously call for greater
his motor vehicle. He was issued a Common Carriers strictness and vigilance on the part of courts of justice
Accident Insurance Policy. On the next year, he with a view to protecting the weaker party from
renewed the policy by paying the annual premium. abuses.
During the effectivity of the renewed policy, the
insured vehicle collided with another car while being Citing the case of Qua Chee Gan vs. Law Union & Rock
driven by Rodolfo Songco, a duly licensed driver and Insurance, "The contract of insurance is one of
son of Federico (the vehicle owner). As a result, perfect good faith (uberima fides) not for the insured
Federico Songco (father) and Rodolfo Songco (son) alone but equally so for the insurer; in fact, it is more
died, along with other passengers. so for the latter, since its dominant bargaining position
carries with it stricter responsibility."
A claim was filed but was denied by the insurance
company on the pretext that what was insured was a
private vehicle and not a common carrier. During the Landicho vs. GSIS
trial, it was declared by a witness that when insurance [G.R. No. L-28866 March 17, 1972]
agent Benjamin Sambat was inducing Songco to insure
his vehicle, the latter butted in saying, Our vehicle is FACTS:
a private vehicle and not for passengers. But the
agent replied: Regardless of whether your vehicle was On June 1, 1964, the GSIS issued in favor of Flaviano
an owner-type or for passengers, it could still be Landicho, a civil engineer of the Bureau of Public
insured because our company is not owned by the Works, stationed at Mamburao, Mindoro Occidental,
Government. And the Government has nothing to do optional additional life insurance policy No. OG-136107
with our company. in the sum of P7,900. xxx

EH 403 2010-2011 2011-2012


Page 2
INSURANCE LAW CASE DIGESTS
Before the issuance of said policy, Landicho had filed This is particularly true as regards insurance policies,
an application, by filing and signing a printed form of in respect of which it is settled that the " "terms in an
the GSIS on the basis of which the policy was issued. insurance policy, which are ambiguous, equivocal, or
Paragraph 7 of said application States: uncertain ... are to be construed strictly and most
7. xxx I hereby agree as follows: xxx strongly against the insurer, and liberally in favor of
c. That this application serves as a the insured so as to effect the dominant purpose of
letter of authority to the Collecting indemnity or payment to the insured, especially where
Officer of our Office thru the GSIS to a forfeiture is involved" (29 Am. Jur., 181), and the
deduct from my salary the monthly reason for this rule is the "insured usually has no voice
premium in the amount of P33.36, in the selection or arrangement of the words employed
beginning the month of May, 1964, and that the language of the contract is selected with
and every month thereafter until notice great care and deliberation by experts and legal
of its discontinuance shall have advisers employed by, and acting exclusively in the
beenreceived from the System; . interest of, the insurance company." (44 C.J.S., p.
d. That the failure to deduct from my 1174.) 3.
salary the month premiums shall not
make the policy lapse, however, the The equitable and ethical considerations justifying the
premium account shall be considered foregoing view are bolstered up by two (2) factors,
as indebtedness which, I bind myself to namely:
pay the System; . (a) The aforementioned subdivision (c) states "that
e. That my policy shall be made this application serves as a letter of authority to the
effective on the first day of the month Collecting Officer of our Office" the Bureau of Public
next following the month the first Works "thru the GSIS to deduct from my salary the
premium is paid; provided, that it is monthly premium in the amount of P33.36." No such
not more ninety (90) days before or deduction was made and, consequently, not even
after the date of the medical the first premium "paid" because the collecting
examination, was conducted if officer of the Bureau of Public Works was not advised
required." by the GSIS to make it (the deduction) pursuant to
said authority. Surely, this omission of the GSIS should
While still an employee of the Bureau of Public Works, not inure to its benefit. .
Mr. Landicho died in an airplane crash on June 29,
1966. Mrs. Landicho, in her own behalf and that of her (b) The GSIS had impliedly induced the insured to
co-plaintiffs and minor children, Rafael J. and Maria believe that Policy No. OG-136107 was in force, he
Lourdes Eugenia, filed with the GSIS a claim for having been paid by the GSIS the dividends
P15,800, as the double indemnity due under policy No. corresponding to said policy. Had the insured had the
OG-136107. GSIS denied the claim, upon the ground slightest inkling that the latter was not, as yet,
that the policy had never been in force because, effective for non-payment of the first premium, he
pursuant to subdivision (e) of the above-quoted would have, in all probability, caused the same to be
paragraph 7 of the application, the policy "shall be ... forthwith satisfied.
effective on the first day of the month next following
the month the first premium is paid," and no premium WHEREFORE, the decision appealed from should be, it
had ever been paid on said policy. The Lower Court is hereby affirmed, with costs against the defendant-
decided in favor of the petitioner. GSIS appealed to the appellant, Government Service Insurance System. It is
Supreme Court. so ordered. .

ISSUE:
DELA CRUZ V. CAPITAL INS. & SURETY CO., INC.
WON the insurance policy in question has ever been in
force, not a single premium having been paid thereon. DEATH RESULTING FROM BOXING IS AN ACCIDENT
SINCE DEATH IS NOT A NATURAL OR PROBABLE
RULING: Lower Court decision is sustained. RESULT OF BOXING.

(T)he language, of subdivisions (c), (d) and (e) is such Facts:


as to create an ambiguity that should be resolved
against the party responsible therefor defendant Eduardo de la Cruz, employed as a mucker in the
GSIS, as the party who prepared and furnished the Itogon-Suyoc Mines, Inc. in Baguio, was the holder of
application form and in favor of the party misled an accident insurance policy underwritten by the
thereby, the insured employee. Capital Insurance & Surety Co., Inc., In connection
Indeed, our Civil Code provides: with the celebration of the New Year, the Itogon-Suyoc
The interpretation of obscure words or Mines, Inc. sponsored a boxing contest for general
stipulations in a contract shall not favor entertainment wherein the insured Eduardo de la Cruz,
the party who caused the obscurity. 2 a non-professional boxer participated. In the course of
his bout with another person, likewise a non-
professional, of the same height, weight, and size,

EH 403 2010-2011 2011-2012


Page 3
INSURANCE LAW CASE DIGESTS
Eduardo slipped and was hit by his opponent on the blow in the head and would not have died. The fact
left part of the back of the head, causing Eduardo to that boxing is attended with some risks of external
fall, with his head hitting the rope of the ring. He was injuries does not make any injuries received in the
brought to the Baguio General Hospital where he course of the game not accidental.
expired the the following day. The cause of death was
reported as hemorrhage, intracranial, left.
-Ty
Simon de la Cruz, the father of the insured and who
was named beneficiary under the policy, thereupon New Life Enterprises vs Court of Appeals
filed a claim with the insurance company for payment By: Yin Oliveros
of the indemnity under the insurance policy
FACTS:
Defendant insurer set up the defense that the death of Julian Sy and Jose Sy formed a partnership
the insured, caused by his participation in a boxing under the business name of New Life
contest, was not accidental and, therefore, not covered Enterprises. They were holding their business
by insurance. in a two-storey building in Lucena City.

Issue: Julian Sy insured the stocks in trade of New


Life Enterprises under three insurance
Whether the death of Eduardo was accidental since he companies.
entered in the boxing contest voluntarily.
INSURANCE TYPE OF AMOUNT
Ruling: COMPANY INSURANCE
Western Fire Insurance 350, 000. 00
The terms "accident" and "accidental", as used in Guaranty Policy
insurance contracts, have not acquired any technical Corporation - This policy
meaning, and are construed by the courts in their was
ordinary and common acceptation. Thus, the terms renewed
have been taken to mean that which happen by chance Reliance Fire Insurance 300, 000. 00
or fortuitously, without intention and design, and which Surety and Policy
is unexpected, unusual, and unforeseen. An accident is
- There was an
Insurance Co., This policy was additional
an event that takes place without one's foresight or Inc. also renewed.
expectation an event that proceeds from an insurance
unknown cause, or is an unusual effect of a known issued in the
cause and, therefore, not expected. amount of
700, 000. 00
It may be mentioned in this connection, that the Equitable Fire Insurance 200, 000. 00
tendency of court decisions in the United States in the Insurance Policy
recent years is to eliminate the fine distinction Corporation
between the terms "accidental" and "accidental means" TOTAL: 1, 550,
and to consider them as legally synonymous. 000. 00

The generally accepted rule is that, death or injury The building occupied by New Life Enterprises
does not result from accident or accidental means was gutted by fire caused by a faulty electrical
within the terms of an accident-policy if it is the wiring. According to the plaintiffs, the stocks in
natural result of the insured's voluntary act, trade were inside said building and were thus
unaccompanied by anything unforeseen except the burned.
death or injury. 3 There is no accident when a
deliberate act is performed unless some additional, Julian Sy, together with an agent of Reliance
unexpected, independent, and unforeseen happening Insurance, filed his claim. To support his claim,
occurs which produces or brings about the result of he submitted a fire clearance, the insurance
injury or death. 4 In other words, where the death or policies and the inventory of stocks. He further
injury is not the natural or probable result of the testified that the three insurance companies
insured's voluntary act which produces the injury, the are sister companies, and as a matter of fact
resulting death is within the protection of policies when he was following-up his claim with
insuring against the death or injury from accident. Equitable Insurance, the Claims Manager told
him to go first to Reliance Insurance and if said
In the present case, while the participation of the company agrees to pay, they would also pay.
insured in the boxing contest is voluntary, the injury Ultimately, the three insurance companies
was sustained when he slid, giving occasion to the denied plaintiffs' claim for payment due to
infliction by his opponent of the blow that threw him to BREACH OF POLICY CONDITIONS.
the ropes of the ring. Without this unfortunate
incident, that is, the unintentional slipping of the Reliance Surety and Insurance Company
deceased, perhaps he could not have received that claimed that plaintiff violated Policy Condition

EH 403 2010-2011 2011-2012


Page 4
INSURANCE LAW CASE DIGESTS
No. "3" which requires the insured to give
notice of any insurance or insurances already It was incumbent upon petitioner Sy to read
effected covering the stocks in trade. the insurance contracts, and this can be
reasonably expected of him considering that he
The Trial Court ruled in favor of the plaintiff has been a businessperson since 1965.
that was reversed by the Court of Appeals. Furthermore, the contract concerns indemnity
in case of loss in his moneymaking trade of
ISSUE: which he should have been aware as it was
precisely the reason for his procuring the
Whether or not the plaintiff incurred a breach insurance.
in the policy conditions?

RULING: v. What constitutes doing an insurance


business
The Supreme Court ruled in favor of the
insurance companies. D. Perfection of the Contract of Insurance
a. Offer and Acceptance/Consensuality
The terms of the contract are clear and (1) Delay in Acceptance
unambiguous. The insured is specifically (2) Delivery of Policy
required to disclose to the insurer any other
insurance and its particulars that he may have II. CONTRACT OF INSURANCE
effected on the same subject matter. The A. What may be insured (Sections 3, 4 and 5)
knowledge of such insurance by the insurer's B. Parties to the Contract (Sections 6, 7, 8 and
agents is not the "notice" that would estop the 9)
insurers from denying the claim. i. Who may be an insurer
ii. Who may be insured
Thus, it points out that while petitioner Julian iii. Rules on insurance by mortgagor or
Sy claimed that he had informed insurance mortgagee
agent Alvarez regarding the co-insurance on iv. Transfer of insurance from mortgagor to
the property, he contradicted himself by mortgagee
inexplicably claiming that he had not read the
terms of the policies. -Filipinas
-Geagonio
Furthermore, when the words and language -PNB
of documents are clear and plain or
readily understandable by an ordinary
reader thereof, there is absolutely no PALILEO v. COSIO
room for interpretation or construction [G.R. No. L-7667 November 28, 1955]
anymore. Courts are not allowed to make
contracts for the parties; rather, they will FACTS:
intervene only when the terms of the
policy are ambiguous, equivocal, or On Dec. 18, 1951, Palileo obtained from Cosio a loan in
uncertain. The parties must abide by the the sum of 12,000. Pursuant to their agreement,
terms of the contract because such terms Palileo paid to Cosio as interest on the loan a total of
constitute the measure of the insurer's liability P2,250 corresponding to 9 mos from Dec 18, 1951, on
and compliance therewith is a condition the basis of P250 a month, which is more than the
precedent to the insured's right of recovery maximum interest allowed by law. To secure the
from the insurer. payment of the aforesaid loan, defendant required
plaintiff to sign a a document known as Conditional
While it is a cardinal principle of insurance Sale of Residential Bldg purporting to convey to
law that a policy or contract of insurance defendant, with right to repurchase, a two-story
is to be construed liberally in favor of the building of strong materials belonging to plaintiff. This
insured and strictly against the insurer document did not express the true intention of the
company, yet contracts of insurance, like parties, which was merely to place said property as
other contracts, are to be construed security for the payment of the loan. After the
according to the sense and meaning of the execution of the document, defendant insured the
terms which the parties themselves have building against fire with the Associated Insurance &
used. If such terms are clear and Surety Co., Inc. for the sum of P15000, the insurance
unambiguous, they must be taken and policy having been issued in the name of defendant.
understood in their plain, ordinary and The building was partly destroyed by fire and, after
popular sense. Moreover, obligations arising proper demand, defendant collected from the
from contracts have the force of law between insurance company an indemnity of P13,107. Plaintiff
the contracting parties and should be complied demanded from defendant that she be credited with
with in good faith.

EH 403 2010-2011 2011-2012


Page 5
INSURANCE LAW CASE DIGESTS
the necessary amount to pay her obligation out of the mortgagee under the mortgage. This is put upon the
insurance proceeds but defendant refused to do so. analogy of the situation of the insurer to that of a
surety.
Upon these facts, the trial court held that the
defendant should credit the sum of P13,107 received The correct solution, contrary to the trial courts ruling,
by him from the Associated Insurance & Surety Co., should be that the proceeds of the insurance should be
Inc. to the payment of plaintiffs obligation in the sum delivered to the defendant but that her claim against
of P12000, thus considering the agreement fully paid the plaintiff should be considered assigned to the
and leaving a balance of P1107 from the insurance insurance company who is deemed subrogated to the
collected by the defendant; and since plaintiff had paid rights of the defendant to the extent of the money paid
to defendant P2250 for 9 mos for interest which as indemnity.
exceeds the 12 percent per annum legal interest
(P1440 for one year), plaintiff overpaid P810,
defendant should refund plaintiff the total of P1107 C. Insurable Interest
plus P810 and to pay the costs. i. Insurable Interest in life and health
(Section 10)
ISSUE:
El Oriente Fabrica de Tabacos, Inc. vs. Juan
WON the trial court is justified in considering the Posadas, Collector of Internal Revenue
obligation of plaintiff fully compensated by the [G.R. No. 34774, September 21, 1931]
insurance amount and in ordering defendant to refund
to plaintiff the sum of P1107 representing the Facts:
difference of the loan of 12K from the sum of P13107
collected by defendant from insurance notwithstanding Insurer: Manufacturers Life Insurance Co., of Toronto,
the fact that it was proven that the insurance was Canada, thru its local agent E.E. Elser
taken for the benefit of the mortgagor? Insured: A. Velhagen (manager of El Oriente)
Beneficiary: El Oriente Fabrica de Tabacos, Inc.
HELD:
El Oriente, in order to protect itself against the loss
SC modified the judgment of the lower court as that it might suffer by reason of the death of its
follows: (1) the transaction had between the plaintiff manager, whose death would be a serious loss to El
and defendant was merely an equitable mortgage Oriente procured from the Insurer an insurance policy
intended to secure the payment of the loan of 12K; (2) on the life of the said manager for the sum of 50,000
that the proceeds of the insurance amounting to P13, USD with El Oriente as the designated sole beneficiary.
107 was properly collected by defendant who is not The insured has no interest or participation in the
required to account for it to the plaintiff; (3) that the proceeds of said life insurance policy.
collection of said insurance proceeds shall not be
deemed to have compensated the obligation of the El Oriente charged as expenses of its business all the
plaintiff to the defendant, but bars the latter from said premiums and deducted the same from its gross
claiming its payment from the former; and (4) incomes as reported in its annual income tax returns,
defendant shall pay to the plaintiff the sum of P810 which deductions were allowed by Posadas (Collector
representing the overpayment made by plaintiff by of Internal Revenue) upon showing by El Oriente that
way of interest on the loan. such premiums were legitimate expenses of the
business.
The rule is that where a mortgagee, independently of
the mortgagor, insures the mortgaged property in his Upon the death of the manager, El Oriente received all
own name and for his own interest, he is entitled to the proceeds of the life insurance policy together with
the insurance proceeds in case of loss, but in such the interest and the dividends accruing thereon,
case, he is not allowed to retain his claim against the aggregating P104,957.88. Posadas assessed and
mortgagor, but is passed by subrogation to the insurer levied the sum of P3,148.74 as income tax on the
to the extent of the money paid. Or, stated in another proceeds of the insurance policy, which was paid by El
way, the mortgagee may insure his interest in the Oriente under protest. El Oriente claiming exemption
property independently of the mortgagor. In that under Section 4 of the Income Tax Law.
event, upon the destruction of the property the
insurance money paid to the mortgagee will not inure Issue:
to the benefit of the mortgagor, and the amount due
under the mortgage debt remains unchanged. The Whether or not the proceeds of insurance taken by a
mortgagee, however, is not allowed to retain his claim corporation on the life of an important official to
against the mortgagor, but it passes by subrogation to indemnify it against loss in case of his death, are
the insurer, to the extent of the insurance money taxable as income under the Philippine Income Tax
paid. Law?

The general rule and the weight of authority is, that Ruling:
the insurer is thereupon subrogated to the rights of the

EH 403 2010-2011 2011-2012


Page 6
INSURANCE LAW CASE DIGESTS
The Income Tax Law for the Philippines is Act No. THE INSULAR LIFE ASSURANCE COMPANY, LTD.
2833, as amended. In chapter I On Individuals, is vs. CARPONIA T. EBRADO and PASCUALA VDA. DE
to be found section 4 which provides that, "The EBRADO
following incomes shall be exempt from the provisions [G.R. No. L-44059 October 28, 1977]
of this law: (a) The proceeds of life insurance policies
paid to beneficiaries upon the death of the Facts of the Case:
insured ... ." The Chapter on Corporations does not
provide as above. It is certain that the proceeds of life On September 1, 1968, Buenaventura Cristor Ebrado
insurance policies are exempt. It is not so certain that was issued by The Life Assurance Co., Ltd., Policy No.
the proceeds of life insurance policies paid to corporate 009929 on a whole-life for P5,882.00 with a, rider for
beneficiaries upon the death of the insured are likewise Accidental Death for the same amount Buenaventura
exempt. C. Ebrado designated Carpponia T. Ebrado as the
revocable beneficiary in his policy. He to her as his
The situation will be better elucidated by a brief wife.
reference to laws on the same subject in the United
States. The Income Tax Law of 1916 extended to the On October 21, 1969, Buenaventura C. Ebrado died
Philippine Legislature, when it came to enact Act No. when he was hit by a failing branch of a tree. As the
2833, to copy the American statute. Subsequently, the policy was in force, The Insular Life Assurance Co.,
Congress of the United States enacted its Income Tax Ltd. liable to pay the coverage in the total amount of
Law of 1919, in which certain doubtful subjects were P11,745.73, representing the face value of the policy
clarified. Thus, as to the point before us, it was made in the amount of P5,882.00 plus the additional benefits
clear, when not only in the part of the law concerning for accidental death also in the amount of P5,882.00
individuals were exemptions provided for beneficiaries, and the refund of P18.00 paid for the premium due
but also in the part concerning corporations, specific November, 1969, minus the unpaid premiums and
reference was made to the exemptions in favor of interest thereon due for January and February, 1969,
individuals, thereby making the same applicable to in the sum of P36.27.
corporations. This was authoritatively pointed out and
decided by the United States Supreme Court in the Carponia T. Ebrado filed with the insurer a claim for
case of United States vs. Supplee-Biddle Hardware Co. the proceeds of the Policy as the designated
( [1924], 265 U.S., 189), which involved facts quite beneficiary therein, although she admits that she and
similar to those before us. the insured Buenaventura C. Ebrado were merely living
as husband and wife without the benefit of marriage.
To quote the exact words in the cited case of Chief
Justice Taft delivering the opinion of the court: Pascuala Vda. de Ebrado also filed her claim as the
It is earnestly pressed upon us that proceeds of life widow of the deceased insured. She asserts that she is
insurance paid on the death of the insured are in the one entitled to the insurance proceeds, not the
fact capital, and cannot be taxed as income that common-law wife, Carponia T. Ebrado.
proceeds of a life insurance policy paid on the
death of the insured are not usually classed as In doubt as to whom the insurance proceeds shall be
income. paid, the insurer, The Insular Life Assurance Co., Ltd.
commenced an action for Interpleader before the Court
Considering, therefore, the purport of the stipulated of First Instance of Rizal on April 29, 1970.
facts, considering the uncertainty of Philippine law, and
considering the lack of express legislative intention to After the issues have been joined, a pre-trial
tax the proceeds of life insurance policies paid to conference was held. In the pre-trial conference the
corporate beneficiaries, particularly when in the parties submits evidence and make admissions.xxx; 8)
exemption in favor of individual beneficiaries in the that the beneficiary designated by the insured in the
chapter on this subject, the clause is inserted "exempt policy is Carponia Ebrado and the insured made
from the provisions of this law," we deem it reasonable reservation to change the beneficiary but although the
to hold the proceeds of the life insurance policy in insured made the option to change the beneficiary,
question as representing an indemnity and not taxable same was never changed up to the time of his death
income. and the wife did not have any opportunity to write the
company that there was reservation to change the
The foregoing pronouncement will result in the designation of the parties it agreed that a decision be
judgment being reversed and in another judgment rendered based on and stipulation of facts as to who
being rendered in favor of El Oriente. among the two claimants is entitled to the policy.

On September 25, 1972, the trial court rendered


ii. Rules on change of beneficiary (Section judgment declaring among others, Carponia T. Ebrado
11) disqualified from becoming beneficiary of the insured
Buenaventura Cristor Ebrado and directing the
Digested by: Anne payment of the insurance proceeds to the estate of the
deceased insured. The trial court held that.It is patent
from the last paragraph of Art. 739 of the Civil Code

EH 403 2010-2011 2011-2012


Page 7
INSURANCE LAW CASE DIGESTS
that a criminal conviction for adultery or concubinage
is not essential in order to establish the disqualification In the caw before Us, the requisite proof of common-
mentioned therein. Neither is it also necessary that a law relationship between the insured and the
finding of such guilt or commission of those acts be beneficiary has been conveniently supplied by the
made in a separate independent action brought for the stipulations between the parties in the pre-trial
purpose. The guilt of the donee (beneficiary) may be conference of the case. It case agreed upon and
proved by preponderance of evidence in the same stipulated therein that the deceased insured
proceeding (the action brought to declare the nullity of Buenaventura C. Ebrado was married to Pascuala
the donation). Ebrado with whom she has six legitimate children; that
during his lifetime, the deceased insured was living
Since it is agreed in their stipulation during the pre- with his common-law wife, Carponia Ebrado, with
trial that the deceased insured and defendant Carponia whom he has two children. These stipulations are
T. Ebrado were living together as husband and wife nothing less than judicial admissions which, as a
without being legally married and that the marriage of consequence, no longer require proof and cannot be
the insured with the other defendant Pascuala Vda. de contradicted. A fortiori, on the basis of these
Ebrado was valid and still existing at the time the admissions, a judgment may be validly rendered
insurance in question was purchased there is no without going through the rigors of a trial for the sole
question that defendant Carponia T. Ebrado is purpose of proving the illicit liaison between the
disqualified from becoming the beneficiary of the policy insured and the beneficiary. In fact, in that pretrial,
in question and as such she is not entitled to the the parties even agreed "that a decision be rendered
proceeds of the insurance upon the death of the based on this agreement and stipulation of facts as to
insured. who among the two claimants is entitled to the policy."

Issue of the Case: ACCORDINGLY, the appealed judgment of the lower


court is hereby affirmed. Carponia T. Ebrado is hereby
Can a common-law wife named as beneficiary in the declared disqualified to be the beneficiary of the late
life insurance policy of a legally married man claim the Buenaventura C. Ebrado in his life insurance policy. As
proceeds thereof in case of death of the latter? a consequence, the proceeds of the policy are hereby
held payable to the estate of the deceased insured.
Ruling: Costs against Carponia T. Ebrado.

The SC affirmed the decision of the trial court. SO ORDERED.

under Article 2012 of the same Code, "any person who


is forbidden from receiving any donation under Article
739 cannot be named beneficiary of a fife insurance SOUTHERN LUZON EMPLOYEES ASSN. V. GOLPEO
policy by the person who cannot make a donation to Digested by Margaret Frances Aparte
him. Common-law spouses are, definitely, barred from
receiving donations from each other. Article 739 of the Note:
new Civil Code provides: The following donations shall
be void: A common law wife of the insured who has a legal wife
1. Those made between persons who were guilty of is disqualified as beneficiary. It is not required that
adultery or concubinage at the time of donation; there be a previous conviction for adultery or
2. Those made between persons found guilty of the concubinage for the prohibition to apply. However, in
same criminal offense, in consideration thereof; an earlier case (such as the present case), the
3. Those made to a public officer or his wife, common-law wife designated prevailed over the legal
descendants or ascendants by reason of his office. wife because the case took place while the Old Civil
Code was still applicable, under which there was no
In the case referred to in No. 1, the action for provision similar to Art.2012.
declaration of nullity may be brought by the spouse of
the donor or donee; and the guilt of the donee may be FACTS:
proved by preponderance of evidence in the same
action. Southern Luzon Employees' Association is composed of
laborers and employees of Laguna tayabas Bus Co.,
The underscored clause neatly conveys that no and Batangas Transportation Company, and one of its
criminal conviction for the offense is a condition purposes is mutual aid of its members and their
precedent. In fact, it cannot even be from the defendants in case of death.
aforequoted provision that a prosecution is needed. On
the contrary, the law plainly states that the guilt of the Roman A. Concepcion was a member until his death on
party may be proved "in the same acting for December 13, 1950. In the form required by the
declaration of nullity of donation. And, it would be association to be accomplished by its members, with
sufficient if evidence preponderates upon the guilt of reference to the death benefit, Roman A. Concepcion
the consort for the offense indicated. The quantum of listed as his beneficiaries Aquilina Maloles, Roman M.
proof in criminal cases is not demanded.

EH 403 2010-2011 2011-2012


Page 8
INSURANCE LAW CASE DIGESTS
Concepcion, Jr., Estela M. Concepcion, Rolando M. The court disagrees with this contention. The contract
Concepcion and Robin M. Concepcion. of life insurance is a special contract and the
destination of the proceeds thereof is determined by
After the death of Roman A. Concepcion, the special laws which deal exclusively with that subject.
association was able to collect voluntary contributions The Civil Code has no provisions which relate directly
from its members amounting to P2,505. Three sets of and specifically to life-insurance contract or to the
claimants presented themselves, namely, (1) Juanita destination of life-insurance proceeds. That subject is
Golpeo, legal wife of Roman A. Concepcion, and her regulate exclusively by the Code of Commerce which
children; (2) Aquilina Maloles, common law wife of provides for the terms of the contract, the relations of
Roman A. Concepcion, and her children, named the parties and the destination of the proceeds of the
beneficiaries by the deceased; and (3) Elsie Hicban, policy.
another common law wife of Roman A. Concepcion,
and her child.
Digested by: Kwin Asunto
The court rendered a decision, declaring the SOCIAL SECURITY SYSTEM vs. CANDELARIA D.
defendants Aquiliana Malolos and her children the sole DAVAC
beneficiaries of the sum of P2,505.00 and ordering the [G.R. No. L-21642. July 30, 1966.]
plaintiff to deliver said amount to them.
Facts:
ISSUE:
This is an appeal from the resolution of the Social
WHETHER OR NOT THE COURT COMMITED ERROR IN Security Commission declaring respondent Candelaria
DESIGNATING A COMMON LAW WIFE OF AN INSURED Davac as the person entitled to receive the death
AS THE BENEFICIARY INSTEAD OF THE LEGAL WIFE. benefits payable for the death of Petronilo Davac.

Remember: This case took place while the Old Civil The late Petronilo Davac, a former employee of Lianga
Code was still applicable. Bay Logging Co. Inc. became a member of the Social
Security System (SSS for short) on September 1,
HELD: Judgment affirmed. 1957. He designated respondent Candelaria Davac as
his beneficiary and indicated his relationship to her as
The decision is based mainly on the theory that the that of "wife". When he died, each of the respondents
contract between the plaintiff and the deceased Roman (Candelaria Davac and Lourdes Tuplano) filed their
A. Concepcion partook of the nature of an insurance claims for death benefit with the SSS. It appears from
and that, therefore, the amount in question belonged their respective claims and the documents submitted
exclusively to the beneficiaries, invoking the following in support thereof, that the deceased contracted two
pronouncements of this Court in the case of Del Val vs. marriages, the first, with claimant Lourdes Tuplano on
Del Val, 29 Phil., 534: August 29, 1946, who bore him a child, Romeo Davac
With the finding of the trial court that the proceeds and the second, with claimant Candelaria Davac on
of the life-insurance policy belongs exclusively to January 18, 1949, with whom he had a minor
the defendant as his individual and separate daughter, Elizabeth Davac. Due to their conflicting
property, we agree. That the proceeds of an claims, the processing thereof was held in abeyance,
insurance policy belong exclusively to the whereupon the SSS filed this petition praying that
beneficiary and not to the estate of the person respondents be required to interplead and litigate
whose life was insured, and that such proceeds are between themselves their conflicting claims over the
the separate and individual property of the death benefits in question.
beneficiary, and not of the heirs of the person
whose life was insured, is the doctrine in America. Issue:
We believe that the same doctrine obtains in these
Islands by virtue of section 428 of the Code of Whether or not the Social Security Commission acted
Commerce, which reads: correctly in declaring respondent Candelaria Davac as
"The amounts which the underwriter must deliver the person entitled to receive the death benefits in
to the person insured, in fulfillment of the contract, question.
shall be the property creditors of any kind
whatsoever of the person who effected the Held: yes.
insurance in favor of the formers."
The benefit receivable under the Social Security Act is
AS TO THE CONTENTION OF THE COUNSELS in the nature of a special privilege or an arrangement
PLAINTIFF THAT THE PROCEEDS OF THE INSURANCE secured by the law, pursuant to the policy of the State
POLICY WERE DONATION OR GIFT MADE BY THE to provide social security to the workingmen. The
FATHER DURING HIS LIFETIME, SUCH THAT UNDER amounts that may thus be received cannot be
THE CIVIL CODE ARE NOT BETTERMENTS AND SHALL considered as property earned by the member during
BE CONSIDERED AS PART OF THE LEGAL PORTION. his lifetime, and, hence, do not form part of the
properties of the conjugal partnership or of the estate
of the said member. They are disbursed from a public

EH 403 2010-2011 2011-2012


Page 9
INSURANCE LAW CASE DIGESTS
special fund created by Congress pursuant to the AQUINO, J., concurring:
declared policy of the Republic "to develop, establish
gradually and perfect a social security system which . . There may be instances, like the instant case, where in
. shall provide protection against the hazards of legal succession the estate is distributed according to
disability, sickness, old age and death." (Section 1, the rules on legitime without applying the rules on
Republic Act No. 1792.) Consequently, if there is a intestate ion. The reason is that sometimes the estate
named beneficiary and the designation is not invalid, it is not even sufficient to satisfy the legitimes. The
is not the heirs of the employee who are entitled to legitimes of the primary compulsory heirs, like a child
receive the benefits, unless they are the designated or descendant, should first be satisfied.
beneficiaries themselves. It is only when there is no
designated beneficiary or when the designation is void In this case the decedent's legal heirs are his
that the laws of succession become applicable. The legitimate child, his widow and two intimate children.
Social Security Act is not a law of succession. His estate is partitioned among those heirs by giving
them their respective time.

Digested by: Rachel R. Aying The legitimate child gets one-half of the estate as his
RE: CLAIMS FOR BENEFITS OF THE HEIRS OF THE legitime which is regarded as his share as a legal heir
LATE MARIO V. CHANLIONGCO, FIDELA B Art 888, Civil Code).
CHANLIONGCO, MARIO B. CHANLIONGCO II, MA.
ANGELINA C. BUENAVENTURA and MARIO C. The widow's legitime is one-fourth of the estate. That
CHANLIONGCO, JR. represents also her share as a legal heir. The
remaining one-fourth of the estate, which is the free
Facts: portion, goes to the illegitimate children in equal
shares, as their legitime, Pursuant to the provision that
This matter refers to the claims for retirement benefits 'the legitimate of the illegitimate children shall be
filed by the heirs of the late ATTY. MARIO V. taken from the portion of the estate at the free
CHANLIONGCO an attorney of the Court, it is in the disposal of the testator, provoked that in no case shall
records that at the time of his death, Atty. Chanliongco the total legitime of such illegitimate children exceed
was more than 63 years of age, with more than 38 that free portion, and that the legitime of the surviving
years of service in the government. He did not have spouse must first be fully satisfied.
any pending criminal administrative or not case against
him, neither did he have any money or property The rule in Santillon vs. Miranda, L-19281, June 30,
accountability. The highest salary he received was 1965, 14 SCRA 563, that when the surviving spouse
P18,700.00 per annum. concurs with only one legitimate child, the spouse is
entitled to one-half of the estate and the gets the
Aside from his widow, Dra. Fidel B. Chanliongco and an other half, t to article 996 of the Civil Code, does not
only Intimate Mario it appears that there are other apply to the case because here intimate children
deceased to namely, Mrs. Angelina C. , Jr., both born concur with the surviving spouse and the intimate
out of wedlock to Angelina R Crespo, and duly child.
recognized by the deceased. Except Mario, Jr., who is
only 17 years of age, all the claimants are of legal age. In this case, to divide the estate between the surviving
spouse and the ligitemate child that deprive the
According to law, the benefits accruing to the deceased illegitimate children of their legitime. So, the
consist of: (1) retirement benefits; (2) money value of decendent's estate is distributed in the proportion of
terminal leave; (3) life insurance and (4) refund of 1/2 for the legitimate child, 1/4 for the widow and 1/8
retirement premium. each for the two illegitimate children.
From the records now before US, it appears that the
GSIS had already the release the life insurance Also not of possible application to this case is the rule
proceeds; and the refund of rent to the claimants. that the legal of an acknowledge natural child is 1/2 of
the legitime of the legitimate child of that the of the
RULING: spurious child is 2/5 of that of the of the intimate child
or 4/5 of that of that of the acknowledged natural
The record also shows that the late Atty. Chanliongco child.
died ab intestato and that he filed or over to state in
his application for membership with the GSIS the The rule be applied because the estate is not sufficient
beneficiary or benefits of his retirement benefits, to cover legitimes of all compulsory heirs. That is one
should he die before retirement. Hence, the retirement of the flaws of the law of succession. A situation as in
benefits shall accrue to his estate and will be the instant case may arise where the illegitimate
distributed among his Legal heirs in with the benefits children get less than their legitime. With respect to
on intestate s , as in the caw of a fife if no benefit is the decendant's unpaid salary and the money value of
named in the policy (Vda. de vs. GSIS, L-28093, Jan. his leave, the same are conjugal properties because of
30, 1971, 37 SCRA 315, 325). the rule that property "obtained by the or work, or as
salary of the spouses, or either of them", is conjugal in
character.

EH 403 2010-2011 2011-2012


Page 10
INSURANCE LAW CASE DIGESTS
By virtue of the contract between Tomas B. Lianco and
the Archbishop, Lianco erected the building of which
iii. Forfeiture of beneficiarys interest the premises in question form part and became owner
(Section 12) thereof . He transferred the ownership of the premises
iv. Insurable Interest in property (Sections in question to kaw Eng Si, who in turn transferred it to
13, 14, 15, 16, 17, 18) plaintiff Juan Golangco .Lianco and the actual occupant
of the premises acknowledged plaintiff's right to collect
TRADERS INSURANCE & SURETY CO vs. JUAN rentals thereon in a compromise agreement which was
GOLANGCO Y OTRA incorporated in a judicial judgment. Both at the time of
[Sep 21, 1954, En Banc G.R. No. L-6442] the issuance of the policy and at the time of the fire,
plaintiff Golangco was in legal possession of the
(I really had a hard time deciphering the facts of this premises, collecting rentals from its occupant. It seems
case as it was written in Spanish ( facts from the lower plain that if the premises were destroyed - as they
court as adopted by SC), but I assure you I have full were - by fire, Golangco would be, as he was, directly
faith that this digest is as accurate corresponding to damnified thereby; and hence he had an insurable
the full text). interest therein (section 13, Insurance Law).

Facts: It is to be noted that the policy so worded indicates


that the fire insurance policy includes 'rent or other
Tomas Lianco and the Archbishop ( no name indicated) subject matter of insurance in respect of or in
entered into a contract of lease on a parcel of land connection with any building or any property contained
owned by church . As lessee, Lianco erected a building in any building'. The argument of Traders Insurance
on the leased portion of the churchs land. Lianco later that a policy of insurance must specify the interest of
transferred ownership of this building to Kaw Eng Si, the insured in the property insured, if he is not the
who later transferred the same to Golangco. This absolute owner thereof, is not meritorious because it
transfers by Lianco of his right to lease and the was the Traders, not Golangco, who prepared that
buildingownership were without consent of the policy, and it cannot take advantage of its own acts to
Archbishop. The Archbishop filed an ejectment case plaintiff's detriment; and, in any case, this provision
against Lianco, who appears to be occupants of the was substantially complied with by Golangco when he
premises building with others paying rent to Golangco. made a full and clear statement of his interests to
This right of Golangco to receive rent on the building Trader's manager.
was judicially recognized in a case decided between
Lianco and some others occupying the premises The contract between Lianco and the Archbishop only
pursuant to a compromise agreement. At the moment, forbade Lianco from transferring 'his rights as LESSEE
the Archbishop did not exercise his option to question but the contracts Lianco made in favor of Kaw Eng Si
Golangcos rights as lessee, as the transfer by Lianco and plaintiff Golangco did not transfer such rights; and
was without the Archbishops consent. On April 7, hence no written consent thereto was necessary. At
1949,Golangco applied for fire insurance with Traders worst, the contract would be voidable, but not a void
Insurance and Surety Co. of which Golangco was contract, at the option of the Archbishop; but this
issued fire insurance policy stating that all insurance would not deprive Golangco of his insurable interest
covered under said policy, includes the 'rent or other until such option were exercised; and it does not
subject matter of insurance in respect of or in appear that it was ever exercised.
connection with any building or any property contained
in any building'. On June 5, 1949, fire ravaged the The ejectment case filed by the Archbishop against
building premises pursuant of which Golangco Lianco did not remove nor destroy plaintiff's insurable
requested Traders Insurance to pay the insurance interest: first, because plaintiff was not a party thereto
amount of 10,000 including the amount of rent P1,100 and cannot be bound thereby; and second, because
monthly. Traders insurance refused to pay the the judgment of the Municipal Court, at least as late as
insurance as pertaining to the rent averring that February 14, 1950, had not been executed so far as
Golangco has no insurable interest therein. possession of the premises were concerned; so that,
as far as plaintiff Golangco was concerned, his right to
Issue: the premises and to the rentals thereon continued to
exist on June 5, 1949 when the fire took place."
WON Golangco has insurable interest ( in the property)
on the rent of the building premises which may
lawfully/validly be subject of insurance? FILIPINO MERCHANTS INSURANCE CO., INC., vs.
COURT OF APPEALS and CHOA TIEK SENG
Ruling: [G.R. No. 85141 November 28, 1989]

Yes, Sec. 13 of the Insurance Code provides that FACTS:


Every interest in the property, whether real or
personal, or any relation thereto, or liability in respect 1. CHOA TIEK SENG was the consignee of the shipment
thereof of such nature that a contemplated peril might of fishmeal loaded on board a vessel and unloaded at
directly damnify the insured, is an insurable interest. the Port of Manila. The goods were described as 600

EH 403 2010-2011 2011-2012


Page 11
INSURANCE LAW CASE DIGESTS
metric tons of fishmeal in new gunny bags of 90 kilos
each. He insured said shipment with FILIPINO Insurable interest in property may consist in:
MERCHANTS insurance company from Bangkok, (a) an existing interest;
Thailand to Manila against all risks. (b) an inchoate interest founded on an existing
2. When the fishmeal were unloaded from the ship at interest; or
Manila unto the arrastre contractor and surveyor, it (c) an expectancy, coupled with an existing interest in
was ascertained and certified that in such discharge that out of which the expectancy arises.
105 bags were in bad order condition. The extent of
shortage or loss totalling 227 bags amounting to The contract of shipment, whether under F.O.B.,
12,148 kilos was noted in the Bad Order Certificate. C.I.F., or C. & F. as in this case, is immaterial in the
3. The consignee made a formal claim against the determination of whether the vendee has an insurable
Insurance Company for P51,568.62. The latter refused interest or not in the goods in transit. The perfected
to pay the claim so the consignee brought an action contract of sale even without delivery vests in the
against the insurance company. vendee an equitable title, an existing interest over the
4. The trial court rendered judgment, which was affirmed goods sufficient to be the subject of insurance.
by the Court of appeals, in favor of the consignee,
ordering the insurance company to pay the former the 2. No. An "all risks policy" should be read literally as
sum of P51,568.62. meaning all risks whatsoever and covering all losses
5. The insurance company claimed that under the "all by an accidental cause of any kind. The very nature
risks" clause of the marine insurance policy it is not of the term "all risks" must be given a broad and
liable to the consignee for the partial loss of the cargo comprehensive meaning as covering any loss other
because of the absence of proof of some fortuitous than a willful and fraudulent act of the insured.
event, casualty, or accidental cause to which the loss
is attributable. The insurance company argues that the The term "all risks" cannot be given a strained
consignee has the burden to adduce evidence, which it technical meaning, to the effect that it extends to all
failed to do, that the alleged loss to the cargo was due damages/losses suffered by the insured cargo except:
to a fortuitous event. (a) loss or damage or expense proximately caused by
To put at rest all doubts on the matter, the Court delay, and
discussed on the issue of lack of insurable interest, (b) loss or damage or expense proximately caused by
despite being raised by the insurance company for the the inherent vice or nature of the subject matter
first time on appeal. insured.

ISSUES: Purpose of All Risks insurance


The very purpose of all risks" insurance is to give
1. Whether or not the consignee had an insurable protection to the insured in those cases where
interest difficulties of logical explanation or some mystery
2. Whether or not there must be some fortuitous event surround the loss or damage to property.
for the claim to be compensable under all risks
marine insurance policy An "all asks" policy has been evolved to grant greater
protection than that afforded by the "perils clause," in
RULING: order to assure that no loss can happen through the
incidence of a cause neither insured against nor
1. Yes, the consignee had an insurable interest in said creating liability in the ship; it is written against all
goods which may be a valid subject of a contract of losses, that is, attributable to external causes.
insurance.
Burden of Proof
His interest over the goods is based on the perfected Generally, the burden of proof is upon the insured to
contract of sale. The perfected contract of sale show that a loss arose from a covered peril, but under
between him and the shipper of the goods operates to an "all risks" policy the burden is not on the insured to
vest in him an equitable title even before delivery or prove the precise cause of loss or damage for which it
before be performed the conditions of the sale. seeks compensation.

Section 13 of the Insurance Code defines insurable The insured only has the initial burden of proving that
interest in property as every interest in property, the cargo was in good condition when the policy
whether real or personal, or any relation thereto, or attached and that the cargo was damaged when
liability in respect thereof, of such nature that a unloaded from the vessel; thereafter, the burden then
contemplated peril might directly damnify the insured. shifts to the insurer to show the exception to the
coverage.

In principle, anyone has an insurable interest in The burden of the insured, therefore, is to prove
property who derives a benefit from its existence or merely that the goods he transported have been lost,
would suffer loss from its destruction whether he has destroyed or deteriorated. Thereafter, the burden is
or has not any title in, or lien upon or possession of shifted to the insurer to prove that the loss was due to
the property. excepted perils. To impose on the insured the burden

EH 403 2010-2011 2011-2012


Page 12
INSURANCE LAW CASE DIGESTS
of proving the precise cause of the loss or damage share in the loss but the same was refused. Hence,
would be inconsistent with the broad protective complainants demanded from the other three
purpose of "all risks" insurance. respondents the balance of each share in the loss but
the same was also refused. Petitioner then filed a
As held in Paris-Manila Perfumery Co. vs. Phoenix complaint in intervention claiming the proceeds of the
Assurance Co., Ltd. the basic rule is that the fire insurance policy issued by respondent Travellers
insurance company has the burden of proving that the Multi-Indemnity but the latter alleged that it is not
loss is caused by the risk excepted and for want of entitled to indemnity for lack of insurable interest
such proof, the company is liable. before the loss of the insured premises.

Special type of insurance Issue:


Coverage under an "all risks" provision of a marine
insurance policy creates a special type of insurance Whether or not petitioner has an insurable interest.
which extends coverage to risks not usually
contemplated and avoids putting upon the insured the Ruling:
burden of establishing that the loss was due to the
peril falling within the policy's coverage. The insurer Respondent insurance company did not assail the
can avoid coverage upon demonstrating that a specific validity of the insurance policy taken out by petitioner
provision expressly excludes the loss from coverage. over the mortgaged property. Neither did it deny that
the said property was totally razed by fire within the
A marine insurance policy providing that the insurance period covered by the insurance. Respondent advanced
was to be "against all risks" must be construed as an affirmative defense of lack of insurable interest on
creating a special insurance and extending to other the part of the petitioner that before the occurrence of
risks than are usually contemplated, and covers all the peril insured against the Palomos had already paid
losses except such as arise from the fraud of the their credit due the petitioner. Respondent having
insured. admitted the material allegations in the complaint has
the burden of proof to show that petitioner has no
In the present case, there being no showing that the insurable interest over the insured property at the time
loss was caused by any of the excepted perils i.e. the contingency took place. Upon that point, there is a
delay or the inherent vice or nature of the subject failure of proof. Respondent exerted no effort to
matter insured, the insurer is liable under the policy. present any evidence to substantiate its claim, while
petitioner did. For said respondent's failure, the
decision must be adverse to it.
v. When insurable interest must exist
(Section 19) However, respondent Insurance Commission absolved
respondent insurance company from liability on the
Digested by: Birondo basis of the certification issued by the then Court of
Tai Tong Chuache & Co. v. Insurance Commission First Instance of Davao, Branch II, that in a certain
civil action against the Palomos, Arsenio Lopez Chua
Facts: stands as the complainant and not Tai Tong Chuache.
From said evidence respondent commission inferred
Complainants acquired a parcel of land and a building that the credit extended by herein petitioner to the
they assumed the mortgage of the latter in favor of Palomos secured by the insured property must have
SSS, which was insured with respondent SSS been paid. Such is a glaring error which cannot be
Accredited Group of Insurers. On April 19, 1975, sanctioned. Respondent Commission's findings are
Azucena Palomo obtained a loan from petitioner Tai based upon a mere inference.
Tong Chuache Inc. securing it with a mortgage was
executed over the land and the building in favor of The record of the case shows that the petitioner to
petitioner. On April 25, 1975, Arsenio support its claim for the insurance proceeds offered as
Chua, petitioners representative insured the latter's evidence the contract of mortgage which has not been
interest with Travellers Multi-Indemnity Corporation. cancelled nor released. It has been held in a long line
On June 11, 1975, Pedro Palomo secured fire of cases that when the creditor is in possession of the
insurance covering the building with respondent Zenith document of credit, he need not prove non-payment
Insurance Corporation and on July 16, 1975, another for it is presumed. The validity of the insurance policy
fire insurance was procured from respondent Philippine taken by petitioner was not assailed by private
British Assurance Company, covering the same respondent. Moreover, petitioner's claim that the loan
building and the contents thereof. However, on July extended to the Palomos has not yet been paid was
31, 1975, the building and the contents were totally corroborated by Azucena Palomo who testified that
razed by fire. they are still indebted to herein petitioner. Public
respondent argues however, that if the civil case really
Respondents, Zenith Insurance, Phil. British Assurance stemmed from the loan granted to Azucena Palomo by
and SSS Accredited Group of Insurers, paid their petitioner the same should have been brought by Tai
corresponding shares of the loss. Demand was made Tong Chuache or by its representative in its own
from respondent Travellers Multi-Indemnity for its behalf. From the above premise respondent concluded

EH 403 2010-2011 2011-2012


Page 13
INSURANCE LAW CASE DIGESTS
that the obligation secured by the insured property of Pacific Life. Upon the payment of the insurance
must have been paid. premium, the binding deposit receipt was issued to
private respondent Ngo Hing. Likewise, petitioner
The premise is correct but the conclusion is wrong. Mondragon handwrote at the bottom of the back page
Citing Rule 3, Sec. 2 respondent pointed out that the of the application form his strong recommendation for
action must be brought in the name of the real party in the approval of the insurance application. Then on April
interest. Correct! However, it should be borne in mind 30, 1957, Mondragon received a letter from Pacific Life
that petitioner being a partnership may sue and be disapproving the insurance application. The letter
sued in its name or by its duly authorized stated that the said life insurance application for 20-
representative. The fact that Arsenio Lopez Chua is the year endowment plan is not available for minors below
representative of petitioner is not questioned. seven years old, but Pacific Life can consider the same
Petitioner's declaration that Arsenio Lopez Chua acts as under the Juvenile Triple Action Plan, and advised that
the managing partner of the partnership was if the offer is acceptable, the Juvenile Non-Medical
corroborated by respondent insurance company. Thus Declaration be sent to the company.
Chua as the managing partner of the partnership may
execute all acts of administration including the right to The non-acceptance of the insurance plan by Pacific
sue debtors of the partnership in case of their failure to Life was allegedly not communicated by petitioner
pay their obligations when it became due and Mondragon to private respondent Ngo Hing. Instead,
demandable. Or at the very least, Chua being a on May 6, 1957, Mondragon wrote back Pacific Life
partner of petitioner Tai Tong Chuache & Company is again strongly recommending the approval of the 20-
an agent of the partnership. Being an agent, it is year endowment insurance plan to children, pointing
understood that he acted for and in behalf of the out that since 1954 the customers, especially the
firm. Public respondent's allegation that the civil case Chinese, were asking for such coverage.
filed by Arsenio Chua was in his capacity as personal
creditor of spouses Palomo has no basis. It was when things were in such state that on May 28,
1957 Helen Go died of influenza with complication of
The respondent insurance company having issued a bronchopneumonia. Thereupon, private respondent
policy in favor of herein petitioner which policy was of sought the payment of the proceeds of the insurance,
legal force and effect at the time of the fire, it is bound but having failed in his effort, he filed the action for
by its terms and conditions. Upon its failure to prove the recovery of the same before the Court of First
the allegation of lack of insurable interest on the part Instance of Cebu, which rendered the adverse decision
of the petitioner, respondent insurance company is and as earlier referred to against both petitioners.
must be held liable.
ISSUE:

vi. Rules in change of interest (Sections 20, Whether the respondent is entitled to the insurance
21, 22, 23, 24)
vii. Void Stipulations (Section 25) SC:

D. Concealment Since petitioner Pacific Life disapproved the insurance


i. Definition (Sections 26, 27) application of respondent Ngo Hing, the binding
deposit receipt in question had never become in force
Digested by: Lo, Justine at any time.
GREAT PACIFIC LIFE ASSURANCE COMPANY vs.
HONORABLE COURT OF APPEALS As held by this Court, where an agreement is made
[G.R. No. L-31845 April 30, 1979] between the applicant and the agent, no liability shall
attach until the principal approves the risk and a
FACTS: receipt is given by the agent. The acceptance is merely
conditional and is subordinated to the act of the
It appears that on March 14, 1957, private respondent company in approving or rejecting the application.
Ngo Hing filed an application with the Great Pacific Life Thus, in life insurance, a "binding slip" or "binding
Assurance Company (hereinafter referred to as Pacific receipt" does not insure by itself
Life) for a twenty-year endowment policy in the
amount of P50,000.00 on the life of his one-year old It bears repeating that through the intra-company
daughter Helen Go. Said respondent supplied the communication of April 30, 1957 Pacific Life
essential data which petitioner Lapulapu D. disapproved the insurance application in question on
Mondragon, Branch Manager of the Pacific Life in Cebu the ground that it is not offering the twenty-year
City wrote on the corresponding form in his own endowment insurance policy to children less than
handwriting. Mondragon finally type-wrote the data on seven years of age. What it offered instead is another
the application form which was signed by private plan known as the Juvenile Triple Action, which private
respondent Ngo Hing. The latter paid the annual respondent failed to accept. In the absence of a
premium the sum of P1,077.75 going over to the meeting of the minds between petitioner Pacific Life
Company, but he retained the amount of P1,317.00 as and private respondent Ngo Hing over the 20-year
his commission for being a duly authorized agent endowment life insurance in the amount of P50,000.00

EH 403 2010-2011 2011-2012


Page 14
INSURANCE LAW CASE DIGESTS
in favor of the latter's one-year old daughter, and with binding receipt, and concealment, as legally defined,
the non-compliance of the above quoted conditions having been combated by herein private respondent.
stated in the disputed binding deposit receipt, there
could have been no insurance contract duly perfected
between then Acordingly, the deposit paid by private ii. Concealment of Material Fact in bad
respondent shall have to be refunded by Pacific Life. faith (Section 28)

We are not impressed with private respondent's SEGUNDINA MUSGI, ET AL., vs. WEST COAST
contention that failure of petitioner Mondragon LIFE INSURANCE CO.
to communicate to him the rejection of the [G.R. No. L-41794 August 30, 1935]
insurance application would not have any
adverse effect on the allegedly perfected Facts:
temporary contract In this first place, there was
no contract perfected between the parties who The plaintiffs, as beneficiaries, brought suit against the
had no meeting of their minds. Private defendant to recover the value of two life insurance
respondent, being an authorized insurance agent policies. Arsenio T. Garcia was insured by the
of Pacific Life at Cebu branch office, is defendant company in the sum of P5, 000. Arsenio T.
indubitably aware that said company does not Garcia was again insured by the defendant company in
offer the life insurance applied for. When he filed the sum of P10,000. Subsequently, Arsenio died. Even
the insurance application in dispute, private with the demand made by the plaintiffs to the
respondent was, therefore, only taking the chance that defendant company to pay the two policies, defendant
Pacific Life will approve the recommendation of refused to pay
Mondragon for the acceptance and approval of the
application in question along with his proposal that the It is to be noted that in both applications, the insured
insurance company starts to offer the 20-year had to answer inquiries as to his state of health and
endowment insurance plan for children less than seven that of his family, which he did voluntarily. In each of
years. Nonetheless, the record discloses that Pacific the said applications the following question was asked:
Life had rejected the proposal and recommendation. "1. What physician or practitioner or any other person
Secondly, having an insurable interest on the life of his not named above have you consulted or been treated
one-year old daughter, aside from being an insurance by, and for what illness, or ailment? (If none, so
agent and an offense associate of petitioner state.)" In the first application, the insured answered
Mondragon, private respondent Ngo Hing must have "None", and in the second, "No". These answers of the
known and followed the progress on the processing of insured as well as his other statements contained in his
such application and could not pretend ignorance of applications were one of the causes or considerations
the Company's rejection of the 20-year endowment life for the issuance of the policies, and they so positively
insurance application. appear therein. After the death of the insured and as a
result of the demand made by the beneficiaries upon
This Court is of the firm belief that private respondent the defendant to pay the value of the policies, the
had deliberately concealed the state of health and latter discovered that the aforementioned answers
physical condition of his daughter Helen Go. Where were false and fraudulent, because the truth was that
private respondent supplied the required the insured, before answering and signing the
essential data for the insurance application form, applications and before the issuance of the policies,
he was fully aware that his one-year old had been treated in the General Hospital by a lady
daughter is typically a mongoloid child. Such a physician for different ailments.
congenital physical defect could never be
ensconced nor distinguished. Nonetheless, The defendant contends that the two policies did not
private respondent, in apparent bad faith, create any valid obligation because they were
withheld the fact material to the risk to be fraudulently obtained by the insured.
assumed by the insurance company. As an
insurance agent of Pacific Life, he ought to know, Issue:
as he surely must have known. his duty and
responsibility to such a material fact. Had he Whether the two answers given by the insured in his
diamond said significant fact in the insurance applications are false, and if they were the cause, or
application form Pacific Life would have verified the one of the causes, which induced the defendant to
same and would have had no choice but to disapprove issue the policies?
the application outright.
Ruling:
Whether intentional or unintentional the concealment
entitles the insurer to rescind the contract of The concealment and the false statements constituted
insurance. Private respondent appears guilty thereof. fraud because the defendant by reason thereof
accepted the risk which it would otherwise have flatly
We are thus constrained to hold that no insurance refused. When not otherwise specially provided for by
contract was perfected between the parties with the the Insurance Law, the contract of life insurance is
noncompliance of the conditions provided in the governed by the general rules of the civil law regarding

EH 403 2010-2011 2011-2012


Page 15
INSURANCE LAW CASE DIGESTS
contracts. Article 1261 of the Civil Code provides that
there is no contract unless there should be, in addition The deceased answered question No. 5(a) in the
to consent and a definite object, a consideration for affirmative but limited his answer to a consultation
the obligation established. And article 1276 provides with a certain Dr. Reinaldo D. Raymundo of the
that the statement of a false consideration shall render Chinese General Hospital on February 1986, for cough
the contract void. The two answers being one of the and flu complications. The other questions were
considerations of the policies, and it appearing that answered in the negative.
they are false and fraudulent, it is evident that the
insurance contracts were null and void and did not give Petitioner discovered that two weeks prior to his
rise to any right to recover their value or amount. A application for insurance, the insured was examined
similar case was already decided by this court in and confined at the Lung Center of the Philippines,
Argente vs. West Coast Life Insurance Co. (51 Phil., where he was diagnosed for renal failure. During his
725). confinement, the deceased was subjected to urinalysis,
ultra-sonography and hematology tests.
In discussing the legal phase of the case, this court
said: Respondent Bernarda Bacani and her husband,
respondent Rolando Bacani, filed an action for specific
One ground for the rescission of a contract of performance against petitioner with the Regional Trial
insurance under the Insurance Act is a "concealment", Court. Petitioner filed its answer with counterclaim and
which in section 25 is defined as "A neglect to a list of exhibits consisting of medical records furnished
communicate that which a party knows and ought to by the Lung Center of the Philippines.
communicate".
Petitioner filed its Request for Admissions relative to
In view of the foregoing, appellant's first two the authenticity and due execution of several
assignments of error are well founded, wherefore, the documents as well as allegations regarding the health
appealed judgment is reversed and the defendant of the insured. Private respondents failed to oppose
absolved from the complaint. said request or reply thereto, thereby rendering an
admission of the matters alleged.
In ruling for private respondents, the trial court
Digested by: Brian O. Dalanon concluded that the facts concealed by the insured were
Sunlife Assurance Company of Canada vs. CA made in good faith and under a belief that they need
not be disclosed. Moreover, it held that the health
Facts: history of the insured was immaterial since the
insurance policy was "non-medical".
On April 15, 1986, Robert John B. Bacani procured a
life insurance contract for himself from petitioner at Petitioner appealed to the Court of Appeals, which
P100,000.00 with double indemnity in case of affirmed the decision of the trial court. The appellate
accidental death. The designated beneficiary was his court ruled that petitioner cannot avoid its obligation
mother, respondent Bernarda Bacani. by claiming concealment because the cause of death
was unrelated to the facts concealed by the insured. It
On June 26, 1987, the insured died in a plane crash. also sustained the finding of the trial court that
Respondent Bernarda Bacani filed a claim with matters relating to the health history of the insured
petitioner, seeking the benefits of the insurance policy were irrelevant since petitioner waived the medical
taken by her son. Petitioner conducted an investigation examination prior to the approval and issuance of the
and its findings prompted it to reject the claim. insurance policy. Moreover, the appellate court agreed
with the trial court that the policy was "non-medical.
In its letter, petitioner informed respondent Bernarda
Bacani, that the insured did not disclose material facts Issue:
relevant to the issuance of the policy, thus rendering
the contract of insurance voidable. A check Whether or not the insured concealed informations in
representing the total premiums paid in the amount of the application of his policy and whether or not the
P10,172.00 was attached to said letter. petitioner has the right to rescind the contract.

Petitioner claimed that the insured gave false Ruling:


statements in his application when he answered the
following questions: Section 26 of The Insurance Code is explicit in
5. Within the past 5 years have you: requiring a party to a contract of insurance to
a) consulted any doctor or other health communicate to the other, in good faith, all facts
practitioner? within his knowledge which are material to the contract
b) submitted to: EGG, X-rays, blood tests or other and as to which he makes no warranty, and which the
tests? other has no means of ascertaining. Said Section
c) attended or been admitted to any hospital or provides: A neglect to communicate that which a party
other medical facility? knows and ought to communicate, is called
b) urine, kidney or bladder disorder? concealment.

EH 403 2010-2011 2011-2012


Page 16
INSURANCE LAW CASE DIGESTS
(1) I have not been confined in any hospital,
The terms of the contract are clear. The insured is sanitarium or infirmary, nor receive any medical or
specifically required to disclose to the insurer matters surgical advice/attention within the last five (5)
relating to his health. years.
(2) I have never been treated nor consulted a
The information which the insured failed to disclose physician for a heart condition, high blood
were material and relevant to the approval and pressure, cancer, diabetes, lung, kidney, stomach
issuance of the insurance policy. The matters disorder, or any other physical impairment.
concealed would have definitely affected petitioner's (3) I am, to the best of my knowledge, in good
action on his application, either by approving it with health.
the corresponding adjustment for a higher premium or EXCEPTIONS:
rejecting the same. Moreover, a disclosure may have Xxx
warranted a medical examination of the insured by
petitioner in order for it to reasonably assess the risk Issue:
involved in accepting the application.
Whether or not there is a material concealment
Thus, "goad faith" is no defense in concealment. The
insured's failure to disclose the fact that he was Held:
hospitalized for two weeks prior to filing his application
for insurance, raises grave doubts about his bonafides. There is a material concealment.
It appears that such concealment was deliberate on his
part. On appeal by Great Pacific, the Court of Appeals
reversed and set aside the decision of the Insurance
We, therefore, rule that petitioner properly exercised Commissioner and dismissed Thelma Canilang's
its right to rescind the contract of insurance by reason complaint and Great Pacific's counterclaim. The Court
of the concealment employed by the insured. of Appeals found that the failure of Jaime Canilang to
disclose previous medical consultation and treatment
constituted material information which should have
iii. Concealment of Matters proving falsity been communicated to Great Pacific to enable the
of a warranty (Section 29) latter to make proper inquiries.
iv. What need not be disclosed unless
required (Sections 30, 32-35) Canilang failed to disclose, under the caption
v. Test of materiality in concealment "Exceptions," that he had twice consulted Dr. Claudio
(Section 31) who had found him to be suffering from "sinus
tachycardia" and "acute bronchitis."

Digested by: Grace Jayne Dingal The provisions of P.D. No. 1460, also known as the
Vda. DE CANILANG v. COURT OF Insurance Code of 1978 read as follows:
Sec. 26. A neglect to communicate that which a
Facts: party knows and ought to communicate, is called a
concealment. xxx xxx xxx
On 18 June 1982, Jaime Canilang consulted Dr. Sec. 28. Each party to a contract of insurance
Claudio and was diagnosed as suffering from "sinus must communicate to the other, in good faith, all
tachycardia And was latter found to have "acute factors within his knowledge which are material to
bronchitis." On next day, Jaime applied for a "non- the contract and as to which he makes no
medical" insurance policy with respondent Great Pacific warranty, and which the other has not the means
Life Assurance naming his wife, Thelma Canilang, as of ascertaining.
his beneficiary. Jaime was issued ordinary life
insurance Policy effective as of 9 August 1982. On 5 The information concealed must be information which
August 1983, Jaime Canilang died of "congestive heart the concealing party knew and "ought to [have]
failure," "anemia," and "chronic anemia." Petitioner, communicate[d]," that is to say, information which
widow and beneficiary of the insured, filed a claim with was "material to the contract." The test of materiality
Great Pacific which the insurer denied upon the ground is contained in Section 31 of the Insurance Code of
that the insured had concealed material information 1978 which reads:
from it. Petitioner filed a complaint against Great Sec. 31. Materially is to be determined not by the
Pacific with the Insurance Commission for recovery of event, but solely by the probable and reasonable
the insurance proceeds. During the hearing called by influence of the facts upon the party to whom the
the Insurance Commissioner, petitioner testified that communication is due, in forming his estimate of
she was not aware of any serious illness suffered by the disadvantages of the proposed contract, or in
her late husband. making his inquiries.

The medical declaration which was set out in the The information which Jaime Canilang failed to disclose
application for insurance executed by Jaime Canilang was material to the ability of Great Pacific to estimate
read as follows: xxxx the probable risk he presented as a subject of life

EH 403 2010-2011 2011-2012


Page 17
INSURANCE LAW CASE DIGESTS
insurance. Had Canilang disclosed his visits to his been intentional rather than merely inadvertent. For
doctor in the insurance application, it may be Jaime Canilang could not have been unaware that his
reasonably assumed that Great Pacific would have heart beat would at times rise to high and alarming
made further inquiries and would have probably levels and that he had consulted a doctor twice in the
refused to issue a non-medical insurance policy or, at 2 months before applying for non-medical insurance.
the very least, required a higher premium for the same The last medical consultation took place just the day
coverage. The materiality of the information withheld before the insurance application was filed. Jaime
by Great Pacific did not depend upon the state of mind Canilang went to visit his doctor precisely because of
of Jaime Canilang. A man's state of mind or subjective the discomfort and concern brought about by his
belief is not capable of proof in our judicial process, experiencing "sinus tachycardia."
except through proof of external acts or failure to act
from which inferences as to his subjective belief may We find it difficult to take seriously the argument that
be reasonably drawn. Neither does materiality depend Great Pacific had waived inquiry into the concealment
upon the actual or physical events which ensue. by issuing the insurance policy notwithstanding
Materiality relates rather to the "probable and Canilang's failure to set out answers to some of the
reasonable influence of the facts" upon the party to questions in the insurance application. Such failure
whom the communication should have been made, in precisely constituted concealment on the part of
assessing the risk involved in making or omitting to Canilang. Petitioner's argument, if accepted, would
make further inquiries and in accepting the application obviously erase Section 27 from the Insurance Code of
for insurance; that "probable and reasonable influence 1978.
of the facts" concealed must, of course, be determined
objectively, by the judge ultimately. It remains only to note that the Court of Appeals
The insurance Great Pacific applied for was a "non- finding that the parties had not agreed in the pretrial
medical" insurance policy. In Saturnino v. Philippine- before the Insurance Commission that the relevant
American Life Insurance Company, this Court held issue was whether or not Jaime Canilang had
that: intentionally concealed material information from the
.. . if anything, the waiver of medical examination insurer, was supported by the evidence of record, i.e.,
[in a non-medical insurance contract] renders even the Pre-trial Order itself dated 17 October 1984 and
more material the information required of the the Minutes of the Pre-trial Conference dated 15
applicant concerning previous condition of health October 1984, which "readily shows that the word
and diseases suffered, for such information "intentional" does not appear in the statement or
necessarily constitutes an important factor which definition of the issue in the said Order and Minutes."
the insurer takes into consideration in deciding
whether to issue the policy or not . . . WHEREFORE, the Petition for Review is DENIED for
lack of merit and the Decision of the Court of is
The Insurance Code of 1978 was amended by AFFIRMED.
B.P. Blg. 874. This subsequent statute modified
Section 27 of the Insurance Code of 1978 so as to read
as follows: BERNARDO ARGENTE vs. WEST COAST LIFE
Sec. 27. A concealment whether intentional or INSURANCE CO.
unintentional entitles the injured party to rescind a
contract of insurance. Facts:
On February 9, 1925, Bernardo Argente signed an
The Commissioner is wrong when it said that by application for joint insurance with his wife in the sum
deleting the phrase "intentional or unintentional," the of P2,000. The wife, Vicenta de Ocampo, signed a like
Insurance Code of 1978 (prior to its amendment by application for the same policy. Both applications, with
B.P. Blg. 874) intended to limit the kinds of the exception of the names and the signatures of the
concealment which generate a right to rescind on the applicants, were written by Jose Geronimo del Rosario,
part of the injured party to "intentional concealments." an agent for the West Coast Life Insurance Co. But all
"Intentional" and "unintentional" cancel each other out. the information contained in the applications was
The deletion of the phrase "whether intentional or furnished the agent by Bernardo Argente.
unintentional" could not have had the effect of
imposing an affirmative requirement that a Pursuant to this application, Bernado Agente and
concealment must be intentional if it is to entitle the Vicenta De Ocampo underwent a medical examination
injured party to rescind a contract of insurance. The on February 10, 1925 which was recorded in the
restoration in 1985 by B.P. Blg. 874 of the phrase Medical Examiners Report.
"whether intentional or unintentional" merely
underscored the fact that all throughout (from 1914 to On May 9, 1925, Bernardo Argente and his wife
1985), the statute did not require proof that submitted to the West Coast Life Insurance Co. an
concealment must be "intentional" in order to amended application for insurance, increasing the
authorize rescission by the injured party. amount thereof to P15,000, and asked that the policy
be dated May 15, 1925. The amended application was
The nature of the facts not conveyed to the insurer accompanied by the documents entitled "Short Form
was such that the failure to communicate must have

EH 403 2010-2011 2011-2012


Page 18
INSURANCE LAW CASE DIGESTS
Medical Report." In both of these documents appear there has or has not been a material concealment
certain questions and answers. must rest largely in all cases upon the form of the
questions propounded and the exact terms of the
On November 18, 1925, Vicenta de Ocampo died of contract.
cerebral apoplexy. Thereafter Bernardo Argente
presented a claim in due form to the West Coast Life
Insurance Co. for the payment of the sum of P15,000 IGNACIO SATURNINO vs. THE PHILIPPINE
the amount of the joint life Insurance policy. In an AMERICAN LIFE INSURANCE COMPANY
investigation conducted by the Manger of the Manila [G.R. No. L-16163, 28 February 1963]
office of the insurance company, it was found out the
the answers given by the insured in their medical FACTS:
examinations with regard to their health and previous
illness and medical attention were untrue. On May 25, Estefania Saturnino obtained a 20-year endowment
1926, wrote him to the effect that the claim was non-medical insurance. This kind of policy dispenses
rejected because the insurance was obtained through with the medical examination of the applicant usually
fraud and misrepresentation. required in ordinary life policies. However, two months
prior to the issuance of the policy, Saturnino was
Issue: operated for cancer, involving mastectomy of the right
breast. She did not make a disclosure in her
WON the alleged concealment was immaterial and application for insurance. On the contrary, she stated
insufficient to avoid the policy? that she did not have, nor had she ever had, among
other ailments listed in the application, cancer or other
Held: tumors. Sometime after, Saturnino died of pneumonia,
secondary to influenza. Appellants, who are her
One ground for the rescission of a contract of surviving husband and minor child, demanded
insurance under the Insurance Act is "a concealment," payment of the face value of the policy. The claim was
which in section 25 is defined as "A neglect to rejected and hence an action was subsequently
communicate that which a party knows and ought to instituted.
communicate." In an action on a life insurance policy
where the evidence conclusively shows that the ISSUE:
answers to questions concerning diseases were untrue,
the truth of falsity of the answers becomes the Whether or not the insured made such false
determining factor. In the policy was procured by representations of material facts as to avoid the policy
fraudulent representations, the contract of insurance
apparently set forth therein was never legally existent. HELD:
It can fairly be assumed that had the true facts been
disclosed by the assured, the insurance would never YES. The Insurance Law provides that materiality is to
have been granted. be determined not by the event, but solely by the
probable and reasonable influence of the facts upon
The basis of the rule vitiating the contract in case of the party to whom the communication is due, in
concealment is that it misleads or deceives the insurer forming his estimate of the proposed contract, or in
into accepting the risk, or accepting it at the rate of making his inquiries. The waiver of medical
premium agreed upon. The insurer, relying upon the examination renders even more material the
belief that the assured will disclose every material information required of the applicant concerning
within his actual or presumed knowledge, is misled previous condition of health and diseases suffered, for
into a belief that the circumstance withheld does not such information necessarily constitutes an important
exist, and he is thereby induced to estimate the risk factor which the insurer takes into consideration in
upon a false basis that it does not exist. It therefore deciding whether to issue the policy or not. It is logical
follows that the assurer in assuming a risk is entitled to assume that if Estefania had been properly
to know every material fact of which the assured has appraised of the insureds medical history she would at
exclusive or peculiar knowledge, as well as all material least have been made to undergo medical examination
facts which directly tend to increase the hazard or risk in order to determine her insurability.
which are known by the assured, or which ought to be
or are presumed to be known by him. And a Concealment, whether intentional or unintentional,
concealment of such facts vitiates the policy. entitles the insurer to rescind the contract of
insurance, concealment being defined as negligence
If the assured has exclusive knowledge of material to communicate that which an Insurance Law case
facts, he should fully and fairly disclose the same, party knows and ought to communicate. The basis of
whether he believes them material or not. But the rule vitiating the contract in cases of concealment
notwithstanding this general rule it will not infrequently is that it misleads or deceives the insurer into
happen, especially in life risks, that the assured may accepting the risk, or accepting it at the rate of
have a knowledge actual or presumed of material premium agreed upon. The insurer, relying upon the
facts, and yet entertain an honest belief that they are belief that the assured will disclose every material fact
not material. The determination of the point whether within his actual or presumed knowledge, is misled

EH 403 2010-2011 2011-2012


Page 19
INSURANCE LAW CASE DIGESTS
into a belief that the circumstance withheld does not compliance with the requisites and the investigation
exist, and he is thereby induced to estimate the risk carried on by the defendant company, and it had been
upon a false basis that it does not exist. satisfied concerning the physical condition of the
applicant, it accepted the application for insurance and
The judgment appealed from, dismissing the complaint on November 6, 1912, issued policy No. 5592, Exhibit
and awarding the return to appellants of the premium A, which has been made a part of the complaint,
already, paid, with interest at 6% up to January 29, whereby the said insurance company insured the life of
1959, affirmed, with costs against appellants. the said Dominador Albay in the sum of P5,000,
payable in the event of his death to Francisca Eguaras;
that on December 6, 1912, said policy No. 5592 being
E. Representation in force, the insured Dominador Albay, died in the
i. Definition, Interpretation (Sections 36- municipality of Santa Cruz, Laguna, and despite the
39) fact that the beneficiary submitted satisfactory proofs
ii. What representation qualifies or not of his death and that the defendant company
(Section 40) investigated the event, still it refused and continues to
iii. Sections 41-44 refuse to pay to the plaintiff the value of the policy,
Exhibit A, thereby causing damages estimated at
Digested by: Gestopa, Gevina V. P1,000. The court was therefore asked to render
Insular Life Ass. Co. vs. Pineda judgment against the Great Eastern Life Assurance
Company, Ltd., and its general agent, West G. Smith,
Facts: by sentencing them to pay to the plaintiff the sum of
P5,000, the value of policy No. 5592, plus the sum of
The insured, a fisherman, in his application answered P1,000 for damages inflicted upon them, in addition to
no to question of whether he drank beer or other the costs of the suit.
intoxicants, whether he ever suffered from any ailment
of the lungs and whether he had ever spat blood. The demurrer filed to the foregoing complaint having
been overruled, counsel for the insurance company
Subsequently, the insured signed a statement that he and for West G. Smith replied thereto, admitting the
did drink beer, but very seldom, that he had a chronic allegations of the complaint with respect to the legal
cough for many years and that he had experienced a status of the parties by denying all the rest, and
few sputums, slightly bloody. setting forth in special defense that the insurance
policy issued in the name of Dominador [Albay] had
Ruling: been obtained through fraud and deceit known and
consented to by the interested parties and is therefore
The policy is valid. We do not consider the allegedly completely illegal, void, and ineffective; wherefore he
conflicting statements as misrepresentations of prayed that the defendants be absolved from the
material facts. That the insured drank beer but very complaint, with the costs against the plaintiff.
seldom is almost similar to not at all.
Issue:
We are not aware of the non-insurability of a person
affected with a chronic cough. WON the life insurance obtained is legal and valid or
whether on the contrary it was issued through fraud
With respect to the few sputums slightly bloody, we and deceit, and in such case, whether the defendant,
consider the statement as not inconsistent with not The Great Eastern Life Assurance Company, Ltd., is
having spat blood, since traces of blood in insureds still under obligation to pay the value thereof to the
sputum could be due to some temporary and plaintiff.
unimportant ailments as a bloody tooth, a sore throat
or a bad cold. Ruling:

It appears from the record that the insured had


iv. Misrepresentation, effect (Section 45) knowledge of the false replied contained in the two
applications for insurance and knowing permitted fraud
Digested by: Roxanne A. Huyo to be practised upon the insurance company, for in his
Eguaras v. Great Eastern Life Ass. Co. acknowledgment and consent his mother-in-law was
designated as the beneficiary of the insurance, despite
Facts: the fact that he had children and his mother was still
living. In the present case the fraud consisted in the
On April 14, 1913, counsel for Francisca Eguaras filed fact that a healthy and robust person was substituted
a written complaint in the said Laguna court, alleging in place of insured invalid when Dr. Vidal made the
as a cause of action that about October 14, 1912, her physical examination of the one who seeking to be
son-in-law Dominador Albay had applied in writing to insured, for the real person who desired to be insured
the defendant insurance company to insure his life for and who ought to have been examined was in bad
the sum of P5,000, naming as the beneficiary in case health on and before the date of executing the
of his death the plaintiff Francisca Eguaras; that after insurance contract of which facts the insured

EH 403 2010-2011 2011-2012


Page 20
INSURANCE LAW CASE DIGESTS
Dominador Albay and the insurance agent Ponciano
Remigio had full knowledge.
Digested by: Patricia Ko
It is therefore proven that the signatures on the Bernardo Argente vs. West Coast Life Insurance
insurance applications reading "Dominador Albay" are [March 19, 1928, G.R. No. L-24899]
false and forged; that the person who presented
himself to Dr. Vidal to be examined was not the real FACTS:
Dominador Albay, but another different person; that at
the time of the application for insurance and the
Bernardo Argente and his wife Vicenta de Ocampo
issuance of the policy which is the subject matter of
signed an application for joint life insurance for P2,000,
this suit the real Dominador Albay was informed of all
later amended to P15,000. All the information
those machinations, wherefore it is plain that the
contained in the applications was furnished to the
insurance contract between the defendant and
agent by Bernardo Argente.
Dominador Albay is null and void because it is false,
fraudulent and illegal.
Pursuant to their applications, Bernardo and Vicenta
were examined separately by Dr. Cesareo Sta. Ana, a
Article 1269 of the Civil Code states:
medical examiner for the West Coast Life Insurance
There is deceit when by words or insidious
Co., The information or answers to the questions
machinations on the part of one of the contracting
contained on the face of the Medical Examiner's Report
parties the other is induced to execute a contract
were furnished to the doctor by the applicants,
which without them he would not have made.
Bernardo and Vicente Argente.
It is essential to the nature of the deceit, to which the
Later, Vicenta died of cerebral apoplexy. Bernardo
foregoing article refers, that said deceit be prior to or
presented a claim for the payment of the sum of
contemporaneous with the consent that is a necessary
P15,000 the amount of the joint life Insurance policy.
requisite for perfecting the contract, but not that it
After investigation it was discovered that the answers
may have occurred or happened thereafter. A contract
given by the insured in their medical examinations with
is therefore deceitful, for the execution whereof the
regard to their health and previous illness and medical
consent of one of the parties has been secured by
attendance were untrue. For that reason, the West
means of fraud, because he was persuaded by words
Coast Life Insurance Co. refused to pay the claim of
or insidious machinations, statements or false
Bernardo Argente.
promises, and a defective consent wrung from him,
It is admitted that it appears in the Medical Examiner's
even though such do not constitute estafa or any other
Report that Bernardo Argente, in response to the
criminal subject to the penal law.
question asked by the medical examiner, "Have you
ever consulted a physician for, or have you ever
With this array of circumstantial evidence derived from
suffered from any ailment or disease of, the brain or
facts duly proven as a result of the present suit, we
nervous system?" answered "No." To the question,
get, if not a moral certainly, at least a full conviction
"Have you consulted a physician for any ailment or
that when Castor Garcia presented himself to be
disease not included in your above answer," answered
examined by the physician Vidal in place of Dominador
"Yes. Nature of Ailment, Disease or Injury. Scabies,
Albay, serious deceit occurred in perfecting the
Number of attacks 1, Date 1911. Duration 1 month,
insurance contract, for had the agent of the company
Severity Fair, results and, if within five years, name
not been deceived it would not have granted the
and address of every physician consulted. Dr. P.
insurance applied for by Albay, nor would it have
Guazon. Cured. Dr. Guazon is dead now." And to the
executed the contract by virtue of whereof payment is
question, "What physician or physicians, if any, not
claimed of the value of policy obtained through fraud;
named above, have you consulted or been treated by,
and consequently on such assumptions it is improper,
within the last five years and for what illness or
nor is it permitted by the law, to order collection of the
ailment? (If none, so state)" answered "No." It is,
amount claimed.
however, not disputed that on January 10, 11, and 13,
In a contract executed with the requisites fixed in
1923, Bernardo Argente was confined in the Philippine
article 1261, one of the contracting parties may have
General Hospital where he was treated by Dr. Agerico
given his consent through error, violence, intimidation,
B. M. Sison for cerebral congestion and Bell's Palsy.
or deceit, and in any of such cases the contract is void,
even though, despite this nullity, no crime was
And that Vicenta de Ocampo, in response to the
committed. (Article 1265, Civil Code.) There may not
question asked by the medical examiner, "How
have been estafa in the case at bar, but it was
frequently, if at all, and in what quantity do you use
conclusively demonstrated by the trial that deceit
beer, wine, spirits or other intoxicants?" answered
entered into the insurance contract, fulfillment whereof
"Beer only in small quantities occasionally." To the
is claimed, and therefore the conclusions reached by
question, "Have you ever consulted a physician for or
the court in the judgment it rendered in the criminal
have you ever suffered from any ailment or disease of
proceedings for estafa do not affect this suit, nor do
the brain or nervous system?" answered "No." To the
they influence the decision proper herein, nor can they
question, "What physician or physicians, if any, not
produce in the present suit, over the exception of the
named above, have you consulted or been treated by,
defendant, the force of res adjudicata.

EH 403 2010-2011 2011-2012


Page 21
INSURANCE LAW CASE DIGESTS
within the last five years and for what illness or knowledge, is misled into a belief that the
ailment? (If none, so state)" answered "None." And to circumstance withheld does not exist, and he is
the question, "Are you in good health as far as you thereby induced to estimate the risk upon a false
know and believe?" answered "Yes." It is, however, not basis that it does not exist. The principal question,
disputed that Vicenta de Ocampo was taken by a therefore, must be, Was the assurer misled or
patrolman, at the request of her husband, Bernardo deceived into entering a contract obligation or in fixing
Argente, on May 19, 1924, to the Meisic police station, the premium of insurance by a withholding of material
and from there was transferred to the San Lazaro information of facts within the assured's knowledge or
Hospital. In San Lazaro Hospital, her case was presumed knowledge?
diagnosed by the admitting physician as "alcoholism,"
but later Doctor Domingo made a diagnosis of probable It therefore follows that the assurer in assuming
"manic-depressive psychosis," and still, later in Mary a risk is entitled to know every material fact of
Chiles Hospital, made a final diagnosis of "phycho- which the assured has exclusive or peculiar
neurosis." knowledge, as well as all material facts which
directly tend to increase the hazard or risk which
ISSUE: are known by the assured, or which ought to be
or are presumed to be known by him. And a
was there false misrepresentation? concealment of such facts vitiates the policy. "It does
not seem to be necessary . . . that the . . . suppression
RULING: of the truth should have been willful."

Yes, there was misrepresentation giving the insurer a If it were but an inadvertent omission, yet if it were
right to rescind the contract. material to the risk and such as the plaintiff should
have known to be so, it would render the policy void.
The court found from the evidence that the But it is held that if untrue or false answers are given
representations made by Bernardo Argente and his in response to inquiries and they relate to material
wife in their applications to the defendant for life facts the policy is avoided without regard to the
insurance were false with respect to their estate of knowledge or fraud of assured, although under the
health during the period of five years preceding the statute statements are representations which must be
date of such applications, and that they knew the fraudulent to avoid the policy. So under certain codes
representations made by them in their applications the important inquiries are whether the concealment
were false. was willful and related to a matter material to the risk.

In the policy was procured by fraudulent


representations, the contract of insurance THE INSULAR LIFE ASSURANCE CO., LTD. v.
apparently set forth therein was never legally SERAFIN D. FELICIANO ET AL.
existent. It can fairly be assumed that had the [G.R. No. L-47593 December 29, 1943]
true facts been disclosed by the assured, the
insurance would never have been granted. Insular Life filed this case (Motion for reconsideration)

And it is declared that if a material fact concealed Facts:


by assured it is equivalent to a false
representation that it does not exist and that the Evaristo Feliciano, who died on September 29, 1935,
essentials are the truth of the representations was suffering with advanced pulmonary tuberculosis
whether they were intended to mislead and did when he signed his applications for insurance with the
insurer accept them as true and act upon them to petitioner on October 12, 1934. On that same date
his prejudice. So it is decided that under a stipulation Doctor Trepp, who had taken X-ray pictures of his
voiding the policy for concealment or lungs, informed the respondent Dr. Serafin D.
misrepresentation of any material fact or if his interest Feliciano, brother of Evaristo, that the latter "was
is not truly stated or is either than the sole and already in a very serious ad practically hopeless
unconditional ownership the facts are unimportant that condition." Nevertheless the question contained in the
insured did not intend to deceive or withhold application "Have you ever suffered from any
information as to encumbrances even though no ailment or disease of the lungs, pleurisy, pneumonia or
questions were asked. asthma?" appears to have been answered , "No".

x x x x x x x x x False answer above referred to, as well as the others,


was written by the Company's soliciting agent David,
The basis of the rule vitiating the contract in case of in collusion with the medical examiner Dr. Gregorio
concealment is that it misleads or deceives the insurer Valdez, for the purpose of securing the Company's
into accepting the risk, or accepting it at the rate of approval of the application so that the policy to be
premium agreed upon. The insurer, relying upon issued thereon might be credited to said agent in
the belief that the assured will disclose every connection with the inter-provincial contest which the
material within his actual or presumed Company was then holding among its soliciting agents
to boost the sales of its policies. Agent David bribed

EH 403 2010-2011 2011-2012


Page 22
INSURANCE LAW CASE DIGESTS
Medical Examiner Valdez with money which the former A 20- year endowment non-medical insurance was
borrowed from the applicant's mother by way of issued to Estefania Saturnino after submission of her
advanced payment on the premium, according to the application to the appelle ( Philippine American Life
finding of the Court of Appeals. Said court also found Insurance) and payment of P339.25 as the first years
that before the insured signed the application he, as premium. The policy dispenses with the medical
well as the members of his family, told the agent and examination of the applicant usually required in other
the medical examiner that he had been sick and ordinary life policies. Nevertheless, it required for a
coughing for some time and that he had gone three detailed information in the application concerning the
times to the Santol Sanatorium and had X-ray pictures applicants health medical history.
of his lungs taken; but that in spite of such information
the agent and the medical examiner told them that the It was discovered that prior to the issuance of the
applicant was a fit subject for insurance. policy, Saturnino was operated on for cancer, involving
complete removal of the right breast, including the
The policy and the application constitute the entire pectoral muscles and the glands found in the right
contract between the parties hereto. Said contract armpit. Estefania Saturnino however did not the
provides xxx and the Company shall not be bound by disclose the fact of operation in the application. She
any promise or representation heretofore or hereafter even stated therein that she did not have, nor had she
given by any person other than the above-named ever had, among other ailments listed in the
officials, and by them only in writing and signed application, cancer or other tumors; that she had not
conjointly as stated consulted any physician, undergone any operation or
suffered any injury within the preceding five years;
Issue: and that she had never been treated for nor did she
ever have any illness or disease peculiar to her sex,
Whether or not Insular Life is bound by the acts of its particularly of the breast, ovaries, uterus, and
agents? Whether or not Feliciano is entitled to the menstrual disorders. The application also recites that
insurance benefits? the foregoing declarations constituted "a further basis
for the issuance of the policy."
Held:
Saturnino died of pneumonia, prompting the husband
NO. Sain unsurance policy is void ab initio. Insular Life and minor child to demand payment of the face value
is not bound by the acts of its agents, as in this case of the policy. The claim was rejected and this suit was
there was connivance between the medical examiner, subsequently instituted.
the agent and Feliciano.From the facts of the case we
cannot escape the conclusion that the insured acted in Issues:
connivance with the soliciting agent and the medical
examiner of the Company in accepting the policies in W/N the insured made such false representations of
question." By accepting the policy Feliciano became material facts to avoid the policy.
charged with knowledge of its contents, whether he
actually read it or not. The insured, therefore, had no Ruling:
right to rely and we cannot believe he relied in good
faith upon the oral representation of said agent and There can be no dispute that the information given by
medical examiner that he (the applicant) was a fit her in her application for insurance was false, namely,
subject for insurance notwithstanding that he had been that she had never had cancer or tumors, or consulted
and was still suffering with advanced pulmonary any physician or undergone any operation within the
tuberculosis. We are to conclude that the insured was preceding period of five years.
a co-participant, and co-responsible with Agent David
and Medical Examiner Valdez, in the fraudulent The Insurance Law (Section 30) provides that
procurement of the policies in question and that by "materiality is to be determined not by the event, but
reason thereof said policies are void ab initio. solely by the probable and reasonable influence of the
facts upon the party to whom the communication is
SC: motion for reconsideration is sustained and the due, in forming his estimate of the proposed contract,
judgment of the Court of Appeals is hereby reversed. or in making his inquiries."
Let another judgment be entered in favor of the
respondents and against the petitioner for the refund It seems to be the contention of appellants that the
of the premiums amounting to P1,389, with legal facts subject of the representation were not material in
interest thereon from the date of the complaint, and view of the "non-medical" nature of the insurance
without any finding as to costs. applied for, which does away with the usual
requirement of medical examination before the policy
is issued.
Digested by: dlanor
Saturnino vs. Philippine American Life Insurance The waiver of medical examination renders even
more material the information required of the
Facts: applicant concerning previous condition of health
and diseases suffered, for such information

EH 403 2010-2011 2011-2012


Page 23
INSURANCE LAW CASE DIGESTS
necessarily constitutes an important factor which constitute a cause of defense. Tan Chay further
the insurer takes into consideration in deciding contends that section 47 of the Insurance Act should
whether to issue the policy or not. be applied, and that when so applied, defendant is
barred and estopped to plead and set forth the matters
Appellants also contend there was no fraudulent alleged in its special defense. That section is as
concealment of the truth inasmuch as the insured follows:
herself did not know, since her doctor never told her, Whenever a right to rescind a contract
that the disease for which she had been operated on of insurance is given to the insurer by any
was cancer. In the first place the concealment of the provision of this chapter, such right must be
fact of the operation itself was fraudulent, as there exercised previous to the commencement of an
could not have been any mistake about it, no matter action on the contract.
what the ailment. Secondly, in order to avoid a
policy it is not necessary to show actual fraud on The Trial court rendered judgment in favor of Tan
the part of the insured. Chay Heng holding that an insurer cannot avoid a
policy which has been procured by fraud unless he
In this jurisdiction a concealment, whether intentional brings an action to rescind it before he is sued thereon.
or unintentional, entitles the insurer to rescind the
contract of insurance, concealment being defined as Issue:
"negligence to communicate that which a party knows
and ought to communicate". WON Section 47 can be applied to the matters alleged
in the special defense of Tan Chay Heng.

v. Materiality of representation (Section 46) Held: NO.


vi. Rules on concealment and representation
(Section 47) Precisely, the defense of West Coast was that the
vii. Right of rescission/Incontestability policy was acquired through fraud in its execution, the
(Section 48) policy is void ab initio, and therefore, no valid contract
was ever made. Its action then cannot be for rescission
because an action to rescind is founded upon and
Digested by: Kristine Oja presupposes the existence of the contract. Hence,
Tan Chay Hong v. West Coast Life Ins Co. West Coasts defense is not barred by Sec. 47.

Facts: In the instant case, it will be noted that even in its


prayer, the defendant does not seek to have the
In 1925, defendant West Coast Life Insurance alleged insurance contract rescinded. It denies that it
Company accepted and approved a life insurance ever made any contract of insurance on the life of Tan
policy for the sum of P10,000 in favor of Tan Caeng in Caeng, or that any such a contract ever existed, and
which the plaintiff (Tan Chay Heng) was the sole that is the question which it seeks to have litigated by
beneficiary. Tan Ceang died on May 10, 1925 and in its special defense. In the very nature of things, if the
June, 1925, Tan Chay Heng submitted the proofs of defendant never made or entered into the contract in
the death of Tan Ceang with a claim for the payment question, there is no contract to rescind, and, hence,
of the policy. West Coast refused to pay and in section 47 upon which the lower court based its
consequence thereof Tan Chay Heng prayed before the decision in sustaining the demurrer does not apply.
court for judgment, with legal interest from the date of
the policy, and costs. As stated, an action to rescind a contract is founded
upon and presupposes the existence of the contract
West Coast filed an answer in which it made a general which is sought to be rescinded. If all of the material
and specific denial on the allegations in the complaint. matters set forth and alleged in the defendant's special
West Coast further filed an amended answer, alleging plea are true, there was no valid contract of insurance,
special defenses that the insurance policy on the life of for the simple reason that the minds of the parties
Tan Ceang, upon which Tan Chay Hengs action is never met and never agreed upon the terms and
based, was obtained in confabulation with one Go conditions of the contract. We are clearly of the
Chulian and Dr. V. S. Locsin, thru fraud and deceit opinion that, if such matters are known to exist by a
.West Coast said that it was made to appear that Tan preponderance of the evidence, they would constitute
Caeng was single, a merchant, of good health a valid defense to plaintiff's cause of action. Upon the
condition and not a drug user, when in fact he was question as to whether or not they are or are not true,
married, a laborer, suffering from tuberculosis and we do not at this time have or express any opinion,
addicted to drugs. West Coast now denies liability but we are clear that section 47 does not apply to the
based on these misrepresentations and prays that it be allegations made in the answer, and that the trial court
absolved from plaintiff's complaint. erred in sustaining the demurrer.

To this special defense, Tan Chay Heng, claiming that The judgment of the lower court is reversed and the
it was a cross-complaint, filed a general demurrer upon case is remanded for such other and further
the ground that it does not state facts sufficient to

EH 403 2010-2011 2011-2012


Page 24
INSURANCE LAW CASE DIGESTS
proceedings as are not inconsistent with this opinion,
with costs against the plaintiff. The deceased, by affixing his signature on the
application form, affirmed the correctness of all the
entries and answers appearing therein. It is but to be
-Argente expected that he, a businessman, would not have
affixed his signature on the application form unless he
clearly understood its significance. For, the
presumption is that a person intends the ordinary
Author: Aldrin Montesco consequence of his voluntary act and takes ordinary
TAN VS. CA care of his concerns. [Sec. 5(c) and (d), Rule 131,
[174 SCRA 403] Rules of Court].

FACTS: The evidence for respondent company shows that on


September 19,1972, the deceased was examined by
Businessman Tan Lee Siong applied for life insurance Dr. Victoriano Lim and was found to be diabetic and
policy with American Life Insurance Company in the hypertensive; that by January, 1973, the deceased
amount of P80,000.00 by virtue of which he was was complaining of progressive weight loss and
issued Policy No. 1082467 effective November 6,1973, abdominal pain and was diagnosed to be suffering
with his sons as designated beneficiaries thereof. On from hepatoma, (t.s.n. August 23, 1976, pp. 8-10;
April 1975, Tan Lee Siong died and the sons Exhibit 2). Another physician, Dr. Wenceslao Vitug,
subsequently filed their claim for the insurance testified that the deceased came to see him on
proceeds. But in September of the same year, the December 14, 1973 for consolation and claimed to
company sent a letter denying petitioners' claim and have been diabetic for five years. (t.s.n., Aug.
rescinded the policy by reason of the alleged 23,1976, p. 5; Exhibit 6) Because of the concealment
misrepresentation and concealment of material facts made by the deceased of his consultations and
made by the deceased Tan Lee Siong in his application treatments for hypertension, diabetes and liver
for insurance. The premiums paid on the policy, disorders, respondent company was thus misled into
however, were thereupon refunded. accepting the risk and approving his application as
medically standard (Exhibit 5- C) and dispensing with
further medical investigation and examination (Exhibit
ISSUES: 5-A). For as long as no adverse medical history is
revealed in the application form, an applicant for
Whether or not the insurance company no longer had insurance is presumed to be healthy and physically fit
the right to rescind the contract of insurance as and no further medical investigation or examination is
rescission must allegedly be done during the lifetime of conducted by respondent company
the insured within two years and prior to the
commencement of the action.
F. Insurance Policy
RULING: i. Definition (Section 49)

The so-called "incontestability clause" precludes the Digested by: Archie Necesario
insurer from raising the defenses of false Vicente Tang vs Court of Appeals, Philippine
representations or concealment of material facts American Life Insurance Company
insofar as health and previous diseases are concerned
if the insurance has been in force for at least two years Facts:
during the insured's lifetime. The phrase "during the
lifetime" found in Section 48 simply means that the Lee See Guat, a widow, 61 years old and an illeterate
policy is no longer considered in force after the insured who spoke only Chinese, applied on two separate
has died. The key phrase in the second paragraph of times for an insurance on her life from Philippine
Section 48 is "for a period of two years." American Life Insurance Co. (Company), amounting to
a total of P100,000. The application consisted of two
As noted by the Court of Appeals, to wit: parts, both in the English language. The second part of
The policy was issued on November 6, 1973 and the her application dealt with her state of health and
insured died on April 26,1975. The policy was thus in because her answers indicated that she was healthy,
force for a period of only one year and five months. the Company approved her two applications, with her
Considering that the insured died before the two-year nephew Vicente Tang as her benefeciary.
period had lapsed, respondent company is not,
therefore, barred from proving that the policy is Less than a year from her application, Lee See Guat
void ab initio by reason of the insured's fraudulent died of lung cancer. Tang tried to claim the value of
concealment or misrepresentation. Moreover, the insurance policy but the Company refused to pay
respondent company rescinded the contract of on the ground that the insured was guilty of
insurance and refunded the premiums paid on concealment and misrepresentation at the time she
September 11, 1975, previous to the commencement applied for the two policies. Both the trial court and
of this action on November 27,1975.

EH 403 2010-2011 2011-2012


Page 25
INSURANCE LAW CASE DIGESTS
the Court of Appeals ruled in favor of the insurance Provisional Policy:
company. The above-mentioned life is to be assured in
accordance with the terms and conditions contained or
Issue: inserted by the Company in the policy which may be
granted by it in this particular case for four months
Whether or not Article 1332 of the Civil Code is only from the date of the application, provided that
applicable to this case. the Company shall confirm this agreement by issuing a
policy on said application when the same shall be
Ruling: submitted to the Head Office in Montreal. Should the
Company not issue such a policy, then this agreement
"Art. 1332. When one of the parties is unable to read, shall be null and void ab initio, and the Company shall
or if the contract is in a language not understood by be held not to have been on the risk at all, but in such
him, and mistake or fraud is alleged, the person case the amount herein acknowledged shall be
enforcing the contract must show that the terms returned.
thereof have been fully explained to the former."
Luis Lim y Garcia died on August 23, 1917, after the
It is the position of the petitioner that because Lee See issuance of the provisional policy but before approval
Guat was illiterate and spoke only Chinese, she could of the application by the home office of the insurance
not be held guilty of concealment of her health history company. The instant action is brought by the
because the applications for insurance were in English beneficiary, Pilar C. de Lim, to recover from the Sun
and the insurer has not proved that the terms thereof Life Assurance Company of Canada the sum of P5,000,
had been fully explained to her. the amount named in the provisional policy.

It should be noted that under Art. 1332, the obligation Court of First Instance of Zamboanga sustained the
to show that the terms of the contract had been fully demurrer to plaintiff's complaint on the ground that it
explained to the party who is unable to read or fails to state a cause of action.
understand the language of the contract, when fraud
or mistake is alleged, devolves on the party seeking to Issues:
enforce it. Here the insurance company is not seeking
to enforce the contracts; on the contrary, it is seeking (1) WON the contract was consummated?
to avoid their performance. It is petitioner who is (2) WON this is a form of cover notes? (note: not
seeking to enforce them even as fraud or mistake is 100% sure about this )
not alleged. Accordingly, respondent company was
under no obligation to prove that the terms of the Ruling:
insurance contracts were fully explained to the other
party. Even if we were to say that the insurer is the (1) NO. The contract, to be binding from the date of
one seeking the performance of the contracts by the application, must have been a completed contract,
avoiding paying the claim, it has to be noted as above one that leaves nothing to be done, nothing to be
stated that there has been no imputation of mistake or completed, nothing to be passed upon, or determined,
fraud by the illiterate insured whose personality is before it shall take effect. There can be no contract of
represented by her beneficiary the petitioner herein. In insurance unless the minds of the parties have met in
sum, Art. 1332 is inapplicable to the case at bar. agreement. As we read and understand the so-called
provisional policy it amounts to nothing but an
Decision of the Court of Appeals is affirmed. acknowledgment on behalf of the company, that it has
received from the person named therein the sum of
money agreed upon as the first year's premium upon a
ii. Form of insurance Policy; riders, policy to be issued upon the application, if the
etc)/Contents (Sections 50-51) application is accepted by the company.
iii. Cover notes (Section 52)
SC finds the 2nd rule (Joyce on Insurance, relied by
Digested by: Sharmine M. Odchigue appellant) to be applicable . The Rule states;
De Lim vs. Sun Life Ass. Co. of Canada Where an agreement is made between the applicant
[41 Phil 263] and the agent whether by signing an application
containing such condition, or otherwise, that no liability
Facts: shall attach until the principal approves the risk and a
receipt is given buy the agent, such acceptance is
On July 6, 1917, Luis Lim y Garcia of Zamboanga merely conditional, and it subordinated to the act of
applied for a policy of insurance on his life (Sun Life the company in approving or rejecting; so in life
Assurance Company of Canada) in the sum of P5,000. insurance a "binding slip" or "binding receipt" does not
His wife, Pilar C. de Lim, plaintiff, was the beneficiary. insure of itself.
The first premium of P433 was paid by Lim, and upon
such payment the company issued what was called a (2) NO. Sec 52. Cover notes may be issued to
"provisional policy." bind insurance temporarily pending the issuance

EH 403 2010-2011 2011-2012


Page 26
INSURANCE LAW CASE DIGESTS
of the policy. In this case there is no temporary
insurance. We uphold petitioner's submission that the Cover Note
Cases which the COURT find applicable to this case: was not without consideration. The fact that no
separate premium was paid on the Cover Note before
In the case of Steinle vs. New York Life Insurance Co. the loss insured against occurred, does not militate
([1897], 81 Fed., 489} the facts were that the amount against the validity of petitioner's contention, for no
of the first premium had been paid to an insurance such premium could have been paid, since by the
agent and a receipt given therefor. The receipt, nature of the Cover Note, it did not contain, as all
however, expressly declared that if the application was Cover Notes do not contain particulars of the shipment
accepted by the company, the insurance shall take that would serve as basis for the computation of the
effect from the date of the application but that if the premiums. As a logical consequence, no separate
application was not accepted, the money shall be premiums are intended or required to be paid on a
returned. The trite decision of the circuit court of Cover Note.
appeal was, "On the conceded facts of this case, there
was no contract to life insurance perfected and the At any rate, it is not disputed that petitioner paid in full
judgment of the circuit court must be affirmed." all the premiums as called for by the statement issued
by private respondent after the issuance of the two
In the case of Cooksey vs. Mutual Life Insurance Co. regular marine insurance policies, thereby leaving no
([1904], 73 Ark., 117) the person applying for the life account unpaid by petitioner due on the insurance
insurance paid and amount equal to the first premium, coverage, which must be deemed to include the Cover
but the application and the receipt for the money paid, Note. If the Note is to be treated as a separate policy
stipulated that the insurance was to become effective instead of integrating it to the regular policies
only when the application was approved and the policy subsequently issued, the purpose and function of the
issued. The court held that the transaction did not Cover Note would be set at naught or rendered
amount to an agreement for preliminary or temporary meaningless, for it is in a real sense a contract, not a
insurance. mere application for insurance which is a mere offer.

SC did not find any error on the part of trial court in


sustaining the demurrer and dismissing the case. iv. Application of insurance proceeds
Lastly, counsel for appellee admits the liability of the (Section 53)
company for the return of the first premium to the
estate of the deceased. Sun Life Assurance Company Digested by: Gayle Opsima
of Canada will immediately pay to the estate of the FRANCISCO DEL VAL ET AL.vs. ANDRES DEL VAL
late Luis Lim y Garcia the of P433. SO ORDERED.
Facts:

Digested by: Kristine Oja Plaintiff and defendant are siblings who are heirs at
Pacific Timber Export Corp. v. CA law of Gregorio Nacianceno del Val, who died intestate.
The deceased, during his lifetime, took out an
Facts: insurance on his life for the sum of P40,000 and made
it payable to the defendant as sole beneficiary.
On March 13, 1963, Pacific Timber secured temporary
insurance from the Workmens Insurance Co. for its Plaintiffs contend that the amount of the insurance
exportation of logs to Japan. Workmen Insurance policy belonged to the estate of the deceased and not
issued on said date Cover Note 1010 insuring said to the defendant personally; that, therefore, they are
cargo. The regular marine policies were issued by entitled to a partition not only of the real and personal
the company in favor of Pacific Timber on April 2, property, but also of the P40,000 life insurance.
1963. The two marine policies bore the number Issue:
53H01032 and 53H01033. After the issuance of the
cover note but before the issuance of the two policies, Whether or not the insurance policy belongs to the
some of the logs intended to be exported were lost estate
due to a typhoon. Pacific Timber filed its claim with
the company, but the latter refused, contending that Ruling:
the cover note insuring the cargo is null and void for
lack of valuable consideration. With the finding of the trial court that the proceeds of
the life-insurance policy belong exclusively to the
Issue: defendant as his individual and separate property, we
agree. That the proceeds of an insurance policy belong
WON the cover note was without consideration, thus exclusively to the beneficiary and not to the estate of
null and void. the person whose life was insured, and that such
proceeds are the separate and individual property of
Held: the beneficiary, and not of the heirs of the person
whose life was insured, is the doctrine in America. We
It was with consideration. believe that the same doctrine obtains in these Islands

EH 403 2010-2011 2011-2012


Page 27
INSURANCE LAW CASE DIGESTS
by virtue of section 428 of the Code of Commerce, then assigned the accident to the Bayne
which reads: Adjustment Co. for appraisal of the damage.
Enrique Mora, without the knowledge and consent
"The amounts which the underwriter must deliver to of the H.S. Reyes, Inc., authorized the Bonifacio
the person insured, in fulfillment of the contract, shall Bros. Inc. to furnish the labor and materials, some
be the property of the latter, even against the claims of which were supplied by the Ayala Auto Parts Co.
of the legitimate heirs or creditors of any kind The insurance company then drew a check as
whatsoever of the person who effected the insurance proceeds of the insurance policy, payable to the
in favor of the former." order of Enrique Mora or H.S. Reyes, Inc., and
entrusted the check to the H.H. Bayne Adjustment
It is claimed by the attorney for the plaintiffs that the Co. for disposition and delivery to the proper party.
proceeds of the insurance policy were a donation or In the meantime, the car was delivered to Enrique
gift made by the father during his lifetime to the Mora without the consent of the H.S. Reyes, Inc.,
defendant and that, as such, its ultimate destination is and without payment to the Bonifacio Bros.
determined by those provisions of the Civil Code which Inc. and the Ayala Auto Parts Co. of the cost
relate to donations, especially article 819. This article of repairs and materials.
provides that "gifts made to children which are not Upon the theory that the insurance proceeds
betterments shall be considered as part of their legal should be paid directly to them, the Bonifacio Bros.
portion." Inc. and the Ayala Auto Parts Co. filed this
complaint. Their arguments are based on
We cannot agree with these contentions. The contract paragraph 4 of the insurance contract which
of life insurance is a special contract and the provides that "the insured may authorize the repair
destination of the proceeds thereof is determined by of the Motor Vehicle necessitated by damage for
special laws which deal exclusively with that subject. which the company may be liable under the policy
The Civil Code has no provisions which relate directly xxx." It is stressed that the H.H. Bayne Adjustment
and specifically to life-insurance contracts or to the Company's recommendation of payment of the
destination of life insurance proceeds. That subject is appellants' bill for which the Insurance Company
regulated exclusively by the Code of Commerce which drew a check indicates that Mora and the H.H.
provides for the terms of the contract, the relations of Bayne Adjustment Co. acted for and in
the parties and the destination of the proceeds of the representation of that insurance company.
policy.
Issue:

Digested by: Kenn Whether or not there is privity of contract between the
Bonifacio Bros. vs Mora Bonifacio Bros. Inc. and the Ayala Auto Parts Co. on
the one hand and the insurance company on the other.
Facts: Who has better right over the insurance proceeds?

Enrique Mora mortgaged his car to the H.S. Reyes Ruling:


with the condition that the former would insure it
with the latter as beneficiary. The appellants are not mentioned in the contract as
He insured it with the State Bonding & Insurance parties thereto nor is there any clause or provision
Co., Inc., with pertinent provisions of the policy thereof from which to infer that there is an obligation
which read: on the part of the insurance company to pay the cost
xxx xxx xxx of repairs directly to them. It is fundamental that
4. The Insured may authorize the repair of contracts take effect only between the parties thereto,
the Motor Vehicle necessitated by damage except in some specific instances provided by law
for which the Company may be liable under where the contract contains some stipulation in favor
this Policy provided that: (a) The estimated of a third person. Such stipulation is known as
cost of such repair does not exceed the stipulation pour autrui or a provision in favor of a third
Authorized Repair Limit, (b) A detailed person not a pay to the contract. Under this doctrine, a
estimate of the cost is forwarded to the third person is allowed to avail himself of a benefit
Company without delay, subject to the granted to him by the terms of the contract, provided
condition that "Loss, if any is payable to that the contracting parties have clearly and
H.S. Reyes, Inc.," by virtue of the fact that deliberately conferred a favor upon such
said Oldsmobile sedan was mortgaged in favor person. Consequently, a third person not a party to
of the said H.S. Reyes, Inc. and that under a the contract has no action against the parties thereto,
clause in said insurance policy, any loss was and cannot generally demand the enforcement of the
made payable to the H.S. Reyes, Inc. as same. The question of whether a third person has an
Mortgagee; enforcible interest in a contract, must be settled by
xxx xxx xxx determining whether the contracting parties intended
During the effectivity of the insurance contract, the to tender him such an interest by deliberately inserting
car met with an accident. The insurance company terms in their agreement with the avowed purpose of
conferring a favor upon such third person. In this

EH 403 2010-2011 2011-2012


Page 28
INSURANCE LAW CASE DIGESTS
connection, SC has laid down the rule that the fairest Iluminado del Monte, one of the drivers of the
test to determine whether the interest of a third jeepneys operated by Aguilar, bumped with the
person in a contract is a stipulation pour autrui or jeepney abovementioned one Gervacio Guingon who
merely an incidental interest, is to rely upon the had just alighted from another jeepney and as a
intention of the parties as disclosed by their consequence the latter died some days thereafter.
contract. In the instant case the insurance contract
does not contain any words or clauses to disclose an The heirs of Gervacio Guingon filed an action for
intent to give any benefit to any repairmen or damages praying that the sum of P82,771.80 be paid
materialmen in case of repair of the car in question. to them jointly and severally by the defendants, driver
The parties to the insurance contract omitted such Iluminado del Monte, owner and operator Julio Aguilar,
stipulation, which is a circumstance that supports the and the Capital Insurance & Surety Co., Inc. For failure
said conclusion. On the other hand, the "loss payable" to answer the complaint, Del Monte and Aguilar were
clause of the insurance policy stipulates that "Loss, if declared in default. Capital Insurance & Surety Co.,
any, is payable to H.S. Reyes, Inc." indicating that it Inc. answered, alleging that the plaintiff has no cause
was only the H.S. Reyes, Inc. which they intended to of action against it based on the following provision:
benefit. E. Action Against Company
No action shall lie against the Company unless, as
Another cogent reason for not recognizing a right of a condition precedent thereto, the Insured shall
action by the appellants against the insurance have fully complied with all of the terms of this
company is that "a policy of insurance is a distinct and Policy, nor until the amount of the Insured's
independent contract between the insured and insurer, obligation to pay shall have been finally
and third persons have no right either in a court of determined either by judgment against the Insured
equity, or in a court of law, to the proceeds of it, after actual trial or by written agreement of the
unless there be some contract of trust, expressed or Insured, the claimant, and the Company.
implied between the insured and third person." 5 In this Any person or organization or the legal
case, no contract of trust, expressed or implied exists. representative thereof who has secured such
This conclusion is deducible not only from the principle judgment or written agreement shall thereafter be
governing the operation and effect of insurance entitled to recover under this policy to the extent
contracts in general, but is clearly covered by the of the insurance afforded by the Policy. Nothing
express provisions of section 50 of the Insurance Act contained in this policy shall give any person or
which read: organization any right to join the Company as a
The insurance shall be applied exclusively to co-defendant in any action against the Insured to
the proper interests of the person in whose determine the Insured's liability.
name it is made unless otherwise specified in Bankruptcy or insolvency of the Insured or of the
the policy. Insured's estate shall not relieve the Company of
any of its obligations hereunder.
The policy in question has been so framed that "Loss,
if any, is payable to H.S. Reyes, Inc.," which Capital Insurance & Surety Co. contends that the "no
unmistakably shows the intention of the parties. action" clause in the policy closes the avenue to any
third party which may be injured in an accident
wherein the jeepney of the insured might have been
Digested by: ra osorio the cause of the injury of third persons, alleging the
Guingon vs. Del Monte freedom of contracts.

FACTS: ISSUE:

Julio Aguilar owned and operated several jeepneys. He WON the heirs of Guingon have a cause of action
entered into a contract with the Capital Insurance & against Capital Insurance
Surety Co., Inc. insuring the operation of his jeepneys
against accidents with third-party liability. The RULING: Yes.
insurance policy contains the following provisions:
Section II LIABILITY TO THE PUBLIC The policy in the present case is one whereby the
1. The Company, will, subject to the limits of insurer agreed to indemnify the insured "against all
liability, indemnify the Insured in the event of sums . . . which the Insured shall become legally liable
accident caused by or arising out of the use of the to pay in respect of: a. death of or bodily injury to any
Motor Vehicle/s or in connection with the loading person . . . ." Clearly, therefore, it is one for indemnity
or unloading of the Motor Vehicle/s, against all against liability; from the fact then that the insured is
sums including claimant's costs and expenses liable to the third person, such third person is entitled
which the Insured shall become legally liable to to sue the insurer
pay in respect of: The right of the person injured to sue the insurer of
a. death of or bodily injury to any person the party at fault (insured), depends on whether the
b. damage to property contract of insurance is intended to benefit third
persons also or only the insured. And the test applied
has been this: Where the contract provides for

EH 403 2010-2011 2011-2012


Page 29
INSURANCE LAW CASE DIGESTS
indemnity against liability to third persons, then third The trial court rendered a decision sentencing the
persons to whom the insured is liable, can sue the Company to pay to the plaintiffs the sum of P4,000.00
insurer. and the costs.

The next question is on the right of the third person to Issue:


sue the insurer jointly with the insured. The policy
requires, as afore-stated, that suit and final judgment Fieldmen appealed, it contended that plaintiffs have no
be first obtained against the insured; that only cause of action because: 1) the Coquias have no
"thereafter" can the person injured recover on the contractual relation with the Company; and 2) the
policy; it expressly disallows suing the insurer as a co- Insured has not complied with the provisions of the
defendant of the insured in a suit to determine the policy concerning arbitration. (FIRST ISSUE IS the
latter's liability. As adverted to before, the query is MAIN TOPIC)
which procedure to follow that of the insurance
policy or the Rules of Court. Ruling:

In the discussion of the points thus raised, what is As regards the first defense, it should be noted that,
paramount is the interpretation of the insurance although, in general, only parties to a contract may
contract with the aim in view of attaining the bring an action based thereon, this rule is subject to
objectives for which the insurance was taken. The exceptions, one of which is found in the second
Rules of Court provide that parties may be joined paragraph of Article 1311 of the Civil Code of the
either as plaintiffs or defendants, as the right to relief Philippines, reading:
in respect to or arising out of the same transactions is If a contract should contain some stipulation in favor
alleged to exist (Sec. 6, Rule 3). of a third person, he may demand its fulfillment
The "no action" clause in the policy of insurance cannot provided he communicated his acceptance to the
prevail over the Rules of Court provision aimed at obligor before its revocation. A mere incidental benefit
avoiding multiplicity of suits. or interest of a person is not sufficient. The contracting
parties must have clearly and deliberately conferred a
Similarly, in the instant suit, Sec. 5 of Rule 2 on favor upon a third person.
"Joinder of causes of action" and Sec. 6 of Rule 3 on
"Permissive joinder of parties" cannot be superseded, In the stipulations, the Company "will indemnify any
at least with respect to third persons not a party to the authorized Driver who is driving the Motor Vehicle" of
contract, as herein, by a "no action" clause in the the Insured and, in the event of death of said driver,
contract of insurance. the Company shall, likewise, "indemnify his personal
representatives." In fact, the Company "may, at its
option, make indemnity payable directly to the
Digested by: Mark Recto claimants or heirs of claimants ... it being the true
MELECIO COQUIA, MARIA ESPANUEVA and intention of this Policy to protect ... the liabilities of the
MANILA YELLOW TAXICAB CO., Insured towards the passengers of the Motor Vehicle
INC.vs.FIELDMEN'S INSURANCE CO., INC. and the Public" in other words, third parties.
[G.R. No. L-23276, November 29, 1968]
Thus, the policy under consideration is typical of
Facts: contracts pour autrui, this character being made more
manifest by the fact that the deceased driver paid fifty
Fieldmens Insurance Company issued Manila Yellow percent (50%) of the corresponding premiums, which
Taxicab Co. a common carrier accident insurance were deducted from his weekly commissions. Under
policy, covering the period from Dec. 1, 1961 to Dec. these conditions, it is clear that the Coquias who,
1, 1962. admittedly, are the sole heirs of the deceased have
a direct cause of action against the Company, and,
While the policy was in force, or on February 10, 1962, since they could have maintained this action by
a taxicab of the Insured, driven by Carlito Coquia, met themselves, without the assistance of the Insured, it
a vehicular accident at Mangaldan, Pangasinan, in goes without saying that they could and did properly
consequence of which Carlito died. The Insured filed join the latter in filing the complaint herein.
therefor a claim for P5,000.00 to which the Company
replied with an offer to pay P2,000.00, by way of The second defense set up by the Company is based
compromise. The Insured rejected the same and made upon Section 17 of the policy reading:
a counter-offer for P4,000.00, but the Company did If any difference or dispute shall arise with respect
not accept it. Hence, on September 18, 1962, the to the amount of the Company's liability under this
Insured and Carlito's parents, namely, Melecio Coquia Policy, the same shall be referred to the decision of
and Maria Espanueva the Coquias filed a a single arbitrator to be agreed upon by both
complaint against the Company to collect the proceeds parties or failing such agreement of a single
of the aforementioned policy. In its answer, the arbitrator, to the decision of two arbitrators, one to
Company admitted the existence thereof, but pleaded be appointed in writing by each of the parties
lack of cause of action on the part of the plaintiffs. within one calendar month after having been
required in writing so to do by either of the parties

EH 403 2010-2011 2011-2012


Page 30
INSURANCE LAW CASE DIGESTS
and in case of disagreement between the Development Insurance Corporation v.s. IAC and
arbitrators, to the decision of an umpire who shall PHILIPPINE UNION REALTY DEVELOPMENT
have been appointed in writing by the arbitrators CORPORATION
before entering on the reference and the costs of [G.R. No. 71360 July 16, 1986]
and incident to the reference shall be dealt with in
the Award. And it is hereby expressly stipulated Facts:
and declared that it shall be a condition precedent
to any right of action or suit upon this Policy that A fire broke out in the building of the private
the award by such arbitrator, arbitrators or umpire respondent and it sued for recovery of damages from
of the amount of the Company's liability hereunder the petitioner on the basis of an insurance contract
if disputed shall be first obtained. between them. The private respondent's building was
insured by petitioner against fire for P2,500,000.00
The record shows, however, that none of the parties to under an open policy, Policy No. RY/F-082.
the contract invoked this section, or made any
reference to arbitration, during the negotiations Petitioner refused to pay advancing the argument that
preceding the institution of the present case. In fact, only the building and not the elevators were insured.
counsel for both parties stipulated, in the trial court,
that none of them had, at any time during said Note: Actually, the petitioner in this case already lost
negotiations, even suggested the settlement of the through default. The SC considered its delay
issue between them by arbitration, as provided in said inexcusable, however, it proceeded to discuss the
section. Their aforementioned acts or omissions had merits of the case.
the effect of a waiver of their respective right to
demand an arbitration. Issue:

In the decision of the Supreme Court of Minnesota in WON the respondent is entitled to indemnity.
Independent School Dist. No. 35, St. Louis County vs.
A. Hedenberg & Co., Inc.7 from which we quote: Ruling:
The test for determining whether there has been a
waiver in a particular case is stated by the author The petitioner's claim that the insurance covered only
of an exhaustive annotation in 117 A.L.R. p. 304, the building and not the elevators is absurd, to say the
as follows: "Any conduct of the parties inconsistent least. This Court has little patience with puerile
with the notion that they treated the arbitration arguments that affront common sense, let alone basic
provision as in effect, or any conduct which might legal principles with which even law students are
be reasonably construed as showing that they did familiar. The circumstance that the building insured is
not intend to avail themselves of such provision, seven stories high and so had to be provided with
may amount to a waiver thereof and estop the elevators-a legal requirement known to the petitioner
party charged with such conduct from claiming its as an insurance company-makes its contention all the
benefits". more ridiculous.
xxx xxx xxx
The decisive facts here are that both parties from No less preposterous is the petitioner's claim that the
the inception of their dispute proceeded in entire elevators were insured after the occurrence of the fire,
disregard of the provisions of the contract relating a case of shutting the barn door after the horse had
to arbitration and that neither at any stage of such escaped, so to speak. 4 This pretense merits scant
dispute, either before or after commencement of attention. Equally undeserving of serious consideration
the action, demanded arbitration, either by oral or is its submission that the elevators were not damaged
written demand, pleading, or otherwise. Their by the fire, against the report of The arson
conduct was as effective a rejection of the right to investigators of the INP 5 and, indeed, its own
arbitrate as if, in the best Coolidge tradition, they expressed admission in its answer 6 where it affirmed
had said, "We do not choose to arbitrate". As that the fire "damaged or destroyed a portion of the
arbitration under the express provisions of article 7th floor of the insured building and more particularly
40 was "at the choice of either party," and was a Hitachi elevator control panel."
chosen by neither, a waiver by both of the right to
arbitration followed as a matter of law. There is no reason to disturb the factual findings of the
lower court, as affirmed by the Intermediate Appellate
WHEREFORE, the decision appealed from should be as Court, that the heat and moisture caused by the fire
it is hereby affirmed in toto, with costs against the damaged although they did not actually burn the
herein defendant-appellant, Fieldmen's Insurance Co., elevators. Neither is this Court justified in reversing
Inc. It is so ordered. their determination, also factual, of the value of the
loss sustained by the private respondent in the amount
of P508,867.00.
v. Sections 54-59
vi. Open Policy (Section 60) The only remaining question to be settled is the
amount of the indemnity due to the private respondent
Digested by: George under its insurance contract with the petitioner. This

EH 403 2010-2011 2011-2012


Page 31
INSURANCE LAW CASE DIGESTS
will require an examination of this contract, Policy No. applying the open policy clause as expressly agreed
RY/F-082, as renewed, by virtue of which the upon by the parties in their contract, we hold that the
petitioner insured the private respondent's building private respondent is entitled to the payment of
against fire for P2,500,000.00. indemnity under the said contract in the total amount
of P508,867.00.
The petitioner argues that since at the time of the fire
the building insured was worth P5,800,000.00, the
private respondent should be considered its own vii. Valued Policy (Section 61)
insurer for the difference between that amount and the
face value of the policy and should share pro rata in Digested by: Mark Rossel Salomon
the loss sustained. Accordingly, the private respondent Harding vs. Commercial Union Ass. Co.
is entitled to an indemnity of only P67,629.31, the rest
of the loss to be shouldered by it alone. In support of Facts:
this contention, the petitioner cites Condition 17 of the
policy, which provides: Henry Harding bought an automobile which was
If the property hereby insured shall, at the subsequently given to her wife as a gift. The said
breaking out of any fire, be collectively of greater automobile was repaired and repainted at the Luneta
value than the sum insured thereon then the Garage. Luneta Garage, acting as agent for Smith, Bell
insured shall be considered as being his own & Company (limited), solicited of the plaintiff Mrs.
insurer for the difference, and shall bear a ratable Harding the insurance of said automobile by the
proportion of the loss accordingly. Every item, if Commercial Union Assurance Company. A proposal
more than one, of the policy shall be separately was made, followed by an inspection as to thevakue of
subject to this condition. the automobile and thereafter Commercial Union
Assurance Company issued a policy of insurance upon
However, there is no evidence on record that the said proposal., in which policy the said automobile was
building was worth P5,800,000.00 at the time of the described as of the 'present value' of P3,000.
loss; only the petitioner says so and it does not back
up its self-serving estimate with any independent Subsequently, the said automobile was totally
corroboration. On the contrary, the building was destroyed by fire.
insured at P2,500,000.00, and this must be
considered, by agreement of the insurer and the Issue:
insured, the actual value of the property insured on
the day the fire occurred. This valuation becomes even W/N there was a proper valuation in the insurance
more believable if it is remembered that at the time policy.
the building was burned it was still under construction
and not yet completed. Ruling: There was a proper valuation.

The Court notes that Policy RY/F-082 is an open policy The ordinary test of the value of property is the price it
and is subject to the express condition that: will commend in the market if offered for sale. But that
Open Policy test cannot, in the very nature of the case, be applied
This is an open policy as defined in Section 57 of at the time application is made for insurance. Men may
the Insurance Act. In the event of loss, whether honestly differ about the value of property, or as to
total or partial, it is understood that the amount of what it will bring in the market; and such differences
the loss shall be subject to appraisal and the are often very marked among those whose special
liability of the company, if established, shall be business it is to buy and sell property of all kinds. The
limited to the actual loss, subject to the applicable assured could do no more than estimate such value;
terms, conditions, warranties and clauses of this and that, it seems, was all that he was required to do
Policy, and in no case shall exceed the amount of in this case. His duty was to deal fairly with the
the policy. Company in making such estimate. The special finding
shows that he discharged that duty and observed good
As defined in the aforestated provision, which is now faith. We shall not presume that the Company, after
Section 60 of the Insurance Code, "an open requiring the assured in his application to give the
policy is one in which the value of the thing 'estimated value,' and then to covenant that he had
insured is not agreed upon but is left to be stated all material facts in regard to such value, so far
ascertained in case of loss. " This means that the as known to him, and after carrying that covenant, by
actual loss, as determined, will represent the express words, into the written contract, intended to
total indemnity due the insured from the insurer abandon the theory upon which it sought the contract,
except only that the total indemnity shall not and make the absolute correctness of such estimated
exceed the face value of the policy. value a condition precedent to any insurance
whatever. The application, with its covenant and
The actual loss has been ascertained in this case and, stipulations, having been made a part of the policy,
to repeat, this Court will respect such factual that presumption cannot be indulged without imputing
determination in the absence of proof that it was to the Company a purpose, by studied intricacy or an
arrived at arbitrarily. There is no such showing. Hence, ingenious framing of the policy, to entrap the assured

EH 403 2010-2011 2011-2012


Page 32
INSURANCE LAW CASE DIGESTS
into incurring obligations which, perhaps, he had no after the delivery of the goods or the date when
thought of assuming." the goods should have been delivered.
Section 163 of the Insurance Law (Act No. 2427)
provides that "the effect of a valuation in a policy of The stipulation is but a repetition of a provision
fire insurance is the same as in a policy of marine contained in section 3 (6) of the United States Carriage
insurance." of Goods by Sea, Act of 1936, which was adopted and
made applicable to the Philippines by Commonwealth
By the terms of section 149 of the Act cited, the Act 65 and by express agreement incorporated by
valuation in a policy of marine insurance is conclusive reference in the bill of lading.
if the insured had an insurable interest and was not
guilty of fraud. The case for the insurer stands on a different footing,
for its claim of prescription is founded upon the terms
of the policy and not upon the bill of lading. Under our
viii. Running Policy (Section 62) law the time limit for bringing a civil action upon a
ix. Limitation to commence action (Section written contract is ten years after the right of action
63) accrues. (Sec. 43, Act 190; Art. 1144, New Civil Code.)
But counsel for the insurer claim that this statutory in
Digested by: Jamee S. Singco the policy:
Eagle Star Ins. vs. Chia Yu No suit action on this Policy, for the recovery of
any claim, shall be sustainable in any Court of law
Facts: or equity unless the insured shall have fully
complied with all the terms and conditions of this
Atkin, Kroll & Co., loaded on the S. S. Roeph Silverlight Policy nor unless commenced with twelve (12)
owned and operated by Leigh Hoegh & Co., A/S, of months next after the happening of the loss . . .
San Francisco California, 14 bales of assorted
underwear valued at P8,085.23 consigned to Chia Yu To this the court cannot agree.
in the City of Manila. The shipment was insured against
all risks by Eagle Star Ins. Co. of San Francisco, Examining the policy sued upon in the present case,
California, under a policy issued to the shipper and by we find that its prescriptive clause, if given effect in
the latter assigned to the consignee. When the vessel accordance with the terms of the policy, would reduce
started discharging its cargo into the custody of the the period allowed the insured for bringing his action
Manila Terminal Co., Inc., which was then operating to less than one year. This is so because the said
the arrastre service for the Bureau of Customs the 14 clause makes the prescriptive period begin from the
bales consigned to Chia Yu, only 10 were delivered to happening of the loss and at the same time provides
him as the remaining 3 could not be found. Three of that the no suit on the policy shall be sustainable in
those delivered were also found damaged to the extent any court unless the insured shall have first fully
of 50 per cent. Chia Yu claimed indemnity for the complied with all the terms and conditions of the
missing and damaged bales. But the claim was policy, among them that which requires that, as so as
declined, first, by the carrier and afterward by the the loss is determined, written claim therefor be filed
insurer, whereupon Chia Yu brought the present action with the carrier and that the letter to the carrier and
against both, including their respective agents in the the latter's reply should be attached to the claim
Philippines. More than two years after delivery of the papers to be sent to the insurer. It is obvious that
damaged bales and the date when the missing bales compliance with this condition precedent will
should have been delivered, the action was resisted by necessarily consume time and thus shorten the period
the defendants principally on the ground of for bringing suit to less than one year if the period is to
prescription. begin, as stated in the policy, from "the happening of
the loss." Being contrary to the law of the forum, such
The trial court found for plaintiff and rendered stipulation cannot be given effect.
judgment in his favor for the sum claimed plus legal
interest and costs. The judgment was affirmed by the When did the cause of action accrue? On that question
Court of Appeals. we agree with the court below that plaintiff's cause of
action did not accrue until his claim was finally rejected
Issue: by the insurance company. This is because, before
such final rejection, there was no real necessity for
whether plaintiff's action has prescribed bringing suit. As the policy provides that the insured
should file his claim, first, with the carrier and then
Ruling: with the insurer, he had a right to wait for his claim to
be finally decided before going to court. The law does
On the part of the carrier the defense of prescription is not encourages unnecessary litigation.
made to rest on the following stipulation of the bill of
lading: In concluding, we may state that contractual
In any event the carrier and the ship shall be limitations contained in insurance policies are regarded
discharged from all liability in respect of loss or with extreme jealousy by courts and will be strictly
damage unless suit is brought within one year construed against the insurer and should not be

EH 403 2010-2011 2011-2012


Page 33
INSURANCE LAW CASE DIGESTS
permitted to prevent a recovery when their just and of destruction have not yet disappeared. It is in the
honest application would not produce that result. nature of a condition precedent to the liability of the
Wherefore, the judgment appealed from is reversed insurer, or in other terms, a resolutory cause, the
with respect to the carrier and its agents but affirmed purpose of which is to terminate all liabilities in case
with respect to the insurance company and its agents, the action is not filed by the insured within the period
with costs against the latter. stipulated.

The bringing of the action against the Paramount


Digested by: Patrina T. Soco Surety & Insurance Company, the agent of the
Ang vs. Fulton Fire Insurance Co. defendant Company cannot have any legal effect
except that of notifying the agent of the claim. Beyond
FACTS: such notification, the filing of the action can serve no
other purpose. There is no law giving any effect to
On September 9, 1953, defendant Fulton Fire such action upon the principal. Besides, there is no
Insurance Company issued a policy No. F-4730340, in condition in the policy that the action must be filed
favor of P. & S Department Store (Sally C. Ang) over against the agent, and this Court cannot by
stocks of general merchandise, consisting principally of interpretation, extend the clear scope of the
dry goods, contained in a building occupied by the agreement beyond what is agreed upon by the parties.
plaintiffs at Laoag, Ilocos Norte. The insurance was
issued for one year, but the same was renewed for Decision of RTC reversed.
another year on September 31, 1954. On December
17, 1954, the store containing the goods insured was
destroyed by fire. On December 30, following, plaintiffs Sun Insurance vs CA
executed the first claim form. On April 6, 1956, the [G.R. No. 89741 March 13, 1991]
Fulton Fire Insurance Company wrote the plaintiffs that
their claim was denied. This denial of the claim was Facts:
received by the plaintiffs on April 19, 1956. Herein private respondent Emilio Tan took from
petitioner, Sun Insurance, a P300,000.00 property
Defendant claims that under paragraph 13 of the insurance policy to cover his interest in the electrical
policy, if the loss or damage is occasioned by the supply store of his brother housed in a building in Iloilo
willful act of the insured, or if the claim is made and City. Four (4) days after the issuance of the policy, the
rejected but no action is commenced within 12 months building was burned including the insured store. On
after such rejection, all benefits under the policy would August 20, 1983, Tan filed his claim for fire loss with
be forfeited, and that since the claim of the plaintiffs petitioner, but on February 29, 1984, petitioner wrote
was denied and plaintiffs received notice of denial on Tan denying the latter's claim. On April 3, 1984, Tan
April 18, 1956, and they brought the action only on wrote petitioner, seeking reconsideration of the denial
May 5, 1958, all the benefits under the policy have of his claim. On September 3, 1985, Tan's counsel
been forfeited. wrote the Insurer inquiring about the status of his April
The court below held that the bringing of the action in 3, 1984 request for reconsideration. Petitioner
the Court of First Instance of Manila on May 11, 1956 answered the letter on October 11, 1985, advising
to assert their claim, tolled the running of the 12 Tan's counsel that the Insurer's denial of Tan's claim
month period within which the action must be filed. It remained unchanged, enclosing copies of petitioners'
said that even if the case was filed against the agent letters of February 29, 1984 and May 17, 1985
rather than the defendant, it was merely a procedural (response to petition for reconsideration). On
mistake. The complaint was dismissed by the Court November 20, 1985 filed a case with the RTC of Iloilo,
without prejudice on September 3, 1957, and motion but petitioner filed a motion to dismiss on the alleged
for reconsideration dated September 21, 1957. The ground that the action had already prescribed. Said
instant complaint was filed on May 8, 1958 therefore it motion was denied; and petitioner's motion for
is still within the one-year prescriptive period. reconsideration was also denied. CA also denied his
appeal. Thus the instant petition.
ISSUE:
Issue:
Whether the filing of the previous suit tolled or
suspended the running of the prescriptive period. WON the RTC and CA is correct in ruling against Sun
Insurance when it filed a motion to dismiss Emilio
RULING: Tans complaint on the ground that petitioners action
has already prescribed.
The condition contained in the insurance policy that
claims must be presented within one year after Corollary issue:
rejection is not merely a procedural requirement. The
condition is an important matter, essential to a prompt WON that MOTION FOR RECONSIDERATION filed by
settlement of claims against insurance companies, as it Tan on April 3, 1984 INTERRUPTS THE TWELVE (12)
demands that insurance suits be brought by the MONTHS PRESCRIPTIVE PERIOD TO CONTEST THE
insured while the evidence as to the origin and cause DENIAL OF THE INSURANCE CLAIM.

EH 403 2010-2011 2011-2012


Page 34
INSURANCE LAW CASE DIGESTS
But as pointed out by the petitioner insurance
Held: company, the rejection referred to should be construed
as the rejection, in the first instance, for if what is
The petition was considered and the CA decision, which being referred to is a reiterated rejection conveyed in a
affirms RTC decision, was reversed and set aside. SC resolution of a petition for reconsideration, such should
ruled that the MR filed by Tan on April 1984 did not have been expressly stipulated.
interrupt the twelve months prescriptive period.
Thus, to allow the filing of a motion for reconsideration
While it is a cardinal principle of insurance law that a to suspend the running of the prescriptive period of
policy or contract of insurance is to be construed twelve months, a whole new body of rules on the
liberally in favor of the insured and strictly against the matter should be promulgated so as to avoid any
insurer company, yet, contracts of insurance, like other conflict that may be brought by it.
contracts, are to be construed according to the sense
and meaning of the terms which the parties This Court uses the phrase "final rejection", the same
themselves have used. If such terms are clear and cannot be taken to mean the rejection of a petition for
unambiguous, they must be taken and understood in reconsideration as insisted by respondents. Such was
their plain, ordinary and popular sense. Condition 27 of clearly not the meaning contemplated by this Court.
the Insurance Policy, which is the subject of the The Insurance policy in said case provides that the
conflicting contentions of the parties, reads: insured should file his claim, first, with the carrier and
27. Action or suit clause If a claim be made then with the insurer. The "final rejection" being
and rejected and an action or suit be not referred to in said case is the rejection by the
commenced either in the Insurance insurance company.
Commission or in any court of competent
jurisdiction within twelve (12) months from
receipt of notice of such rejection, or in case of Digested by: Ugbinar, Krystel Marie
arbitration taking place as provided herein, ACCFA vs. Alpha Insurance and Surety Co. Inc.
within twelve (12) months after due notice of [G.R. No. L-24566 July 29, 1968]
the award made by the arbitrator or arbitrators
or umpire, then the claim shall for all purposes Facts:
be deemed to have been abandoned and shall
not thereafter be recoverable hereunder. According to the allegations of the complaint, in order
to guarantee the Asingan Farmers' Cooperative
Respondent Tan, in his letter addressed to the Marketing Association, Inc. (FACOMA) against loss on
petitioner insurance company dated April 3, 1984 account of "personal dishonesty, amounting to larceny
(Rollo, pp. 50-52), admitted that he received a copy of or estafa of its Secretary-Treasurer, Ricardo A.
the letter of rejection on April 2, 1984. Thus, the 12- Ladines, the appellee, Alpha Insurance & Surety
month prescriptive period started to run from the said Company had issued, on 14 February 1958, its bond,
date of April 2, 1984, for such is the plain meaning and No. P-FID-15-58, for the sum of Five Thousand Pesos
intention of Section 27 of the insurance policy. (P5,000.00) with said Ricardo Ladines as principal and
the appellee as solidary surety. On the same date, the
The condition contained in an insurance policy that Asingan FACOMA assigned its rights to the appellant,
claims must be presented within one year after Agricultural Credit Cooperative and Financing
rejection is not merely a procedural requirement but Administration (ACCFA for short), with approval of the
an important matter essential to a prompt settlement principal and the surety.
of claims against insurance companies as it demands
that insurance suits be brought by the insured while During the effectivity of the bond, Ricardo Ladines
the evidence as to the origin and cause of destruction converted and misappropriated, to his personal
have not yet disappeared. benefit, some P11,513.22 of the FACOMA funds, of
which P6,307.33 belonged to the ACCFA. Upon
It is apparent that Section 27 of the insurance policy discovery of the loss, ACCFA immediately notified in
was stipulated pursuant to Section 63 of the Insurance writing the survey company on 10 October 1958, and
Code, which states that: presented the proof of loss within the period fixed in
Sec. 63. A condition, stipulation or agreement the bond; but despite repeated demands the surety
in any policy of insurance, limiting the time for company refused and failed to pay. Whereupon, ACCFA
commencing an action thereunder to a period filed suit against appellee on 30 May 1960.
of less than one year from the time when the
cause of action accrues, is void. Defendant Alpha Insurance & Surety Co., Inc., (now
appellee) moved to dismiss the complaint for failure to
The insured's cause of action or his right to file a claim state a cause of action, giving as reason that (1) the
either in the Insurance Commission or in a court of same was filed more than one year after plaintiff made
competent jurisdiction commences from the time of claim for loss, contrary to the eighth condition of the
the denial of his claim by the Insurer, either bond, providing as follows: .
expressly or impliedly.
EIGHT LIMITATION OF ACTION

EH 403 2010-2011 2011-2012


Page 35
INSURANCE LAW CASE DIGESTS
No action, suit or proceeding shall be had or so far, therefore, as condition eight of the bond
maintained upon this Bond unless the same be requires action to be filed within one year from the
commenced within one year from the time of filing of the claim for loss, such stipulation contradicts
making claim for the loss upon which such the public policy expressed in Section 61-A of the
action, suit or proceeding, is based, in Philippine Insurance Act. Condition eight of the bond,
accordance with the fourth section hereof. therefore, is null and void, and the appellant is not
bound to comply with its provisions.
(2) the complaint failed to show that plaintiff had filed
civil or criminal action against Ladines, as required by As a consequence of the foregoing, condition eight of
conditions 4 and 11 of the bond; and (3) that Ladines the Alpha bond is null and void, and action may be
was a necessary and indispensable party but had not brought within the statutory period of limitation for
been joined as such. written contracts (New Civil Code, Article 1144). The
case of Ang vs. Fulton Fire Insurance Co., 2 S.C.R.A.
At first, the Court of First Instance denied dismissal; 945 (31 July 1961), relied upon by the Court a quo, is
but, upon reconsideration, the court reversed its no authority against the views herein expressed, since
original stand, and dismissed the complaint on the the effect of Section 61-A of the Insurance Law on the
ground that the action was filed beyond the contractual terms of the Policy or contract was not there
limitation period considered.

Issue: The condition of previous conviction (paragraph b,


clause 4, of the contract) having been deleted by
whether or not the provision of a fidelity bond that no express agreement and the surety having assumed
action shall be had or maintained thereon unless solidary liability, the other grounds of the motion to
commenced within one year from the making of a dismiss are equally untenable. A creditor may proceed
claim for the loss upon which the action is based, is against any one of the solidary debtors, or some or all
valid or void, in view of Section 61-A of the Insurance of them simultaneously (Article 1216, New Civil Code).
Act invalidating stipulations limiting the time for
commencing an action thereon to less than one year WHEREFORE, the appealed order granting the motion
from the time the cause of action accrues. to dismiss is reversed and set aside, and the records
are remanded to the Court of First Instance, with
Ruling: instructions to require defendant to answer and
thereafter proceed in conformity with the law and the
We find the appeal meritorious. Rules of Court. Costs against appellee. So ordered.

A fidelity bond is, in effect, in the nature of a contract


of insurance against loss from misconduct, and is Digested by: Sheiryl G. Vinas
governed by the same principles of interpretation: Mayer Steel Pipe Corp vs. CA
Mechanics Savings Bank & Trust Co. vs. Guarantee
Company, 68 Fed. 459; Pao Chan Wei vs. Nemorosa, Facts:
103 Phil. 57. Consequently, the condition of the bond
in question, limiting the period for bringing action Hongkong Govt. contracted Mayer Steel to
thereon, is subject to the provisions of Section 61-A of manufacture steel pipes. The pipes covered by 3
the Insurance Act (No. 2427), as amended by Act invoices were insured with South Sea Insurance for
4101 of the pre-Commonwealth Philippine Legislature, $212K, while the others under another 3 separate
prescribing that: invoices where insured with Charter Insurance for
SEC. 61-A A condition, stipulation or agreement $149K. They were issued an all risks insurance
in any policy of insurance, limiting the time for policies. When the pipes reached Hongkong, it was
commencing an action thereunder to a period of damaged, though it was certified in good condition
less than one year from the time when the cause before it was loaded in the ship. Mayer filed a claim
of action accrues is void. with Charter but they were only given HK$49K and so
they filed a case for the remainder of the claim in
Since a "cause of action" requires, as essential court. RTC found for Mayer, which was reversed by CA,
elements, not only a legal right of the plaintiff and a on the ground of prescription.
correlative obligation of the defendant but also "an act
or omission of the defendant in violation of said legal Issue:
right" (Maao Sugar Central vs. Barrios, 79 Phil. 666),
the cause of action does not accrue until the party WON COGSA would apply, and therefore the cause of
obligated refuses, expressly or impliedly, to comply action has prescribed.
with its duty (in this case, to pay the amount of the
bond). The year for instituting action in court must be Ruling:
reckoned, therefore, from the time of appellee's refusal
to comply with its bond; it can not be counted from the No. COGSA, does not apply and the cause of action will
creditor's filing of the claim of loss, for that does not prescribe only after 10 years. COGSA applies to a
import that the surety company will refuse to pay. In claim against the carrier under the contract of

EH 403 2010-2011 2011-2012


Page 36
INSURANCE LAW CASE DIGESTS
carriage, which must be filed within 2 years from Facts: This is the story of your moms life.
delivery of the goods. This is a claim against the (oops.)
insurer under the insurance contract and therefore will
prescribe in 10 years. This is a story about a consignee/buyer who bought
fishmeal products from Bangkok and had it delivered
Respondent court erred in applying Section 3(6) of the to the port of Manila. He entered into an insurance
Carriage of Goods by Sea Act. contract with defendant insurance company
(FilMerchant) under policy no. M-2678 for P267,653.59
Section 3(6) of the Carriage of Goods by Sea Act and for goods described as 600 metric tons of fishmeal
states that the carrier and the ship shall be discharged in new gunny bags of 90 kilos each. What was actually
from all liability for loss or damage to the goods if no imported was 59.940mtons in 666 gunny bags. Upon
suit is filed within one year after delivery of the goods arrival at Manila, arrastre and defendants surveyor
or the date when they should have been delivered. found 227 bags in bad order condition. Because of this
Under this provision, only the carrier's liability is loss, buyer formally claimed from FilMerchant but the
extinguished if no suit is brought within one year. But said insurance company refused to pay. He brought
the liability of the insurer is not extinguished because suit. Trial court ruled for him and against FilMerchant,
the insurer's liability is based not on the contract of CA affirmed trial court hence this petition.
carriage but on the contract of insurance.
FilMerchant argues:
The Filipino Merchants case is different from the case (1) CA erred in the interpretation and application of
at bar. In Filipino Merchants, it was the insurer which the all risk clause of maritime insurance policy. It
filed a claim against the carrier for reimbursement of says it should not be held liable for partial loss
the amount it paid to the shipper. The ruling in Filipino notwithstanding the clear absence of proof of some
Merchants should apply only to suits against the carrier fortuitous event, casualty, or accidental cause to which
filed either by the shipper, the consignee or the the loss is attributable.
insurer. When the court said in Filipino Merchants that
Section 3(6) of the Carriage of Goods by Sea Act (2) Respondent had no insurable interest in the subject
applies to the insurer, it meant that the insurer, like cargo. The shipment reveals that it is a C & F
the shipper, may no longer file a claim against the contract of shipment. The seller, not the consignee,
carrier beyond the one-year period provided in the law. paid for the shipment. As there was yet no delivery to
But it does not mean that the shipper may no longer the consignee, ownership (and interest) does not yet
file a claim against the insurer because the basis of the pass to him.
insurer's liability is the insurance contract. An
insurance contract is a contract whereby one party, for Issues:
a consideration known as the premium, agrees to
indemnify another for loss or damage which he may W/N CA was correct in its interpretation of the all
suffer from a specified peril . An "all risks" insurance risk clause in the maritime
policy covers all kinds of loss other than those due to insurance contract.
willful and fraudulent act of the insured. 12 Thus, when
private respondents issued the "all risks" policies to W/N the insured had insurable interest over the
petitioner Mayer, they bound themselves to indemnify property insured.
the latter in case of loss or damage to the goods
insured. Such obligation prescribes in ten years, in Ruling:
accordance with Article 1144 of the New Civil Code.
a) All risks policy has no technical meaning.
Granted. CA reversed. The clause in question reproduced:
5. This insurance is against all risks of loss or
damage to the subject-matter insured but shall
in no case be deemed to extend to cover loss,
*In no event shall the publisher be held liable for any damage, or expense proximately caused by
consequential damages arising out of or related to the delay or inherent vice or nature of the subject-
use or inability to use this case brief material. matter insured. Claims recoverable hereunder
INSURANCE LAW barcrammer_jed shall be payable irrespective of percentage
limitation to commence action/insurable
interest in property An "all risks policy" should be read literally as meaning
FILIPINO MERCHANTS INSURANCE CO., Inc. v. all risks whatsoever and covering all losses by an
CA accidental cause of any kind. The very nature of the
GR No. 85141 Nov. 28, 1989 Second Division term "all risks" must be given a broad and
[Regalado] comprehensive meaning as covering any loss other
than a willful and fraudulent act of the insured. 7 This
Nature: is pursuant to the very purpose of an "all risks"
Petition for review on certiorari challenging a CA insurance to give protection to the insured in those
decision finding FilMerchant liable to pay plaintiff. cases where difficulties of logical explanation or some
mystery surround the loss or damage to property.

EH 403 2010-2011 2011-2012


Page 37
INSURANCE LAW CASE DIGESTS
it was not accompanied by the surrender of the policy
Burden of proof shifts to the insurer. for cancellation,
Generally, the burden of proof is upon the insured to
show that a loss arose from a covered peril, but under Issue:
an "all risks" policy the burden is not on the insured to
prove the precise cause of loss or damage for which it Whether the policy as effectively cancelled or not.
seeks compensation. The insured under an "all risks
insurance policy" has the initial burden of proving that Held:
the cargo was in good condition when the policy
attached and that the cargo was damaged when This case hinges on the interpretation of paragraph 10
unloaded from the vessel; thereafter, the burden of the policy, reading:
then shifts to the insurer to show the exception This insurance may be terminated anytime at the
to the coverage. request of the Insured, in which case the Company will
retain the customary period rate for the time the
b) Vendee/Consignee has insurable interest policy has been enforced. This insurance may also at
SC: The shipment contract being that of cost and any time be terminated at the option of the Company,
freight (C&F) is immaterial in the determination on notice to that effect being given to the insured, in
of insurable interest. The perfected contract of which case the Company shall be liable to repay on
sale vests in the vendee an equitable title, an demand a ratable proportion of the premium for the
interest sufficient enough to be insurable. expired term from the date of cancellment
Further, Art. 1523 NCC provides that where, in
pursuance of a contract of sale, the seller is Pursuant to this stipulation,the contract in question
authorized or required to send the goods to the could be terminated at any time upon the unilateral
buyer, delivery of the goods to a carrier, whether act of either party. Whichever party exercised the
named by the buyer or not, for, the purpose of option did not need the approval. Consent nor
transmission to the buyer is deemed to be a concurrence of the other thereto, That consent was
delivery of the goods to the buyer, the exceptions given at the making of the contract. Moreover,
to said rule not obtaining in the present case. The pursuant to her letter, plaintiff considered the contract
Court has heretofore ruled that the delivery of terminated upon receipt of said letter by the defendant
the goods on board the carrying vessels (deade el recibo de la presente0,
partake of the nature of actual delivery since,
from that time, the foreign buyers assumed the Decision of the lower court dismissing the4 action to
risks of loss of the goods and paid the insurance recover the amount pf fire insurance policy is affirmed.
premium covering them.
(note: this ruling is deemed modigfied by Sec. 65}
WHEREFORE, the instant petition is DENIED and the
assailed decision of the respondent Court of Appeals is
AFFIRMED in toto. Saura Import & Export Co. vs. Phil. Inter-
National Surety Co., Inc. and Philippine National
x. Cancellation of Policy (Sections 64-65) Bank
[G.R. No. L-15184, May 31, 1963]
Digested by: Vicente Ynclino
Paulino vs Capital Ins. & Surety Co. Facts:
[105 Phil. 1315, May 15, 1959]
Saura Import & Export Co Inc., mortgaged to the
Facts: Phil. National Bank Davao, a parcel of land to
secure the payment of promissory note of P27,000.00
Plaintiff Paulino secured a fire insurance policy issued with a condition from PNB that Saura shall insure the
by defendant Capital Insurance on Feb. 8, 1952. On mortgaged property at all times against fire and
April 30, 1952 the Plaintiff wrote the defendant earthquake for an amount satisfactory to PNB,
requesting cancellation of the policy, which the latter indorsing to the latter policies of such insurance.
received on May 10, 1952.
Pursuant to the requirement, Saura insured the
The plaintiff did not return the policy nor de4manded building together with its contents with the Philippine
for the return of the proportionate premium and International Surety, an insurance firm acceptable
neither did the defendant offer to breturn the to PNB for P29,000.00, against fire for the period of 1
premium. The property covered by the policy was year from October 2, 1954 and as further required the
destroyed by fire on aug. 15, 1952. insurance policy was endorsed to PNB.

The defendant refused to pay plaintiffs claim on the 13 days after the issuance of the fire insurance policy,
ground that the policy was cancelled as of May 10, insurer cancelled it, effective as of the date of issue.
1952. Plaintiff contends in this appeal that her letter, Notice of cancellation was given to PNB in writing, sent
dated April 30, 1952, was a mere request orb offer to via Registered Mail, personally addressed to Fortunato
cancel the policy and did not terminate the same since

EH 403 2010-2011 2011-2012


Page 38
INSURANCE LAW CASE DIGESTS
Domingo, PNB Branch Manager. The latter received it primary duty of the insurance company to notify the
on Nov. 8, 1954. insured, but it did not.

On April 6, 1955, the building and its contents worth Thus, liability attached principally to the insurance
P40,685.69 were burned. Saura filed a claim and upon company, for its failure to give notice of the
presentation of notice of loss with PNB, Saura learned cancellation of the policy to Saura.
for the 1st time that the policy had previously been
cancelled. Insurer refused to pay Saura.
Malayan Insurance vs. Cruz
Issue:
Facts:
Whether or not the notice of cancellation of the policy
to the mortgagee (PNB) was already a substantial Coronacion Pinca has been sustained by the Insurance
compliance of the insurers duty to notify the insured Commission in her claim for compensation for her
of the policy cancellation? burned house. The petitioner now disputes the decision
on the ground that there was no valid insurance
Ruling: contract at the time of the loss because they have
already cancelled the policy for non-payment of
Fire insurance policies and other contracts of insurance premium.
upon property, in addition to the common provision for
cancellation of the policy upon request of the insured, Issue:
generally provide for cancellation by the insurer by
notice to the insured for a prescribed period, which is WON Section 77 of the insurance Code is applicable in
usually 5 days, and the return of the unearned portion this case.
of the premium paid by the insured, such provision for
cancellation upon notice being authorized by statutes Ruling:
in some jurisdiction, either specifically or as a provision
of an adopted standard form of policy. Section 77 is not applicable because payment of
premium was in fact eventually made. There was a
The purpose of provisions or stipulations for notice to premium invoice which was issued to Pinca given by
the insured, is to prevent the cancellation of the policy, the agent of the petitioner who received the payment
without allowing the insured ample opportunity to in behalf of Malayan Insurance.
negotiate for other insurance in its stead. The form
and sufficiency of a notice of cancellation is determined According to Section 65 of the Insurance Code a valid
by policy provisions. cancellation of policy requires the concurrence of the
following conditions 1) Prior notice of cancellation to
In order to form the basis for the cancellation of a insured; 2) Notice must be based on the occurrence
policy, notice to the insured need not be in any after effective date of policy of one of more of the
particular form, in the absence of a statute or policy grounds mentioned; 3) Must be in writing, mailed or
provision prescribing such form, and it is sufficient, so delivered to the insured at the address shown on the
long as it positively and unequivocally indicates to the policy; 4) Must state the grounds relied upon provided
insured, that it is the intention of the company that the in Section 64 of the Insurance Code and upon request
policy shall cease to be binding. Where the policy of the insured, to furnish facts on which cancellation is
contains no provisions that a certain number of days based.
notice shall be given, a reasonable notice and
opportunity to obtain other insurance must be given. However, there was no proof of notice, assuming it
Actual personal notice to the insured is essential to a complied with the other requisites mentioned above,
cancellation under a provision for cancellation by was actually mailed to and received by Pinca. All
notice. The actual receipt by the insured of a notice of Malayan Insurance offers to show that the cancellation
cancellation is universally recognized as a condition was communicated to the insured is its employees
precedent to a cancellation of the policy by the insurer, testimony that the said cancellation was sent by mail
and consequently a letter containing notice of through our mailing section without more.
cancellation which is mailed by the insurer but not
received by the insured, is ineffective as cancellation.
xi. Renewal of Policy (Section 66)
The notice should be personal to the insured and not to
and/or through any unauthorized person by the policy. G. Warranties (Sections 67-76)
In the case at bar, the defendant insurance company, i. Definition, kinds and form of warranty
must have realized the paramount importance of (Section 67-69)
sending a notice of cancellation, when it sent the ii. Express warranty (Sections 70-71)
notice of cancellation of the policy to the bank (as iii. Promissory warranty (Sections 72-73)
mortgagee), but not to the insured with which it iv. Violation of material warranty, effect
(insurance company) had direct dealing. It was the (Section 74)

EH 403 2010-2011 2011-2012


Page 39
INSURANCE LAW CASE DIGESTS
Digested by Margaret Frances Aparte
E. M. BACHRACH, vs. BRITISH AMERICAN It may be added that there was no provision in the
ASSURANCE COMPANY, policy prohibiting the keeping of paints and varnishes
[G.R. No. L-5715 December 20, 1910] upon the premises where the insured property was
stored. If the company intended to rely upon a
FACTS: condition of that character, it ought to have been
plainly expressed in the policy.
Bachrach insured his building against fire with the
British-American Assurance Company. After the II.
effectivity of the policy, the insured stored gasoline,
paints and varnishes within the premises insured. The Upon reading the policy of insurance issued by the
building was burned and the insurer refused to pay the defendant to the plaintiff, it will be noted that there is
loss on the ground that the risk of fire was increased no provision in said policy prohibiting the plaintiff from
by the storage of gasoline, paints and varnishes. The placing a mortgage upon the property insured, but,
insurer also claimed that the plaintiff transferred his admitting that such a provision was intended, we think
interest in and to the property covered by the policy to the lower court has completely answered this
H. W. Peabody & Co. to secure certain indebtedness contention of the defendant. He said, in passing upon
due and owing to said company, and also that the this question as it was presented:
plaintiff had transferred his interest in certain of the It is claimed that the execution of a chattel
goods covered by the said policy to one Macke, to mortgage on the insured property violated what is
secure certain obligations assumed by the said Macke known as the "alienation clause," which is now
for and on behalf of the insured. Such execution of a found in most policies, and which is expressed in
chattel mortgage on the insured property without the policies involved in cases 6496 and 6497 by a
consent to the insurer violated what is known as the purchase imposing forfeiture if the interest in the
"alienation clause,". property pass from the insured. (Cases 6496 and
6497, in which are involved other action against
ISSUE: other insurance companies for the same loss as in
the present action.)
I. Whether the use of the building as a paint and
varnish shop annulled the policy insurance. This clause has been the subject of a vast number of
II. Whether the execution of the chattel mortgages judicial decisions (13 Am. & Eng. Encyc. of Law, 2d
without the knowledge and consent of the insurance ed., pp. 239 et seq.), and it is held by the great weight
company annulled the policy insurance. of authority that the interest in property insured does
not pass by the mere execution of a chattel mortgage
HELD: and that while a chattel mortgage is a conditional sale,
there is no alienation within the meaning of the
The court ruled in negative for both issues. insurance law until the mortgage acquires a right to
take possession by default under the terms of the
I. mortgage. No such right is claimed to have accrued in
the case at bar, and the alienation clause is therefore
The property insured consisted mainly of household inapplicable
furniture kept for the purpose of sale. The preservation
of the furniture in a salable condition by retouching or
otherwise was incidental to the business. Digested by: Kwin Asunto
Young vs. Midland Textile insurance company
The evidence offered by the plaintiff is to the effect [G.R. No. 9370. March 31, 1915.]
that alcohol was used in preparing varnish for the
purpose of retouching, though he also says that the Facts:
alcohol was kept in store and not in the bodega where
the furniture was. The purpose of the present action is to recover the
sum of P3,000 upon an insurance policy. The lower
It is well settled that the keeping of inflammable oils court rendered a judgment in favor of the plaintiff and
on the premises, though prohibited by the policy, does against the defendant for the sum of P2,708.78, and
not void it if such keeping is incidental to the business. costs. From that judgment the defendant appealed to
this court.
Thus, where a furniture factory keeps benzine for the
purposes of operation (Davis vs. Pioneer Furniture The undisputed facts upon which said action is based
Company, 78 N. W. Rep., 596; Faust vs. American Fire are as follows:
Insurance Company, 91 Wis., 158), or where it is used
for the cleaning machinery (Mears vs. Humboldt The plaintiff occupied a building at '321 Calle Claveria,
Insurance Company, 92 Pa. St., 15; 37 Am. Rep., as a residence and bodega (storehouse). On the 29th
647), the insurer can not on that ground avoid of May, 1912, the defendant, in consideration of the
payment of loss, though the keeping of the benzine on payment of a premium of P60, entered into a contract
the premises is expressly prohibited of insurance with the plaintiff promising to pay to the

EH 403 2010-2011 2011-2012


Page 40
INSURANCE LAW CASE DIGESTS
plaintiff the sum of P3,000, in case said residence and fulfillment of the terms of the contract. If the insured
bodega and contents should be destroyed by fire. One cannot bring himself within the terms and conditions of
of the conditions of said contract was that no the contract, he is not entitled to recover for any loss
hazardous goods be stored or kept in the building. suffered. The terms of the contract constitute the
measure of the insurer's liability. If the contract has
On the 4th or 5th of February, 1913, the plaintiff been terminated, by a violation of its terms on the part
placed in said residence and bodega three boxes which of the insured, there can be no recovery. Compliance
belonged to him and which were filled with fireworks with the terms of the contract is a condition precedent
for the celebration of the Chinese new year. to the right of recovery. Courts cannot make contracts
for the parties. While contracts of insurance are
On the 18th day of March, 1913, said residence and construed most favorably to the insured yet they must
bodega and the contents thereof were partially be construed according to the sense and meaning of
destroyed. Fireworks were found in a part of the the terms which the parties themselves have used.
building not destroyed by the fire; that they in no way Astute and subtle distinctions should not be permitted,
contributed to the fire, or to the loss occasioned when the language of the contract is plain and
thereby. unambiguous. Such distinctions tend to bring the law
itself into disrepute.
Issue:
The judgment of the lower court is revoked and the
Whether or not the placing of said fireworks in the defendant is relieved from any responsibility under
building insured, under the conditions above said complaint, and, without any finding as to costs.
enumerated, they being "hazardous goods," is a
violation of the terms of the contract of insurance.
Digested by: Rachel R. Aying
Held: UNION MANUFACTURING CO., INC. and the
REPUBLIC BANK vs. PHILIPPINE GUARANTY CO.,
Yes. INC

The word "stored" has been defined to be a deposit in Facts:


a store or warehouse for preservation or safe keeping;
to put away for future use, especially for future (1) Union Manufacturing Co., Inc. obtained certain
consumption; to place in a warehouse or other place of loans, overdrafts and other credit accommodations
deposit for safe keeping. Said definition does not from the Republic Bank for P415,000.00 with interest
include a deposit in a store, in small quantities, for at 9% per annum from said date and to secure the
daily use. "Daily use" precludes the idea of deposit for payment, Union Manufacturing executed a real and
preservation or safe keeping, as well as a deposit for chattel mortgages on certain properties.
future consumption or safe keeping.
(2) As additional condition of the mortgage contract,
A violation of the terms of a contract of insurance, by Union Manufacturing undertook to secure insurance
either party, will constitute the basis for a termination coverage over the mortgaged properties for the same
of the contractual relations, at the election of the amount of P415,000.00.
other. The right to terminate the contractual relations
exists even though the violation was not the direct (3) As Union Manufacturing failed to secure insurance
cause of the loss. In the present case, the deposit of coverage on the mortgaged properties, Republic Bank
the "hazardous goods," in the building insured, was a procured from the Philippine Guaranty Co., Inc. an
violation of the terms of the contract. Although the insurance coverage on loss against fire for
hazardous goods did not contribute to the loss, the P500,000.00 over the properties of the Union
insurer, at his election, was relieved from liability Said Manufacturing with the annotation that loss or
deposit created a new risk, not included in the terms of damage, if any, under is payable to Republic Bank as
the contract. The insurer had neither been paid, nor its interest may appear, subject however to the printed
had he entered into a contract, to cover the increased conditions of the Fire Insurance Policy Form;
risk.
(4) Fire Insurance Policy No. 43170 was issued for the
Contracts of insurance are contracts of indemnity, sum of P500,000.00 in favor of the assured, Union
upon the terms and conditions specified therein. Manufacturing for which the corresponding premium of
Parties have a right to impose such reasonable was paid by the Republic Bank to Philippine Guaranty
conditions at the time of the making of the contract as
they deem wise and necessary. The rate of premium is (5) Upon the expiration of said fire policy, the same
measured by the character of the risk assumed. The was renewed by the Republic Bank upon payment of
insurer, for a comparatively small consideration, the corresponding premium,
undertakes to guarantee the insured against loss or
damage, upon the terms and conditions agreed upon, (7) A fire occurred in the premises of the Union
and upon no other. When the insurer is called upon to Manufacturing,
pay, in case of loss, he may justly insist upon a

EH 403 2010-2011 2011-2012


Page 41
INSURANCE LAW CASE DIGESTS
(8) Union Manufacturing filed its fire claim with Company, it was categorically set forth that as a
Philippine Guaranty but was denied on the following condition precedent to the right of recovery, there
grounds: (a.)When Philippine Guaranty issued Fire must be compliance on the part of the insured with the
Insurance Policy No. 43170 ... in the sum of terms of the policy. As stated in the opinion of the
P500,000.00 to cover the properties of the Union Court through Justice Johnson: "If the insured has
Manufacturing the same properties were already violated or failed to perform the conditions of the
covered by Fire Policy of the Sincere Insurance contract, and such a violation or want of performance
Company and by insurance policies of the Oceanic has not been waived by the insurer, then the insured
Insurance Agency and (b) when Fire Insurance Policy cannot recover. Courts are not permitted to make
No. 43170 was already in full force and effect, Union contracts for the parties. The function and duty of the
Manufacturing without the consent of Philippine courts consist simply in enforcing and carrying out the
Guaranty Co., Inc., obtained other insurance policies contracts actually made. While it is true, as a general
totalling P305,000.00 over the same properties prior to rule, that contracts of insurance are construed most
the fire. favorably to the insured, yet contracts of insurance,
like other contracts, are to be construed according to
Issue: the sense and meaning of the terms which the parties
themselves have used. If such terms are clear and
whether or not Republic Bank can recover its interest unambiguous they must be taken and understood in
(as mortgagee) from the Fire Insurance Policy with their plain, ordinary and popular sense." More
Philippine Guaranty. specifically, there was a reiteration of this Santa Ana
ruling in a decision by the then Justice, later Chief
Ruling: Justice, Bengzon, in General Insurance & Surety Corp.
v. Ng Hua. Thus: "The annotation then, must be
In as much as the Union Manufacturing has violated deemed to be a warranty that the property was not
the condition of the policy to the effect that it did not insured by any other policy. Violation thereof entitles
reveal the existence of other insurance policies over the insurer to rescind. (Sec. 69, Insurance Act) Such
the same properties, as required by the warranty misrepresentation is fatal in the light of our views in
appearing on the face of the policy and that said Union Santa Ana v. Commercial Union Assurance Company,
Manufacturing Co., Inc. represented that there were no Ltd. ... . The materiality of non-disclosure of other
other insurance policies at the time of the issuance of insurance policies is not open to doubt." As a matter
said defendant's policy, and it appearing furthermore of fact, in a 1966 decision, Misamis Lumber Corp. v.
that while the policy of the defendant was in full force Capital Ins. & Surety Co., Inc., Justice J.B.L. Reyes,
and effect the Union Manufacturing Co., Inc. secured for this Court, made manifest anew its adherence to
other fire insurance policies without the written such a principle in the face of an assertion that thereby
consent of the defendant endorsed on the policy, the a highly unfavorable provision for the insured would be
conclusion is inevitable that both the Republic Bank accorded recognition. This is the language used: "The
and Union Manufacturing Co., Inc. cannot recover insurance contract may be rather onerous ('one sided',
from the same policy of the defendant because as the lower court put it), but that in itself does not
the same is null and void. justify the abrogation of its express terms, terms
which the insured accepted or adhered to and which is
It is to Santa Ana v. Commercial Union Assurance Co., the law between the contracting parties."
that one turns to for the first explicit formulation as to
the controlling principle. As was made clear in the
opinion of this Court, penned by Justice Villa-Real: H. Premium (Sections 77-82)
"Without deciding whether notice of other insurance i. Definition, when insured entitled to it
upon the same property must be given in writing, or (Section 77)
whether a verbal notice is sufficient to render an
insurance valid which requires such notice, whether Philippine Phoenix and Insurance Company vs.
oral or written, we hold that in the absolute absence of Woodworks Inc.
such notice when it is one of the conditions specified in [G.R. No. L-25317 August 6, 1979]
the fire insurance policy, the policy is null and void."
The next year, in Ang Giok Chip v. Springfield Fire & Facts:
Marine Ins. Co., the conformity of the insured to the
terms of the policy, implied from the failure to express Philippine Phoenix and Insurance Company ( Phil.
any disagreement with what is provided for, was Phoenix for short) issued a fire insurance policy in
stressed in these words of the ponente, Justice favor of Woodworks, Inc. upon application of the latter
Malcolm: "It is admitted that the policy before us was insuring Woodworks building and equipments against
accepted by the plaintiff. The receipt of this policy by loss by fire for a one year term from from July 21,
the insured without objection binds both the acceptor 1960 to July 21, 1961. After issuance of the policy,
and the insured to the terms thereof. The insured may Woodworks did not pay the premium as stipulated. On
not thereafter be heard to say that he did not read the April 19, 1961, Phil. Phoenix notified Woodworks of the
policy or know its terms, since it is his duty to read his cancellation of the policy at the same time claiming
policy and it will be assumed that he did so." As far earned premiums still unpaid by Woodworks from July
back as 1915, in Young v. Midland Textile Insurance 21, 1960 to April 19, 1961 worth P7,483.11 for 271

EH 403 2010-2011 2011-2012


Page 42
INSURANCE LAW CASE DIGESTS
days). In said letter, Phil. Phoenix credited the or assessment at the time provided for, the policy
remaining balance ( or from April 19, 1961 to July 21, shall become void or forfeited, or the obligation of
1961 equivalent to 3,110.25 for 94 days) to the insurer shall cease, or words to like effect,
Woodworks account. Woodworks naturally refused to because the contract so prescribes and because
pay Phil. Phoenix s alleged earned premiums such a stipulation is a material and essential part
averring that failure to pay the premium after the of the contract. This is true, for instance, in the
issuance of the policy rendered the insurance policy case of life, health and accident, fire and hail
unenforceable thereby prompting Phil. Phoenix legal insurance policies.
action for recovery of premiums before the lower court
( CFI) of which Phoenix won. Now on appeal before In fact, if the peril insured against had occurred,
the SC on pure questions of law. plaintiff, as insurer, would have had a valid defense
against recovery under the Policy it had issued. Explicit
Issue: in the Policy itself is plaintiff's agreement to indemnify
defendant for loss by fire only "after payment of
1. WON non-payment or failure as the case may be of premium," supra. Compliance by the insured with the
premiums after issuance of the fire insurance policy terms of the contract is a condition precedent to the
rendered such policy invalid or of no effect right of recovery.
The burden is on an insured to keep a policy in
2. Even if the premium is unpaid after issuance of the force by the payment of premiums, rather than on
policy by Phil. Phoenix, granting aguendo that indeed the insurer to exert every effort to prevent the
Phil. Phoenix gave Woodworks credit extensions for insured from allowing a policy to elapse through a
purposes of giving effect of the policy, was there failure to make premium payments. The
acceptance of such credit extended in this case? continuance of the insurer's obligation is
conditional upon the payment of premiums, so that
Ruling: no recovery can be had upon a lapsed policy, the
contractual relation between the parties having
1. Insurance policy became ineffective by non- ceased.
payment of premiums after issuance of policy. Section
77 of the Insurance Code (Presidential Decree No. 612, Moreover, "an insurer cannot treat a contract as valid
promulgated on December 18, 1974), provides that no for the purpose of collecting premiums and invalid for
contract of insurance issued by an insurance company the purpose of indemnity."
is valid and binding unless and until the premium
thereof has been paid, notwithstanding any agreement 2. No express acceptance of credit extensions, if any,
to the contrary. by Woodworks. The Policy provides for pre-payment of
premium. Accordingly; "when the policy is tendered
The insurance policy provides: the insured must pay the premium unless credit is
THE COMPANY HEREBY AGREES with the given or there is a waiver, or some agreement
Insured ... that if the Property above described, or obviating the necessity for prepayment." To constitute
any part thereof, shall be destroyed or damaged an extension of credit there must be a clear and
by Fire or Lightning after payment of Premium, at express agreement therefor."
any time between 4:00 o'clock in the afternoon of An acceptance of an offer to allow credit, if one
the TWENTY FIRST day of JULY One Thousand Nine was made, is as essential to make a valid
Hundred and SIXTY and 4:00 o'clock in the agreement for credit, to change a conditional
afternoon of the TWENTY FIRST day of JULY One delivery of an insurance policy to an unconditional
Thousand Nine Hundred and SIXTY ONE. ... delivery, as it is to make any other contract. Such
(Emphasis supplied) an acceptance could not be merely a mental act or
state of mind, but would require a promise to pay
Insurance is "a contract whereby one undertakes for a made known in some manner to defendant.
consideration to indemnify another against loss,
damage or liability arising from an unknown or
contingent event." The consideration is the
"premium". "The premium must be paid at the time
and in the way and manner specified in the policy and,
if not so paid, the policy will lapse and be forfeited by
its own terms."

Since the premium had not been paid, the policy must
be deemed to have lapsed.
The non-payment of premiums does not merely
suspend but put, an end to an insurance contract,
since the time of the payment is peculiarly of the
essence of the contract.
... the rule is that under policy provisions that
upon the failure to make a payment of a premium

EH 403 2010-2011 2011-2012


Page 43
INSURANCE LAW CASE DIGESTS
by accident, theft and fire for a period of one year
Digested by: Emmanuel from date thereof and every year thereafter until
Servicewide Specialists Inc. vs. CA the mortgage obligation is fully paid with an
[1996 May 8] insurance company, or companies acceptable to
the MORTGAGEE in an amount not less than the
Facts: outstanding balance of the mortgage obligation;
that he will make all loss, if any, under such policy
Respondent spouses Ricardo and Elisa Trinidad or policies, payable to the MORTGAGEE or its
purchased a 1983 Isuzu Gemini car from Autoworld assigns as its interest may appear and forewith
Sales Corporation. The price was P98,156.00 payable deliver such policy or policies to the MORTGAGEE,
in 24 equal monthly installments of P4,089.00 each the said MORTGAGOR further covenants and
month. To secure payment thereof, the Trinidads agrees that in default of his affecting or renewing
executed on the same date a promissory note and a such insurance and delivering the policies so
deed of chattel mortgage on the subject car in favor of endorsed to the MORTGAGEE within five (5) days
Autoworld. Autoworld assigned its interests on the after the execution of this mortgage or the expiry
promissory note and chattel mortgage to Filinvest. date of the insurance the MORTGAGEE, may, at his
These assignments were made with due notice to option, but without any obligation to do so effect
private respondents who delivered 17 checks to such insurance or obtain such renewal for the
Filinvest in full payment of the car. account of the MORTGAGOR and that any money
so disbursed by the MORTGAGEE shall be added to
On November 8, 1985, Filinvest assigned all its rights the principal indebtedness hereby secured and
and interests on the promissory note and chattel shall become due and payable at the time for the
mortgage in favor of Servicewide. payment of the immediately coming or following
installment to be due under the note aforesaid
On November 18, 1985, Ricardo Trinidad received a after the date of such insurance renewal and shall
demand letter from Servicewide stating that an bear interest at the same rate as the principal
assignment of credit had been made by Filinvest in its indebtedness.
favor and that the Trinidads had not paid two
successive installments on the car which had matured Petitioner contends that the matter about the notice is
on July 15 and August 15, 1985. No mention was deemed waived by private respondents because the
made in the letter that Filinvest had paid insurance car should be fully covered at all times. Petitioner
premiums to Perla Compania de Seguros to insure the claims that if, as stated in the Chattel Mortgage,
car against loss and damage corresponding to two private respondents failed to renew the insurance,
years. Trinidads were also never informed by Filinvest petitioner is entitled to renew the same for the account
that their installment payments on the car were of private respondents without any notice to them.
converted to premium payments on the insurance.
The petition is unmeritorious.
After informing private respondents that they failed to
pay the last two consecutive monthly installments, While it is true that the Chattel Mortgage does not say
petitioner demanded that either they pay the whole that notice to the mortgagor of the renewal of the
remaining balance of P6,977.67, including accrued insurance premium by the mortgagee is necessary, at
interest, or return possession of the car to petitioner. the same time, there is no provision that authorizes
petitioner to apply the payments made to it for the
When the Trinidads refused to pay the amount payment of the chattel to the payment of the said
demanded or to return the car, petitioner filed an premiums. From the records of the case, it is clear
action for replevin and damages. that private respondents had fully paid for the car.
This fact was never rebutted by petitioner; it was the
Issue: insurance premiums pertaining to the two-year period
from July 29, 1984 to July 29, 1986 that petitioner
WON petitioner should have applied the installment claims were not paid.
payments made by private respondents for the
payment of the car to the payment of the insurance Both the Regional Trial Court and the Court of Appeals
premiums without prior notice to private respondents. found that before the mortgagee (petitioner) may
effect the renewal of insurance, two conditions must be
Ruling: met: (1) Default by the mortgagor (private
respondents) in effecting renewal of the insurance and
The petition is unmeritorious. (2) failure to deliver the policy with endorsement to
petitioner.
The provision in the Chattel Mortgage subject of the
controversy states: The Court notes an additional element of the
provisions regarding the renewal of the insurance;
"The said MORTGAGOR covenants and agrees that specifically, that petitioner was under no obligation to
he will cause the property/ies herein above effect the same. In other words, petitioner as
mortgaged to be insured against loss or damage mortgagee was not duty-bound to renew the insurance

EH 403 2010-2011 2011-2012


Page 44
INSURANCE LAW CASE DIGESTS
in the event that private respondents failed to do so; it actually been paid; the latter provided that the
was merely optional on its part. premium corresponding to the first 90 days of the
policys term or any renewal is considered paid for the
The question now arises whether private respondents purpose of only making said policy valid and binding
were in default for failing to have the car covered by during said portion of the term and it shall
insurance for the period in question. Private automatically become void and ineffective (without
respondents claim that the car was duly covered and prejudice to the obligation of the insured to pay the
the Court finds no evidence on record showing this corresponding short premium for the said 90 days)
assertion to be false. Petitioner has averred, however, unless prior to the expiration of said period the
that the insurance taken by private respondents was INSURED shall have actually paid to the Company the
only for third-party liability and not the comprehensive total premium and the documentary stamps. Then,
insurance required. ACME, through its President, signed a promissory note
on May 26, 1964 promising to pay the INSURER within
If petitioner was aware that the insurance coverage 90 days from the effective date of the Policy (May 15,
was inadequate, why did it not inform private 1964) the premium and documentary stamps
respondent about it? After all, since petitioner was (P3,331.26) and failure to pay when due, it is agreed
under no obligation to effect renewal thereof, it is but that said policy should stand automatically cancelled.
logical that it should relay to private respondents any
defect of the insurance coverage before itself assuming A fire completely destroyed ACMEs properties on
the same. October13, 1964 prompting it to file a claim against
the INSURER which however disclaimed liability on the
Furthermore, even if the car were not covered with the ground that as of the date of loss, the properties
proper insurance, there is nothing in the provisions of burned were no covered by insurance. ACME has
the Chattel Mortgage that authorizes petitioner to obtained a favorable decision in the trial court when it
apply previous payments for the car to the insurance. sued for collection of insurance proceeds and damages
What is stated is: "x x x that any money so disbursed however the same was reversed by the CA affirming
by the mortgagee shall be added to the principal the INSURERs contention and thus it could not be held
indebtedness hereby secured x x x" talics supplied). liable for any indemnity.
Clear is it that petitioner is not obligated to convert
any of the installments made by private respondents Issue:
for the car to the payment for the renewal of the
insurance. Should it decide to do so, it has to send Whether or not there was an insurance contract at the
notice to private respondents who had already paid in moment of the loss.
full the principal indebtedness in question.
Ruling:
When petitioner wrote private respondents the
November 8, 1985 demand letter regarding non- CA is sustained. ACME, through its President, was fully
payment of the installments, no mention was made of aware that the policy would automatically be cancelled
unpaid insurance premiums. Thus, private if it will not pay the premium before August 13, 1964
respondents were quite justified in ignoring the same (90th day from May 14, 1964). In fact, reminders for
since, to the best of their knowledge, they had already payment being sent were unheeded. Not having paid
paid for the car in full. the 1964-1965 premium within the extension granted,
and pursuant to RA No. 3450, the policy was
automatically cancelled and there was no insurance
ii. Effects of non-payment/partial payment coverage to speak of as of the date of the fire.

Digested by: Birondo It was also ACMEs contention that the INSURER
ACME Shoe Rubber & Plastic Corp. vs. CA which accepted the one-year premium on January 8,
1964, has no right to apply it to the payment of a
Facts: period of coverage prior thereto for the policy was void
under RA 3450 (Sec. 72. xxx. No policy issued by an
Petitioner ACME had been insuring its properties yearly insurance company is valid and binding unless and
against fire with Domestic Insurance Company until the premium thereof has been paid). However,
(INSURER). It continued the insurance for the period of since said Act was approved only on June 20, 1963
May 15, 1962 up to May 15, 1963 for the amount of and was put into effect only on October 1, 1963, it
P200,000. On January 8, 1964, ACME only paid could not retroactively affect the renewal of the
P3,331.26, which the INSURER applied as renewal insurance policy on May 15, 1963. So, ACMEs
premium for the period May 15, 1963 to May 15, 1964 premium payment of January 8, 1964 was properly
and the issued renewal receipt having a stamp thereon applied to the 1963-1964 premium.
that it is subject to Receipt of Payment Clause and
Credit Agreement both attached as riders. The Lastly, ACMEs claim that the INSURER would unjustly
former clause declared that the insurance will be enrich itself if it were to be allowed to apply the one-
deemed valid and binding upon the company only year premium it received to a past period when the
when the premium and documentary stamps have policy was void and the INSURER had incurred no risk

EH 403 2010-2011 2011-2012


Page 45
INSURANCE LAW CASE DIGESTS
is flawed for the reason already stated that the grant credit extension for the premium due. No policy
renewal receipts were issued before RA 3450 was issued by an insurance company is valid and binding
approved and implemented. It is axiomatic that laws unless and until the premium thereof has been paid "
have no retroactive effect unless the contrary is (Italics supplied.) (p. 11, Appellant's Brief.)
provided. What became automatically cancelled by RA
3450 was the 1964-1965 policy for ACMEs failure to Morever, the parties in this case had stipulated:
pay the premium within the 90-day extension granted
and in accordance with the express terms of the IT IS HEREBY DECLARED AND AGREED that not.
promissory note it had signed. withstanding anything to the contrary contained in the
within policy, this insurance will be deemed valid and
Judgment affirmed. binding upon the Company only when the premium
and documentary stamps therefor have actually been
paid in full and duly acknowledged in an official receipt
Digested by: CANTILLAS, IRISH signed by an authorized official/representative of the
ARCE vs. THE CAPITAL INSURANCE & SURETY Company, " (pp. 45-46, Record on Appeal.)
CO., INC.,
It is obvious from both the Insurance Act, as amended,
FACTS: and the stipulation of the parties that time is of the
essence in respect of the payment of the insurance
In Civil Case No. 66466 of the Court of First Instance premium so that if it is not paid the contract does not
of Manila, the Capital Insurance and Surety Co., Inc., take effect unless there is still another stipulation to
(COMPANY) was ordered to pay Pedro Arce (INSURED) the contrary. In the instant case, the INSURED was
the proceeds of a fire insurance policy. Not satisfied given a grace period to pay the premium but the
with the decision, the company appealed to this Court period having expired with no payment made, he
on questions of law. cannot insist that the COMPANY is nonetheless
The INSURED(Pedro ARce) was the owner of a obligated to him.
residential house in Tondo, Manila, which had been
insured with the COMPANY(Capital insurance) since
1961 under Fire Policy No. 24204. On November 27, THE CAPITAL INSURANCE & SURETY CO., INC vs.
1965, the COMPANY sent to the INSURED Renewal PLASTIC ERA CO., INC., AND COURT OF APPEALS
Certificate No. 47302 to cover the period December 5, Digested by: marccarillo
1965 to December 5, 1966. The COMPANY also
requested payment of the corresponding premium in Facts:
the amount of P 38.10.
On Dec. 7, 1960, Capital Insurance & Surety Co., Inc
Anticipating that the premium could not be paid on (Capital Insurance) delivered to Plastic Era
time, the INSURED, thru his wife, promised to pay it Manufacturing Co., Inc (Plastic Era) its Open Fire Policy
on January 4, 1966. The COMPANY accepted the No.22760 wherein the former undertook to insure the
promise but the premium was not paid on January 4, latters building, equipments, raw materials, products
1966. On January 8, 1966, the house of the INSURED and accessories located at Sheridan Street,
was totally destroyed by fire. Mandaluyong, Rizal. The policy expressly provides that
if the property insured would be destroyed or damaged
On January 10, 1966, INSURED's wife presented a by fire after the payment of the premiums, at anytime
claim for indemnity to the COMPANY. She was told that between the Dec. 15 1960 and one o'clock in the
no indemnity was due because the premium on the afternoon of the Dec. 15, 1961, the insurance
policy was not paid. Nonetheless the COMPANY company shall make good all such loss or damage in
tendered a check for P300.00 as financial aid which an amount not exceeding P100k. Plastic Era failed to
was received by the INSURED's daughter, Evelina R. pay its premium and instead executed an
Arce. The COMPANY reiterated that the check was acknowledgment receipt promising to pay 30 days
given "not as an obligation, but as a concession" after date.
because the renewal premium had not been paid, The
INSURED cashed the check but then sued the On Jan.8,1961, Plastic Era delivered a check as partial
COMPANY on the policy. payment of the insurance premium worth 1k.
However, it was dishonored by the Bank of America for
ISSUE: lack of funds on Feb.20, 1961. Records revealed that
Plastic Era had a balance of P1,193.41 as early as
Whether the petitioners are entitled to claim from their Jan.19, 1961. (Note: premium due date was on
policy despite non-payment of their premium. Jan.16,1961)

RULING: Not unexpectedly, the property insured by Plastic Era


was destroyed by fire on Jan.18, 1961 (thats 2 days
SEC. 72. An insurer is entitled to payment of premium after the premium became due). Plastic Era filed a
as soon as the thing insured is exposed to the perils claim for indemnity. Capital Insurance wanted to
insured against, unless there is clear agreement to become part of Philippine Jurisprudence so it denied

EH 403 2010-2011 2011-2012


Page 46
INSURANCE LAW CASE DIGESTS
the claim on the ground that the premium was not would only pay for the loss or damage in case the
paid. same occurs after the payment of the premium.
Considering that the insurance policy is silent as to the
Pissed, Plastic Era then filed a case in the lower court mode of payment, Capital Insurance is deemed to
which renderred judgment in its favor. The appelate have accepted the promissory note in payment of the
court affirmed it and so the issue is brought before the premium.
SC.
Citing a US Case, our Supreme Court sad that that
Issue: although one of conditions of an insurance policy is
that "it shall not be valid or binding until the first
W/N a contract of insurance has been duly perfected premium is paid", if it is silent as to the mode of
between the petitioner, Capital Insurance, and payment, promissory notes received by the company
respondent Plastic Era. must be deemed to have been accepted in payment of
the premium. In other words, a requirement for the
Ruling: payment of the first or initial premium in advance or
actual cash may be waived by acceptance of a
Yup, it has been perfected. promissory note ...

The Insurnace Policy states that: THE COMPANY The fact that the check issued by Plastic Era in partial
HEREBY AGREES with the Insured but subject to the payment of the promissory note was later on
terms and conditions endorsed or otherwise expressed dishonored did not in any way operate as a forfeiture
hereon, which are to be taken as part of this Policy), of its rights under the policy, there being no express
that if the Property described, or any part thereof, stipulation therein to that effect.
shall be destroyed or damaged by Fire or This is how the story ends. :D
Lightning after payment of the Premiums, at anytime
between...xxx
Digested by: Brian O. Dalanon
In clear and unequivocal terms the insurance policy Velasco vs. Apostol
provides that it is only upon payment of the premiums
by Plastic Era that Capital Insurance agrees to insure Facts:
the properties of the former against loss or damage in
an amount not exceeding P100,000.00. It appears on That on November 27, 1973, at about 2:30 p.m.
record that on the day the insurance policy was plaintiffs were riding in their Mercury car driven by
delivered, Plastic Era did not pay the Capital their driver Restitute Guarra, along Quezon Boulevard
Insurance, but instead executed an acknowledgment when an N/S taxicab driven by defendant Dominador
receipt. Santos registered in the name of defendants Alice
Artuz, c/o Norberto Santos, crossed the center island
Q:Could not this have been considered a valid towards their direction, and finally collided with their
payment of the insurance premium? car at the left front part. The said taxicab tried to
return to its original lane, but was unable to climb the
Article 1249 NCC: island, and instead, backtracked, hitting again
plaintiffs' car in the left near portion, causing the
The delivery of promissory notes payable to order, or latter's back portion to turn toward the center hitting a
bills of exchange or other mercantile documents shall jeepney on its right.
produce the effect of payment only when they have
been cashed, or when through the fault of the creditor Originally sued as defendants were Dominador Santos,
they have been impaired. Alice Artuz, and Norberto Santos.
After an answer was filed by said defendants, private
Under this provision the mere delivery of a bill of respondent Maharlika Insurance Co., Inc. was
exchange in payment of a debt does not immediately impleaded as a defendant in an amended complaint
effect payment. It simply suspends the action arising filed by the petitioner with an allegation that the N/S
from the original obligation in satisfaction of which it taxicab was insured against third party liability for
was delivered, until payment is accomplished either P20,000 with private respondent at the time of the
actually or presumptively. But wait... accident.

Acceptance by Capital Insurance considered as In its answer to the amended complaint, respondent
waiver Maharlika Insurance Co., Inc. claimed that there was
no cause of action against it because at the time of the
Significantly, in the case before Us the Capital accident, the alleged insurance policy was not in force
Insurance accepted the promise of Plastic Era to pay due to non-payment of the premium. It further averred
the insurance premium within thirty (30) days from that even if the taxicab had been insured, the
the effective date of policy. By so doing, it has complaint would still be premature since the policy
implicitly agreed to modify the tenor of the insurance provides that the insurer would be liable only when the
policy and in effect, waived the provision therein that it insured becomes legally liable.

EH 403 2010-2011 2011-2012


Page 47
INSURANCE LAW CASE DIGESTS
The delivery of the policy made on March 28, 1974 and
The trial court rendered judgment in favor of the only because the premium was had been paid, in fact,
plaintiff finding that the evidence on the negligence of more than three months before such delivery.
defendant Dominador Santos was uncontroverted and
the proximate cause of the accident was his The said payment was accepted by the insurer without
negligence. 5 Defendants Dominador Santos, Alice any knowledge that the risk insured against had
Artuz, and Norberto Santos were adjudged jointly and already occurred since such fact was concealed by the
severally liable to petitioners. insured and was not revealed to the insurer.

However, Maharlika Insurance Co. was exonerated on The fact withheld could not in any event have
the ground that the policy was not in force for failure influenced the respondent company in entering into
of the therein defendants to pay the initial premium the supposed contract or in estimating the character of
and for their concealment of a material fact. the risk or in fixing the rate premium, for the simple
reason that no such contract existed between the
Petitioners fault the respondent-judge for considering defendants and the company at the time of the
private respondent's defense of late payment of accident.
premium when, according to them, "the same was
waived at the pre-trial. What should be apparent from such actuations of
therein defendants, however, is the presence of bad
Petiotioners theorize that what was stipulated in the faith on their part, a reprehensible disregard of the
pre-trail order does not include the issue on whether principle that insurance contracts are uberrimae fidae
defendant Maharlika Insurance Co., Inc. is liable under and demand the most abundant good faith.
the insurance policy, even as the premium was paid
after the accident in question.
Digested by: Grace Jayne Dingal
The accident for which respondent insurance company VALENZUELA v. COURT OF APPEALS, ARAGON et
is sought to be held liable occurred on November 27, al.
1973 while the initial premium was paid only on
December 11, 1973. Facts:

Petitioners maintain that in spite of this late payment, Arturo Valenzuela is a General Agent of Philippine
the policy is nevertheless binding because there was American General Insurance (Philamgen) since 1965.
an implied agreement to grant a credit extension so as He was authorized to solicit and sell in behalf of
to make the policy effective. To them, the subsequent Philamgen all kinds of non-life insurance, and in
acceptance of the premium and delivery of the policy consideration of services rendered was entitled to
estops the respondent company from asserting that receive the full agent's commission of 32.5% from
the policy is ineffective. Philamgen under the scheduled commission rates.
Issue: From 1973 to 1975, Valenzuela solicited marine
insurance from one of his clients, the Delta Motors in
Whether defendant Maharlika Insurance Co. Inc. is the amount of P4.4 Million from which he was entitled
liable under the insurance policy on account of the to a commission of 32%. However, Valenzuela did not
negligence of defendant Dominador Santos. receive his full commission which amounted to P1.6
Million from the P4.4 Million insurance coverage of the
Ruling: Delta Motors. In 1977, Philamgen started to become
interested in and expressed its intent to share in the
Petitioners' position is bereft of merit commission due Valenzuela on a fifty-fifty basis.
Because of the refusal of Valenzuela, Philamgen
Although there is no express statement as to the fact terminated the General Agency Agreement of
of late payment, this is necessarily deemed included in Valenzuela.
or ineluctably inferred from the issue of whether the
company is liable under the insurance policy it had Issue:
allegedly issued for the vehicle involved and on which
petitioners seek to recover. Whether or not Philamgen could continue to hold
Valenzuela jointly and severally liable with the insured
Issues that are impliedly included therein or may be for unpaid premiums
inferable therefrom by necessary implication are as
much integral parts of the pre-trial order as those that Held: NO.
are expressly stipulated.
In fact, it would be absurd and inexplicable for the The principal cause of the termination of Valenzuela as
respondent company to knowingly disregard or General Agent of Philamgen arose from his refusal to
deliberately abandon the issue of non-payment of the share his Delta commission. The apparent bad faith of
premium on the policy considering that it is the very the private respondents in terminating the General
core of its defense. Agency Agreement of petitioners. The agency involving
petitioner and private respondent is one "coupled with

EH 403 2010-2011 2011-2012


Page 48
INSURANCE LAW CASE DIGESTS
an interest," and, therefore, should not be freely correlatively he had also the right to sue for
revocable at the unilateral will of the latter. With the payment of the premium. But the amendment to
termination of the General Agency Agreement, Sec. 72 has radically changed the legal regime in
Valenzuela would no longer be entitled to commission that unless the premium is paid there is no
on the renewal of insurance policies of clients sourced insurance.
from his agency.
In Philippine Phoenix Surety case, we held:
Despite the termination of the agency, Philamgen Moreover, an insurer cannot treat a contract as
continued to hold Valenzuela jointly and severally valid for the purpose of collecting premiums and
liable with the insured for unpaid premiums. invalid for the purpose of indemnity.
Valenzuela had an interest in the continuation of the No contract of Insurance by an insurance company
agency when it was unceremoniously terminated not is valid and binding unless and until the premium
only because of the commissions he should continue to thereof has been paid, notwithstanding any
receive from the insurance business he has solicited agreement to the contrary
and procured but also for the fact that by the very acts
of the respondents, he was made liable to Philamgen in Since admittedly the premiums have not been paid,
the event the insured fail to pay the premiums due. the policies issued have lapsed. The insurance
They are estopped by their own positive averments coverage did not go into effect or did not continue and
and claims for damages. Therefore, the respondents the obligation of Philamgen as insurer ceased. Hence,
cannot state that the agency relationship between for Philamgen which had no more liability under the
Valenzuela and Philamgen is not coupled with interest. lapsed and inexistent policies to demand, much less
There is an exception to the principle that an agency is sue Valenzuela for the unpaid premiums would be the
revocable at will and that is when the agency has been height of injustice and unfair dealing. In this instance,
given not only for the interest of the principal but for with the lapsing of the policies through the
the interest of third persons or for the mutual interest nonpayment of premiums by the insured there were no
of the principal and the agent. In these cases, it is more insurance contracts to speak of. As this Court
evident that the agency ceases to be freely revocable held in the Philippine Phoenix Surety case, supra "the
by the sole will of the principal. non-payment of premiums does not merely suspend
but puts an end to an insurance contract since the time
The factor rendering Philamgen and the private of the payment is peculiarly of the essence of the
respondents liable in damages is that the termination contract."
by them of the General Agency Agreement was tainted The circumstances of the case, however, require that
with bad faith. If a principal acts in bad faith and with the contractual relationship between the parties shall
abuse of right in terminating the agency, then he is be terminated upon the satisfaction of the judgment.
liable in damages. No more claims arising from or as a result of the
agency shall be entertained by the courts after that
Valenzuela is not liable to Philamgen for the unpaid date.
and uncollected premiums. Under Section 77 of the ACCORDINGLY, the petition is GRANTED.
Insurance Code, the remedy for the non-payment of
premiums is to put an end to and render the insurance
policy not binding UCPB General Insurance vs. Masagana Telamart
Sec. 77 ... [N]otwithstanding any agreement to the Inc. ,
contrary, no policy or contract of insurance is valid [GR 137172]
and binding unless and until the premiums thereof
have been paid except in the case of a life or Facts:
industrial life policy whenever the grace period
provision applies On 15 April 1991, UCPB General Insurance Co. Inc.
(UCPBGen) issued 5 insurance policies covering
In Philippine Phoenix Surety v. Woodworks, we held Masagana Telamart, Inc.'s various property described
that the non-payment of premium does not merely therein against fire, for the period from 22 May 1991
suspend but puts an end to an insurance contract since to 22 May 1992. In March 1992, UCPBGen evaluated
the time of the payment is peculiarly of the essence of the policies and decided not to renew them upon
the contract. And in Arce v. The Capital Insurance and expiration of their terms on 22 May 1992. UCPBGen
Surety Co. Inc. (117 SCRA 63, [1982]), we reiterated advised Masagana's broker, Zuellig Insurance Brokers,
the rule that unless premium is paid, an insurance Inc. of its intention not to renew the policies. On 6
contract does not take effect. Thus: April 1992, UCPBGen gave written notice to Masagana
It is to be noted that Delgado (Capital Insurance & of the non-renewal of the policies at the address stated
Surety Co., Inc. v. Delgado, 9 SCRA 177 [1963] in the policies. On 13 June 1992, fire razed Masagana's
was decided in the light of the Insurance Act property covered by three of the insurance policies
before Sec. 72 was amended by the underscored UCPBGen issued. On 13 July 1992, Masagana
portion. Supra. Prior to the Amendment, an presented to UCPBGen's cashier at its head office 5
insurance contract was effective even if the manager's checks in the total amount of P225,753.95,
premium had not been paid so that an insurer was representing premium for the renewal of the policies
obligated to pay indemnity in case of loss and from 22 May 1992 to 22 May 1993. No notice of loss

EH 403 2010-2011 2011-2012


Page 49
INSURANCE LAW CASE DIGESTS
was filed by Masagana under the policies prior to 14 tendered on a latter date after the occurrence of the
July 1992. On 14 July 1992, Masagana filed with risk (fire) insured against.
UCPBGen its formal claim for indemnification of the
insured property razed by fire. On the same day, 14 Held:
July 1992, UCPBGen returned to Masagana the 5
manager's checks that it tendered, and at the same The answer is easily found in the Insurance Code. No,
time rejected Masagana's claim for the reasons (a) an insurance policy, other than life, issued originally or
that the policies had expired and were not renewed, on renewal, is not valid and binding until actual
and (b) that the fire occurred on 13 June 1992, before payment of the premium. Any agreement to the
Masagana's tender of premium payment. On 21 July contrary is void. The parties may not agree expressly
1992, Masagana filed with the Regional Trial Court, or impliedly on the extension of credit or time to pay
Branch 58, Makati City, a civil complaint against the premium and consider the policy binding before
UCPBGen for recovery of P18,645,000.00, representing actual payment. The case of Malayan Insurance Co.,
the face value of the policies covering Masagana's Inc. vs. Cruz-Arnaldo is not applicable. In that case,
insured property razed by fire, and for attorney's fees. payment of the premium was in fact actually made on
On 23 October 1992, after its motion to dismiss had 24 December 1981, and the fire occurred on 18
been denied, UCPBGen filed an answer to the January 1982. Here, the payment of the premium for
complaint. It alleged that the complaint "fails to state a renewal of the policies was tendered on 13 July 1992,
cause of action"; that UCPBGen was not liable to a month after the fire occurred on 13 June 1992. The
Masagana for insurance proceeds under the policies assured did not even give the insurer a notice of loss
because at the time of the loss of Masagana's property within a reasonable time after occurrence of the fire.
due to fire, the policies had long expired and were not Hence, the Supreme Court reversed and set aside the
renewed. After due trial, on 10 March 1993, the decision of the Court of Appeals in CA-GR CV 42321. In
Regional Trial Court, Branch 58, Makati, rendered lieu thereof, the Court rendered judgment dismissing
decision, (1) authorizing and allowing Masagana to Masagana's complaint and UCPBGen's counterclaims
consign/deposit with this Court the sum of thereto filed with the Regional Trial Court, Branch 58,
P225,753.95 (refused by UCPBGen) as full payment of Makati City, in Civil Case 92-2023, without costs.
the corresponding premiums for the replacement-
renewal policies; (2) declaring Masagana to have fully
complied with its obligation to pay the premium PHILIPPINE PHOENIX SURETY & INSURANCE,
thereby rendering the replacement-renewal policy INC. vs.WOODWORKS, INC.
effective and binding for the duration 22 May 1992 [G.R. No. L-22684 August 31, 1967]
until 22 May 1993; and, ordering UCPBGen to deliver
forthwith to Masagana the said replacement-renewal FACTS:
policies; (3) declaring two of the policies in force from
22 August 1991 up to 23 August 1992 and 9 August That on April 1, 1960, plaintiff issued to defendant Fire
1991 to 9 August 1992, respectively; and (4) ordering Policy No. 9652 for the amount of P300,000.00. The
UCPBGen to pay Masagana the sums of: (a) premiums of said policy amounted to P6, 051.95. The
P18,645,000.00 representing the latter's claim for defendant paid P3,000.00 on September 22, 1960 and
indemnity under three policies and/or its replacement- the plaintiff made several demands on defendant to
renewal policies; (b) 25% of the total amount due as pay the amount of P3,522.09.
and for attorney's fees; (c) P25,000.00 as necessary
litigation expenses; and, (d) the costs of suit. In due Appellee Philippine Phoenix Surety & Insurance Co.,
time, UCPBGen appealed to the Court of Appeals. On 7 Inc. commenced this action in the Municipal Court of
September 1998, the Court of Appeals promulgated its Manila to recover from appellant Woodworks, Inc. the
decision affirming that of the Regional Trial Court with sum of P3,522.09, representing the unpaid balance of
the modification that item 3 of the dispositive portion the premiums on a fire insurance policy issued by
was deleted, and the award of attorney's fees was appellee in favor of appellant for a term of one year
reduced to 10% of the total amount due. The Court of from April 1, 1960 to April 1, 1961. From an adverse
Appeals held that following previous practise, decision of said court, Woodworks, Inc. appealed to
Masagana was allowed a 60 to 90 day credit term for the Court of First Instance of Manila.
the renewal of its policies, and that the acceptance of
the late premium payment suggested an Appeal upon a question of law taken by Woodworks,
understanding that payment could be made later. Inc. from the judgment of the Court of First Instance
UCPBGen appealed. of Manila "ordering the defendant, Woodworks, Inc. to
pay to the plaintiff, Philippine Phoenix Surety &
Issue: Insurance, Inc., the sum of P3,522.09 with interest
thereon at the legal rate of 6% per annum from the
Whether the fire insurance policies issued by UCPBGen date of the filing of the complaint until fully paid, and
to the Masagana covering the period 22 May 1991 to costs of the suit." Hence, this petition.
22 May 1992, had expired on the latter date or had
been extended or renewed by an implied credit Issue:
arrangement though actual payment of premium was

EH 403 2010-2011 2011-2012


Page 50
INSURANCE LAW CASE DIGESTS
Whether or not the non-payment of premium does not installments. All payments were likewise accepted by
cancel the policy in a perfected contract of insurance private respondent.
Whether or not a partial payment of the premium
made the policy effective during the whole period of In January 1984, the policy was again renewed and
the policy private respondent issued to petitioner Insurance
Policy for the period March 1984 to March 1985. On
Ruling: this renewed policy, petitioner made two installment
payments, both accepted by private respondent.
The petition is lack of merit. Thereafter, petitioner refused to pay the balance of the
premium.
There is, consequently, no doubt at all that, as
between the insurer and the insured, there was not Consequently, AHAC filed an action to recover the
only a perfected contract of insurance but a partially unpaid balance of P314,103.05 for Insurance policy. In
performed one as far as the payment of the agreed its answer with counterclaim, Tuscany admitted the
premium was concerned. Thereafter the obligation of issuance of Insurance Policy. It explained that it
the insurer to pay the insured the amount for which discontinued the payment of premiums because the
the policy was issued in case the conditions therefor policy did not contain a credit clause in its favor and
had been complied with, arose and became binding the receipts for the installment payments covering the
upon it, while the obligation of the insured to pay the policy for 1984-85, as well as the two (2) previous
remainder of the total amount of the premium due policies, stated the following reservations: (2)
became demandable. Acceptance of this payment shall not waive any of the
company rights to deny liability on any claim under the
The court did not agree with appellant's theory that policy arising before such payments or after the
non-payment by it of the premium due, produced the expiration of the credit clause of the policy; and (3)
cancellation of the contract of insurance. Such theory Subject to no loss prior to premium payment. If there
would place exclusively in the hands of one of the be any loss such is not covered. Tuscany further
contracting parties the right to decide whether the claimed that the policy was never binding and valid,
contract should stand or not. Rather the correct view and no risk attached to the policy. It then pleaded a
would seem to be this: as the contract had become counterclaim for P152,000.00 for the premiums
perfected, the parties could demand from each other already paid for 1984-85, and in its answer with
the performance of whatever obligations they had amended counterclaim, sought the refund of
assumed. In the case of the insurer, it is obvious that P924,206.10 representing the premium payments for
it had the right to demand from the insured the 1982-85.
completion of the payment of the premium due or sue
for the rescission of the contract. As it chose to Issue:
demand specific performance of the insured's
obligation to pay the balance of the premium, the Whether payment by installment of the premiums due
latter's duty to pay is indeed indubitable. on an insurance policy invalidates the contract of
insurance.
The appealed decision being in accordance with law
and the evidence, the same is affirmed. Ruling:

No. The subject policies are valid even if the premiums


Digested by: Gestopa, Gevina were paid on installments. The records clearly show
Makati Tuscany Condominium Corp. vs. Court of that Tuscany and AHAC intended subject insurance
Appeals policies to be binding and effective notwithstanding the
staggered payment of the premiums. The initial
Facts: insurance contract entered into in 1982 was renewed
in 1983, then in 1984. In those 3 years, the insurer
Sometime in early 1982, private respondent American accepted all the installment payments. Such
Home Assurance Co. (AHAC), represented by American acceptance of payments speaks loudly of the insurer's
International Underwriters (Phils.), Inc., issued in favor intention to honor the policies it issued to Tuscany.
of petitioner Makati Tuscany Condominium Corporation Certainly, basic principles of equity and fairness would
(TUSCANY) Insurance Policy on the latter's building not allow the insurer to continue collecting and
and premises, for a period beginning March 1982 and accepting the premiums, although paid on
ending March 1983, with a total premium of installments, and later deny liability on the lame
P466,103.05. The premium was paid on installments, excuse that the premiums were not prepaid in full.
all of which were accepted by private respondent. Thus, while the import of Section 77 is that
prepayment of premiums is strictly required as a
In February 1983, private respondent issued to condition to the validity of the contract, the Court was
petitioner another Insurance Policy, which replaced not prepared to rule that the request to make
and renewed the previous policy, for a term covering 1 installment payments duly approved by the insurer,
March 1983 to 1 March 1984. The premium in the would prevent the entire contract of insurance from
amount of P466,103.05 was again paid on going into effect despite payment and acceptance of

EH 403 2010-2011 2011-2012


Page 51
INSURANCE LAW CASE DIGESTS
the initial premium or first installment. Section 78 of former the premium of P2,983.50 plus 12% interest
the Insurance Code in effect allows waiver by the from 10 March 1987 until full payment. Hence this
insurer of the condition of prepayment by making an petition for review
acknowledgment in the insurance policy of receipt of
premium as conclusive evidence of payment so far as Issue:
to make the policy binding despite the fact that
premium is actually unpaid. Section 77 merely WON a fire insurance policy is valid, binding and
precludes the parties from stipulating that the policy is enforceable upon mere partial payment of premium
valid even if premiums are not paid, but does not
expressly prohibit an agreement granting credit Ruling:
extension, and such an agreement is not contrary to
morals, good customs, public order or public policy. So The SC finds no merit in the petition; hence, it affirms
is an understanding to allow insured to pay premiums the Court of Appeals.
in installments not so proscribed. At the very least,
both parties should be deemed in estoppel to question Insurance is a contract whereby one undertakes for a
the arrangement they have voluntarily accepted. It consideration to indemnify another against loss,
appearing from the peculiar circumstances that the damage or liability arising from an unknown or
parties actually intended to make the three (3) contingent event. The consideration is the premium,
insurance contracts valid, effective and binding, which must be paid at the time and in the way and
Tuscany may not be allowed to renege on its obligation manner specified in the policy, and if not so paid, the
to pay the balance of the premium after the expiration policy will lapse and be forfeited by its own terms.
of the whole term of the third policy (AH-CPP-
9210651) in March 1985. Moreover, where the risk is The pertinent provisions in the Policy on premium read
entire and the contract is indivisible, the insured is not
entitled to a refund of the premiums paid if the insurer
was exposed to the risk insured for any period, THIS POLICY OF INSURANCE WITNESSETH, THAT only
however brief or momentary. after payment to the Company in accordance with
Policy Condition No. 2 of the total premiums by the
insured as stipulated above for the period
Digested by: Roxanne A. Huyo aforementioned for insuring against Loss or Damage
Tibay v. CA by Fire or Lightning as herein appears, the Property
herein described x x x
Facts: 2. This policy including any renewal
thereof and/or any endorsement thereon is not
On 22 January 1987 private respondent Fortune Life in force until the premium has been fully paid to
and General Insurance Co., Inc. (FORTUNE) issued Fire and duly receipted by the Company in the
Insurance Policy No. 136171 in favor of Violeta R. manner provided herein.
Tibay and/or Nicolas Roraldo on their two-storey Any supplementary agreement seeking to
residential building located at 5855 Zobel Street, amend this condition prepared by agent, broker or
Makati City, together with all their personal effects Company official, shall be deemed invalid and of no
therein. The insurance was for P600,000.00 covering effect.
the period from 23 January 1987 to 23 January 1988. xxx xxx
On 23 January 1987, of the total premium of xxx
P2,983.50, petitioner Violeta Tibay only paid P600.00
thus leaving a considerable balance unpaid. Except only in those specific cases where
corresponding rules and regulations which are or may
On 8 March 1987 the insured building was completely hereafter be in force provide for the payment of the
destroyed by fire. On 10 March 1987 Violeta Tibay paid stipulated premiums in periodic installments at fixed
the balance of the premium. On the same day, she percentage, it is hereby declared, agreed and
filed with FORTUNE a claim on the fire insurance warranted that this policy shall be deemed
policy. Her claim was accordingly referred to its effective, valid and binding upon the Company
adjuster, Goodwill Adjustment Services, Inc. (GASI). only when the premiums therefor have actually
Petitioner forthwith complied and signed a non-waiver been paid in full and duly acknowledged in a
agreement. In a letter dated 11 June 1987, FORTUNE receipt signed by any authorized official or
denied the claim of Violeta for violation of Policy representative/agent of the Company in such
Condition No. 2 and of Sec. 77 of the Insurance Code. manner as provided herein.
Efforts to settle the case before the Insurance
Commission proved futile. Clearly the Policy provides for payment of premium in
full. Accordingly, where the premium has only been
On 19 July 1990 the trial court ruled for petitioners partially paid and the balance paid only after the peril
and adjudged FORTUNE liable. On 24 March 1995 the insured against has occurred, the insurance contract
Court of Appeals reversed the court a quo by declaring did not take effect and the insured cannot collect at all
FORTUNE not to be liable to plaintiff-appellees therein on the policy. This is fully supported by Sec. 77 of the
but ordering defendant-appellant to return to the Insurance Code which provides

EH 403 2010-2011 2011-2012


Page 52
INSURANCE LAW CASE DIGESTS
SEC. 77. An insurer is entitled to payment of the conclusive evidence of the premium payment. The
premium as soon as the thing insured is exposed express mention of exceptions operates to exclude
to the peril insured against. Notwithstanding any other exceptions; conversely, those which are not
agreement to the contrary, no policy or contract of within the enumerated exceptions are deemed included
insurance issued by an insurance company is valid in the general rule. Thus, under Sec. 77, as well as
and binding unless and until the premium thereof Sec. 78, until the premium is paid, and the law has not
has been paid, except in the case of a life or an expressly excepted partial payments, there is no valid
industrial life policy whenever the grace period and binding contract. Hence, in the absence of clear
provision applies. waiver of prepayment in full by the insurer, the insured
cannot collect on the proceeds of the policy.
Apparently the crux of the controversy lies in the
phrase unless and until the premium thereof has been
paid. In Escosura v. San Miguel Brewery, Inc., the iii. Non-Default Options in Life Insurance
Court through Mr. Justice Jesus G. Barrera, iv. Reinstatement of a Lapsed Policy of Life
interpreting the phrase with pay used in connection Insurance
with leaves of absence with pay granted to employees, v. Acknowledgement in policy of receipt of
ruled - x x x the legislative practice seems to be that premium is conclusive (Section 78)
when the intention is to distinguish between full and
partial payment, the modifying term is used x x x.
Digested by: Patricia Ko
Furthermore, the two (2) cases, Phoenix and Tuscany, American Home Assurance vs. Chua
adequately demonstrate the waiver, either express or [G.R. No. 130421. June 28, 1999]
implied, of prepayment in full by the insurer: impliedly,
by suing for the balance of the premium as in Phoenix, FACTS:
and expressly, by agreeing to make premiums payable
in installments as in Tuscany. But contrary to the Petitioner, American Home, is a domestic corporation
stance taken by petitioners, there is no waiver express engaged in the insurance business. Sometime in
or implied in the case at bench. 1990, respondent Chua obtained from petitioner a fire
insurance covering the stock-in-trade of his business,
Conformably with the aforesaid stipulations explicitly Moonlight Enterprises, located at Valencia,
worded and taken in conjunction with Sec. 77 of the Bukidnon. The insurance was due to expire on 25
Insurance Code the payment of partial premium by the March 1990.
assured in this particular instance should not be
considered the payment required by the law and the On 5 April 1990 respondent issued PCIBank Check No.
stipulation of the parties. Rather, it must be taken in 352123 in the amount of P2,983.50 to petitioners
the concept of a deposit to be held in trust by the agent, James Uy, as payment for the renewal of the
insurer until such time that the full amount has been policy. In turn, the latter delivered Renewal Certificate
tendered and duly receipted for. In other words, as No. 00099047 to respondent. The check was drawn
expressly agreed upon in the contract, full payment against a Manila bank and deposited in petitioners
must be made before the risk occurs for the policy to bank account in Cagayan de Oro City. The
be considered effective and in force. corresponding official receipt was issued on 10
April. Subsequently, a new insurance policy, Policy No.
Thus, no vinculum juris whereby the insurer bound 206-4234498-7, was issued, whereby petitioner
itself to indemnify the assured according to law ever undertook to indemnify respondent for any damage or
resulted from the fractional payment of premium. The loss arising from fire up toP200,000 for the period 25
insurance contract itself expressly provided that the March 1990 to 25 March 1991.
policy would be effective only when the premium was
paid in full. It would have been altogether different On 6 April 1990 Moonlight Enterprises was completely
were it not so stipulated. Ergo, petitioners had razed by fire. Total loss was estimated
absolute freedom of choice whether or not to be between P4,000,000 and P5,000,000. Respondent
insured by FORTUNE under the terms of its policy and filed an insurance claim with petitioner and four other
they freely opted to adhere thereto. co-insurers, namely, Pioneer Insurance and Surety
Corporation, Prudential Guarantee and Assurance, Inc.,
Applying further the rules of statutory construction, the Filipino Merchants Insurance Co. and Domestic
case of South Sea Surety and Insurance Company, Insurance Company of the Philippines. Petitioner
Inc. v. Court of Appeals, speaks only of two (2) refused to honor the claim notwithstanding several
statutory exceptions to the requirement of payment of demands by respondent, thus, the latter filed an action
the entire premium as a prerequisite to the validity of against petitioner before the trial court.
the insurance contract. These exceptions are: (a) in
case the insurance coverage relates to life or industrial In its defense, petitioner claimed there was no existing
life (health) insurance when a grace period applies, insurance contract when the fire occurred since
and (b) when the insurer makes a written respondent did not pay the premium. It also alleged
acknowledgment of the receipt of premium, this that even assuming there was a contract, respondent
acknowledgment being declared by law to, be then violated several conditions of the policy,

EH 403 2010-2011 2011-2012


Page 53
INSURANCE LAW CASE DIGESTS
particularly: (1) his submission of fraudulent income and the Court of Appeals are entitled to great weight
tax return and financial statements; (2) his failure to and respect, and will not be disturbed on appeal in the
establish the actual loss, which petitioner assessed absence of any clear showing that the trial court
at P70,000; and (3) his failure to notify to petitioner of overlooked certain facts or circumstances which would
any insurance already effected to cover the insured substantially affect the disposition of the case. [7] We
goods. These violations, petitioner insisted, justified see no reason to depart from this ruling.
the denial of the claim.
According to the trial court the renewal certificate
Petitioner emphasizes that when the fire occurred on 6 issued to respondent contained the acknowledgment
April 1990 the insurance contract was not yet that premium had been paid. It is not disputed that
subsisting pursuant to Article 1249 [3] of the Civil Code, the check drawn by respondent in favor of petitioner
which recognizes that a check can only effect payment and delivered to its agent was honored when
once it has been cashed. Although respondent presented and petitioner forthwith issued its official
testified that he gave the check on 5 April to a certain receipt to respondent on 10 April 1990. Section 306 of
James Uy, the check, drawn against a Manila bank and the Insurance Code provides that any insurance
deposited in a Cagayan de Oro City bank, could not company which delivers a policy or contract of
have been cleared by 6 April, the date of the fire. In insurance to an insurance agent or insurance broker
fact, the official receipt issued for respondents check shall be deemed to have authorized such agent or
payment was dated 10 April 1990, four days after the broker to receive on its behalf payment of any
fire occurred. premium which is due on such policy or contract of
insurance at the time of its issuance or delivery or
Citing jurisprudence,[4] petitioner also contends that which becomes due thereon. [8] In the instant case, the
respondents non-disclosure of the other insurance best evidence of such authority is the fact that
contracts rendered the policy void. petitioner accepted the check and issued the official
receipt for the payment. It is, as well, bound by its
Respondent refutes the reason for petitioners denial of agents acknowledgment of receipt of payment.
his claim. To bolster his argument, respondent cites
Section 66 of the Insurance Code,[5] which requires the Section 78 of the Insurance Code explicitly provides:
insurer to give a notice to the insured of its intention An acknowledgment in a policy or contract of insurance
to terminate the policy forty-five days before the policy of the receipt of premium is conclusive evidence of its
period ends. In the instant case, petitioner opted not payment, so far as to make the policy binding,
to terminate the policy. Instead, it renewed the policy notwithstanding any stipulation therein that it shall not
by sending its agent to respondent, who was issued a be binding until the premium is actually paid.
renewal certificate upon delivery of his check payment
for the renewal of premium. At this precise moment This Section establishes a legal fiction of payment and
the contract of insurance was executed and already in should be interpreted as an exception to Section 77.
effect. Respondent also claims that it is standard
operating procedure in the provinces to pay insurance On whether there were violations of the policy
premiums by check when collected by insurance conditions stipulated
agents.
Is respondent guilty of the policy violations imputed
ISSUES: against him? We are not convinced by petitioners
arguments. Ordinarily, where the insurance policy
1. whether there was a valid payment of premium, specifies as a condition the disclosure of existing co-
considering that respondents check was cashed insurers, non-disclosure thereof is a violation that
after the occurrence of the fire; entitles the insurer to avoid the policy. This condition
2. whether respondent violated the policy by his is common in fire insurance policies and is known as
submission of fraudulent documents and non- the other insurance clause. The purpose for the
disclosure of the other existing insurance inclusion of this clause is to prevent an increase in the
contracts; moral hazard. We have ruled on its validity and the
case of Geagonia v. Court of Appeals[10] clearly
RULING: illustrates such principle. However, we see an
exception in the instant case.
On the payment of premium through check (Relevant
provision: Sec. 78) Citing Section 29[11] of the Insurance Code, the trial
The general rule in insurance laws is that unless the court reasoned that respondents failure to disclose
premium is paid the insurance policy is not valid and was not intentional and fraudulent. The application of
binding. The only exceptions are life and industrial life Section 29 is misplaced. Section 29 concerns
insurance.[6] Whether payment was indeed made is a concealment which is intentional. The relevant
question of fact which is best determined by the trial provision is Section 75, which provides that:
court. The trial court found, as affirmed by the Court A policy may declare that a violation of specified
of Appeals, that there was a valid check payment by provisions thereof shall avoid it, otherwise the breach
respondent to petitioner. Well-settled is the rule that of an immaterial provision does not avoid the policy.
the factual findings and conclusions of the trial court

EH 403 2010-2011 2011-2012


Page 54
INSURANCE LAW CASE DIGESTS
To constitute a violation the other existing insurance 1. The rationale of a group insurance policy of
contracts must be upon the same subject matter and mortgagors, otherwise known as the "mortgage
with the same interest and risk.[12] Indeed, respondent redemption insurance," is a device for the protection of
acquired several co-insurers and he failed to disclose both the mortgagee and the mortgagor. On the part of
this information to petitioner. Nonetheless, petitioner the mortgagee, it has to enter into such form of
is estopped from invoking this argument. The trial contract so that in the event of the unexpected demise
court cited the testimony of petitioners loss adjuster of the mortgagor during the subsistence of the
who admitted previous knowledge of the co-insurers. mortgage contract, the proceeds from such insurance
will be applied to the payment of the mortgage debt,
Indubitably, it cannot be said that petitioner was thereby relieving the heirs of the mortgagor from
deceived by respondent by the latters non-disclosure paying the obligation. In a similar vein, ample
of the other insurance contracts when petitioner protection is given to the mortgagor under such a
actually had prior knowledge thereof. Petitioners loss concept so that in the event of death; the mortgage
adjuster had known all along of the other existing obligation will be extinguished by the application of the
insurance contracts, yet, he did not use that as basis insurance proceeds to the mortgage indebtedness.
for his recommendation of denial. The loss adjuster, Consequently, where the mortgagor pays the
being an employee of petitioner, is deemed a insurance premium under the group insurance policy,
representative of the latter whose awareness of the making the loss payable to the mortgagee, the
other insurance contracts binds petitioner. We, insurance is on the mortgagor's interest, and the
therefore, hold that there was no violation of the mortgagor continues to be a party to the contract. In
other insurance clause by respondent. this type of policy insurance, the mortgagee is simply
an appointee of the insurance fund, such loss-payable
clause does not make the mortgagee a party to the
vi. Return of premium (Sections 79-82) contract.

GREAT PACIFIC LIFE ASSURANCE CORP. vs. The insured private respondent did not cede to the
COURT OF APPEALS AND MEDARDA V. LEUTERIO mortgagee all his rights or interests in the insurance,
the policy stating that: "In the event of the debtor's
FACTS death before his indebtedness with the Creditor [DBP]
shall have been fully paid, an amount to pay the
This is a petition for review under Rule 45 of the Rules outstanding indebtedness shall first be paid to the
of Court, assailing the decision and resolution of the creditor and the balance of sum assured, if there is
Court of Appeals dated May 17, 1994 and January 4, any, shall then be paid to the beneficiary/ies
1994, respectively, in CA G.R. CV No. 18341. The designated by the debtor." When DBP submitted the
appellate court affirmed in toto the judgment of the insurance claim against petitioner, the latter denied
Regional Trial Court of Misamis Oriental in an insurance payment thereof, interposing the defense of
claim filed by private respondent against Great Pacific concealment committed by the insured. Thereafter,
Life Assurance Co. DBP collected the debt from the mortgagor and took
the necessary action of foreclosure on the residential
The Supreme Court found the petition not meritorious. lot of private respondent. In Gonzales La O vs. Yek
Contrary to petitioner's allegations, there was no Tong Lin Fire & Marine Ins. Co. we held: "Insured,
sufficient proof that the insured had suffered from being the person with whom the contract was made, is
hypertension. Aside from the statement of the primarily the proper person to bring suit thereon. . . .
insured's widow who was not even sure if the Subject to some exceptions, insured may thus sue,
medicines taken by Dr. Leuterio were for hypertension, although the policy is taken wholly or in part for the
the petitioner had not proven nor produced any benefit of another person named or unnamed, and
witness who could attest to Dr. Leuterio's medical although it is expressly made payable to another as his
history. Clearly, it had failed to establish that there interest may appear or otherwise. . . . Although a
was concealment made by the insured, hence it cannot policy issued to a mortgagor is taken out for the
refuse payment of the claim. benefit of the mortgagee and is made payable to him,
yet the mortgagor may sue thereon in his own name,
ISSUES especially where the mortgagee's interest is less than
the full amount recoverable under the policy, . . . 'And
1. Whether the Court of Appeals erred in holding in volume 33, page 82, of the same work, we read the
petitioner liable to DBP as beneficiary in a group following: `Insured may be regarded as the real party
life insurance contract from a complaint filed by in interest, although he has assigned the policy for the
the widow of the decedent/mortgagor? NO purpose of collection, or has assigned as collateral
2. Whether Dr. Leuterio concealed that he had security any judgment he may obtain." And since a
hypertension, which would vitiate the insurance policy of insurance upon life or health may pass by
contract? NO transfer, will or succession to any person, whether he
has an insurable interest or not, and such person may
HELD recover it whatever the insured might have recovered,
the widow of the decedent Dr. Leuterio may file the
suit against the insurer, Grepalife.

EH 403 2010-2011 2011-2012


Page 55
INSURANCE LAW CASE DIGESTS
Issues:
2. The question of whether there was
concealment was aptly answered by the appellate W/N Whether or not the loss of the property was
court, thus: "The insured, Dr. Leuterio, had answered caused mainly by explosion and thus not covered by
in his insurance application that he was in good health the insurance policy.
and that he had not consulted a doctor for any of the
enumerated ailments, including hypertension; when he Ruling:
died the attending physician had certified in the death
certificate that the former died of cerebral The real cause of the fire is more or less a matter of
hemorrhage, probably secondary to hypertension. conjecture, upon which there is little, if any, evidence.
From this report, the appellant insurance company
refused to pay the insurance claim. Appellant alleged But the fact remains that there was a fire, and that the
that the insured had concealed the fact that he had plaintiffs property was destroyed. It is true that it may
hypertension. Contrary to appellant's allegations, there be that the explosion was the primary cause of the
was no sufficient proof that the insured had suffered fire, but that is only a matter of conjecture, and upon
from hypertension. Aside from the statement of the that point, the burden of proof was upon the
insured's widow who was not even sure if the defendant.
medicines taken by Dr. Leuterio were for hypertension,
the appellant had not proven nor produced any witness Defendant relies upon section 6 of the policy, as
who could attest to Dr. Leuterio's medical history . . . follows:
Appellant insurance company had failed to establish 6. Unless otherwise expressly stated in the policy
that there was concealment made by the insured, the insurance does not cover
hence, it cannot refuse payment of the claim." The
fraudulent intent on the part of the insured must be (h) Loss or damage occasioned by the
established to entitle the insurer to rescind the explosion; but loss or damage by explosion of gas
contract. Misrepresentation as a defense of the insurer for illuminating or domestic purposes in a building
to avoid liability is an affirmative defense and the duty in which gas is not generated and which does not
to establish such defense by satisfactory and form a part of any gas works, will be deemed to be
convincing evidence rests upon the insurer. In the case loss by fire within the meaning of this policy.
at bar, the petitioner failed to clearly and satisfactorily
establish its defense, and is therefore liable to pay the In answer to that, plaintiff relies upon section 5, which
proceeds of the insurance. is as follows:
5. The insurance does not cover

I. Loss (Sections 83-87) (d) Loss or damage occasioned directly or indirectly,


approximately or remotely by or through or in
Digested by: dlanor consequence of:
Paris-Manila Perfume Co. vs. Phoenix Assurance (1) Earthquake, hurricane, volcanic eruption or other
Co. convulsion of nature, and the company shall not be
liable for loss or damage arising during or within a
Facts: reasonable time after any of the said
occurrences, unless it be proved by the insured to
On May 22, 1924, defendant (Phoenix Assurance Co.) the satisfaction of the company that such loss or
issued to plaintiff (Paris-Manila Perfume Co.) its fire damage was not in any way occasioned by or through
insurance policy No. 841163 in the sum of P13,000 or in consequence of any of the said occurrences
upon the property of the plaintiff at No. 1 Calle
Cisneros, Cavite, insuring plaintiff's property against Examination of the above provisions would reveal that
fire for that amount. section 5 excludes not only the damages which may
immediately result from an earthquake, but also any
That on July 4, 1924, the property covered by the damage which may follow the earthquake, and that
insurance was completely destroyed by fire for the section 6 excludes only the damages which are
total loss to the plaintiff of P38.025.56; that under its the direct result of the explosion itself, and that
policy with the defendant, it promptly presented its it does not except damages which occurred from
claim but the defendant wrongfully and unjustly the fire occuring after the explosion, even though
refused to pay it. the explosion may have been the primary cause
of the fire.
Defendant contended among others that the policy of
insurance did not cover any loss or damage occasioned Moreover, there was no competent evidence as to
by explosion, and the loss was occasioned by an whether the explosion caused the fire or the fire
explosion, and was not covered by the policy. caused the explosion.
-sagabal-
The lower court rendered judgment in favor of the
plaintiff.
Digested by: Ichelle S. Malabuyoc

EH 403 2010-2011 2011-2012


Page 56
INSURANCE LAW CASE DIGESTS
THE EAST FURNITURE INC vs. THE GLOBE & time of the fire. This inventory contains 506 pieces of
RUTGERS FIRE INSURANCE CO. OF NEW YORK furniture and 3,700 board feet of lumber of the alleged
[G.R. No. L-35848 November 22, 1932] total value of P52,061.99. This amount was the total
loss claimed to have been suffered by the plaintiff,
Facts: although we note that in its complaints in these cases
amended it is conceded that some furniture of the
Plaintiff is a duly registered partnership engaged in the value of about P5,000 was saved.
sale of furniture; that the defendant is a company
engaged in the insurance business. Plaintiff insured Regardless of any difference of opinion as to the value
against fire the articles existing in its establishment. of the insured furniture and the extent of the damage
On March 2, 1929, a fire broke out in plaintiff's caused thereto by the fire in question, the fact that
establishment, as a result of which the insured articles the insured only had approximately 202 pieces of
therein found were destroyed by the fire. furniture in the building at the time of the fire and
sought to compel the insurance companies to pay for
Defendants defenses are:(1) that the fire in question 506 pieces conclusively shows that its claim was not
was of intentional origin; (2) that the claims of loss honestly conceived. The trial court's conclusion that
presented by the plaintiff were false and fraudulent; said claim is notoriously fraudulent, is correct.
(3) that the furniture in question had been mortgaged
by the plaintiff to the Manila Finance and Discount Condition 12 of each of the insurance policies sued
Corporation, so that at the time of the fire the plaintiff upon provides that "if the claim be in any respect
was not the only party interested therein, contrary to fraudulent, or if any false declaration be made or used
the representations made in its claims of loss; and (4) in support thereof, or if any fraudulent means or
that the plaintiff violated one of the conditions of the devices are used by the Insured or anyone acting on
policies by refusing to furnish the defendants with a his behalf to obtain any benefit under this policy; or, if
physical inventory of the contents of its store at the the loss or damage be occasioned by the wilful act, or
time of the fire. with the connivance of the Insured, all benefit under
this policy shall be forfeited."
By agreement of the parties the three cases were tried
jointly who after the trial found that the claims
presented by the plaintiff were notoriously fraudulent, J. Notice and Proof of Loss (Sections 88-92)
and, accordingly, sustained defendant's second special
defense and dismissed the complaint in each of the Sharruf & Co. vs Baloise Fire Ins. Co.,
three cases. Hence, this petition. [GR No. 44119, March 30, 1937]

Issue: Facts:

W/N fire was of intentional origin Salomon Sharruf and Eskenazi were doing business
W/N the claim of loss were fraudulent under the firm name of Sharuff & Co. They applied for
insurance the merchandise they had in stock. The
Ruling: insurance companies (Baloise Fire Ins. Co., Sun
Insurance Office, Springfield Insurance Co.) sent their
Fire of Intentional Origin representative to asses and examine the goods. As a
We are thus led to the conclusion that defendants' first result, the insurance companies issued insurance
special defense is well founded that the fire in policies with the total amount of 40,000.00. They then
question was of intentional origin and was caused with entered into a partnership, substituting the name
the connivance of the plaintiff. Neither the interest of Sharruf & Co. with the Sharruf & Eskanzi. About 12.41
the justice nor public policy would be promoted by an oclock on the morning of September 22, 1933, a fire
omission of the courts to expose and condemn broke out, burning and damaging the merchandise
incendiarism once the same is established by insured by Sharruf and Eskanzi.
competent evidence. It would tend to encourage rather
than suppress that great public menace if the courts Issue:
do not expose the crime to public condemnation when
the evidence in a case like the present shows that it Whether or not the claim of loss filed by the plaintiffs is
has really been committed. fraudulent.

Fraudulent claim of loss Ruling:


We may also consider the damage caused by the fire in
relation with defendant's second special defense that With respect to the question whether or not the claim
plaintiff's claims of loss were false and fraudulent. of loss filed by the plaintiffs is fraudulent, it is alleged
by them that the total value of the textiles contained in
To each of the proofs of loss which the plaintiff cases deposited inside the building when the
presented to the respective insurance companies four partnership Sharruf & Eskenazi was formed was
days after the fire was attached an inventory of the P12,000; that of the fancy jewelry with imitation
furniture claimed to have been in the building at the stones from P15,000 to P17,000, and that of the

EH 403 2010-2011 2011-2012


Page 57
INSURANCE LAW CASE DIGESTS
kitchen utensils and tableware made of aluminum, The plaintiffs, upon whom devolve the legal obligation
bronze and glass P10,676. If, as said plaintiffs claim, to prove the existence, at the time of the fire, of the
they had already sold articles, mostly textiles, valued articles and merchandise for the destruction of which
at P8,000, a small quantity of cloth must have been they claim indemnity from the defendant companies,
left at the time the fire occured. In their claim, have not complied with their duty because they have
however, the textiles allegedly consumed by fire and failed to prove by a preponderance of evidence that
damaged by water are assessed by them at P12,000. when the fire took place there where in the burnt
The claim of P12,000 is certainly not attributable to a building articles and merchandise in the total amount
mere mistake in estimate and counting because if they of the insurance policies or that the textiles and other
had textiles worth only P12,000 before the fire and damaged and undamaged goods found in the building
they sold goods, mostly textiles, worth P8,000, surely after the fire were worth P40,000. On the contrary,
textiles in the same amount of P12,000 could not have their own witness, Robles, testified that up to the
been burned and damaged after the fire. Of the month of September, 1933, there were about 39 or 40
kitchen utensils and tableware made of aluminum, cases belonging to the plaintiffs in his garage on
bronze and glass, of which, according to the evidence Mabini Street, indicating thereby that the cases of
for the plaintiffs, they had a stock valued at P10,676, merchandise examined by the agent of the insurance
there were found after the fire articles worth only companies on July 25 and August 15, 1933, and for
P1,248.80. Therefore, utensils valued at P9,427.20 which the insurance policies were issued, were taken
were lacking. A considerable amount of kitchen from the burned building where they were found. So
utensils made of noninflammable and fire-proof great is the difference between the amount of articles
material could not, by the very nature of things have insured, which the plaintiffs claim to have been in the
been totally consumed by the fire. At most, said building before the fire, and the amount thereof shown
articles would have been damaged, as the rest, and by the vestige of the fire to have been therein, that the
would have left traces of their existence. The same most liberal human judgment can not attribute such
may be said of the fancy jewels with imitation stones, difference to a mere innocent error in estimate or
and others of which the fancy jewels with imitation counting but to a deliberate intent to demand of the
stones, and others of which the plaintiffs claim to have insurance companies payment of an indemnity for
had a stock worth from P15,000 to P17,000 at the time goods not existing at the time of the fire, thereby
of the fire, of which only a few valued at P3,471.16, constituting the so-called "fraudulent claim" which, by
were left after the fire. According to said plaintiffs, all express agreement between the insurers and the
the articles, for the alleged loss of which indemnity is insured, is a ground for exemption of the insurers from
sought, were contained in about 40 showcases and civil liability.
wardrobes. According to the testimony of the fire
station chiefs, corrobarated by the photographs of Therefore, as the herein plaintiffs-appellees have acted
record, the flames caused more damage in the upper in bad faith in presenting a fraudulent claim, they are
part of the rooms than in the lower part thereof; since, not entitled to the indemnity claimed by them by virtue
of the ten or eleven cases found inside the building of the insurance policies issued by the defendant-
after the fire, only a few were partially burned and appellant companies in their favor.
others scorched judging from their appearance, the
goods were damaged more by water than by fire.
According to the inventory made by White & Page, Author: Aldrin Montesco
adjusters of the insurance companies, in the presence Malayan Ins. Co. Vs. Cruz Arnaldo
of the plaintiffs themselves and according to data [154 SCRA 672]
supplied by the latter, the total value thereof, aside,
from the articles not included in the inventories FACTS:
Exhibits B, C, and C-1, assessed at P744.50, amounts
to only P8,077.35. If the plaintiffs' claim that at time of On June 7, 1981, the petitioner (hereinafter called
the fire there were about 40 cases inside the burnt (MICO) issued to the private respondent, Coronacion
building were true, a ten or eleven of them were found Pinca, Fire Insurance Policy No. F-001-17212 on her
after the fire, traces of the thirty or twenty-nine cases property for the amount of P14,000.00 effective July
allegedly burnt would be found, since experience has 22, 1981, until July 22, 1982
shown that during the burning of a building all the On October 15,1981, MICO allegedly cancelled the
cases deposited therein are not so reduced to ashes policy for non-payment, of the premium and sent the
that the least vestige thereof cannot be found. In the corresponding notice to Pinca.
case of Go Lu vs. Yorkshire Insurance Co. (43 Phil.,
633), this court laid down the following doctrine: On December 24, 1981, payment of the premium for
This court will legally presume that in an ordinary Pinca was received by Domingo Adora, agent of MICO.
fire fifty bales or boxes of bolt goods of cloth
cannot be wholly consumed or totally destroyed, On January 15, 1982, Adora remitted this payment to
and that in the very nature of things some trace or MICO,together with other payments.
evidence will be left remaining of their loss or
destruction. On January 18, 1982, Pinca's property was completely
burned.

EH 403 2010-2011 2011-2012


Page 58
INSURANCE LAW CASE DIGESTS
On February 5, 1982, Pinca's payment was returned by after the effective date of the policy, of one or more of
MICO to Adora on the ground that her policy had been the following:
cancelled earlier. But Adora refused to accept it.
(a) non-payment of premium;
In due time, Pinca made the requisite demands for
payment, which MICO rejected. She then went to the (b) conviction of a crime arising out of acts increasing
Insurance Commission. It is because she was the hazard insured against;
ultimately sustained by the public respondent that the
petitioner has come to us for relief. (c) discovery of fraud or material misrepresentation;

ISSUE: (d) discovery of willful, or reckless acts or commissions


increasing the hazard insured against;
Whether or not the cancellation of the policy by the
insurance company was valid (e) physical changes in the property insured which
result in the property becoming uninsurable; or
RULING:
(f) a determination by the Commissioner that the
The Supreme Court ruled in favor of the insured. continuation of the policy would violate or would place
the insurer in violation of this Code.
The above provision (Sec. 77 as invoked by the
insurance company) is not applicable because payment As for the method of cancellation, Section 65
of the premium was in fact eventually made in this provides as follows:
case. Notably, the premium invoice issued to Pinca at
the time of the delivery of the policy on June 7, 1981 SEC. 65. All notices of cancellation mentioned in the
was stamped "Payment Received" of the amount of preceding section shall be in writing, mailed or
P930.60 on "12-24-81" by Domingo Adora. This is delivered to the named insured at the address shown
important because it suggests an understanding in the policy, and shall state (a) which of the grounds
between MICO and the insured that such payment set forth in section sixty-four is relied upon and (b)
could be made later, as agent Adora had assured that, upon written request of the named insured, the
Pinca. In any event, it is not denied that this payment insurer will furnish the facts on which the cancellation
was actually made by Pinca to Adora, who remitted the is based. A valid cancellation must, therefore, require
same to MICO. concurrence of the following conditions:

It is not disputed that the premium was actually paid (1) There must be prior notice of cancellation to the
by Pinca to Adora on December 24, 1981, who insured;
received it on behalf of MICO, to which it was remitted
on January 15, 1982. What is questioned is the validity (2) The notice must be based on the occurrence, after
of Pinca's payment and of Adora's authority to receive the effective date of the policy, of one or more of the
it. grounds mentioned;

MICO's acknowledgment of Adora as its agent defeats (3) The notice must be (a) in writing, (b) mailed, or
its contention that he was not authorized to receive delivered to the named insured, (c) at the address
the premium payment on its behalf. It is clearly shown in the policy;
provided in Section 306 of the Insurance Code. And it (4) It must state (a) which of the grounds mentioned
is a well-known principle under the law of agency that: in Section 64 is relied upon and (b) that upon written
request of the insured, the insurer will furnish the facts
Payment to an agent having authority to receive or on which the cancellation is based.
collect payment is equivalent to payment to the
principal himself; such payment is complete when the Thus, it behooved MICO's to make sure that the
money delivered is into the agent's hands and is a cancellation was actually sent to and received by the
discharge of the indebtedness owing to the principal. insured. No such proof in support thereto was
presented by the insurance company.
The policy could be cancelled on any of the
supervening grounds enumerated in Article 64 As it has not been shown that there was a valid
(except "non-payment of premium") provided cancellation of the policy, there was consequently no
the cancellation was made in accordance need to renew it but to pay the premium thereon.
therewith and with Article 65. Payment was thus legally made on
the original transaction and it could be, and was,
Section 64 reads as follows: validly received on behalf of the insurer by its agent
Adora. Adora, incidentally, had not been informed of
SEC. 64. No policy of insurance other than life shall be the cancellation either and saw no reason not to accept
cancelled by the insurer except upon prior notice the said payment.
thereof to the insured, and no notice of cancellation
shall be effective unless it is based on the occurrence,

EH 403 2010-2011 2011-2012


Page 59
INSURANCE LAW CASE DIGESTS
Lastly, Loss and its amount may be determined on the
basis of such proof as may be offered by the insured, Issue:
which need not be of such persuasiveness as is
required in judicial proceedings. 25 If, as in this case, Whether or not Philamlife is exempt from paying the
the insured files notice and preliminary proof of loss margin fee by virtue of the pre-existing obligation
and the insurer fails to specify to the former all the made by the reinsurance treaty.
defects thereof and without unnecessary delay, all
objections to notice and proof of loss are deemed Ruling:
waived under Section 90 of the Insurance Code.
There should not be any misapprehension as to the
distinction between a reinsurance treaty, on the one
K. Guidelines on Claims Settlement hand, and a reinsurance policy or a reinsurance
a. Unfair Claims Settlement; Sanctions cession, on the other. The concept of one and the
b. Prescription of Action other is well expressed thus:
c. Subrogation . . . A reinsurance policy is thus a contract
L. Double Insurance (Sections 93-94) of indemnity one insurer makes with another to
i. Double Insurance (Section 93) protect the first insurer from a risk it has already
ii. Over-Insurance (Section 94) assumed. . . . In contradistinction a reinsurance
M. Reinsurance (Sections 95-98) treaty is merely an agreement between two
insurance companies whereby one agrees
Digested by: Archie Necesario to cede and the other to accept reinsurance
Phil American Life Insurance vs Auditor General business pursuant to provisions specified in the
treaty. The practice of issuing policies by insurance
Facts: companies includes, among other things, the
issuance of reinsurance policies on standard risks
On January 1, 1950, Philippine American Life and also on substandard risks under special
Insurance Company (Philamlife), a domestic life arrangements. The lumping of the different
insurance corporation, and American International agreements under a contract has resulted in the
Reinsurance Company (AIRCO) a corporation term known to the insurance world as "treaties."
organized under the laws of the Republic of Panama, Such a treaty is, in fact, an agreement between
entered into a reinsurance treaty. On their agreement, insurance companies to cover the different
Philamlife agrees to reinsure with AIRCO the entire situations described. Reinsurance treaties and
first excess of such life insurance on the lives of reinsurance policies are not synonymous. Treaties
persons as may be written by the Philamlife under are contracts for insurance; reinsurance policies or
direct application over and above its maximum limit of cessions . . . are contracts of insurance.
retention for life insurance, and AIRCO binds itself to
accept such reinsurances on the same terms. It is also Nothing in that treaty, however, obligates Philamlife to
stipulated that when new policies are applied for and remit to AIRCO a fixed, certain, and obligatory sum by
issued Philamlife can cede automatically any amount, way of reinsurance premiums. All that the reinsurance
within the limits. treaty provides on this point is that Philamlife "agrees
to reinsure." The treaty speaks of a probability; not a
On July 16, 1959, the Philippine government enacted reality. For, without reinsurance, no premium is due.
the Margin Law which provides for the payment of Of course, the reinsurance treaty lays down the duty to
margin fee for all sales of foreign exchange. Pursuant remit premiums if any reinsurance is effected upon
to the law, the Central Bank of the Philippines collected the covenants in that treaty written. So it is that the
the sum of P268,747.48 as foreign exchange margin reinsurance treaty per se cannot give rise to a
on Philamlife remittances to Airco. contractual obligation calling for the payment of
foreign exchange "issued, approved and outstanding
Philamlife subsequently filed with the Central Bank a as of the date this Act [Republic Act 2609] takes
claim for the refund of the above sum of P268,747.48. effect."
The ground therefor was that the reinsurance
premiums so remitted were paid pursuant to the For an exemption to come into play, there must be a
reinsurance treaty, and, therefore, were pre-existing reinsurance policy or, as in the reinsurance treaty
obligations expressly exempt by the Margin Law from provided, a "reinsurance cession" which may be
the margin fee. automatic or facultative.

Auditor General ruled that "[r]emittance of premia on Philamlife's obligation to remit reinsurance premiums
insurance policies issued or renewed on or after July becomes fixed and definite upon the execution of the
16, 1959, or even if issued or renewed before the said reinsurance cession. Because, for every life insurance
date, but their reinsurance was effected, only policy ceded to Airco, Philamlife agrees to pay
thereafter, are not exempt from the margin fee, even premium. 12It is only after a reinsurance cession is
if the reinsurance treaty under which they are made that payment of reinsurance premium may be
reinsured was approved by the Central Bank before exacted, as it is only after Philamlife seeks to remit
July 16, 1959.

EH 403 2010-2011 2011-2012


Page 60
INSURANCE LAW CASE DIGESTS
that reinsurance premium that the obligation to pay favor of the Government Service Insurance System,
the margin fee arises. became a liability when the insured property was
burned on February 16, 1962. Since the policy was
Upon the premise that the margin fee of P268,747.48 issued on July 1, 1961, it was supposed to expire on
was collected on remittances made on reinsurance July 1, 1962. 2 The next day, February 17, ASIAN
effected on or after the Margin Law took effect, refund immediately notified FIELDMEN'S of said fire loss. And
thereof does not come within the coverage of the on February 26, 1962 ASIAN sent its reply stating,
exemption circumscribed in Section 3 of the said law. among other things, as follows:
... we beg to reiterate that your letter of December
The petition for review is hereby denied, and the ruling 7, 1961, terminating said treaties by December 31,
of the Auditor General denying refund is hereby 1961, is not in accordance with the terms thereof,
affirmed. since there was no prior three months' notice.
However, considering the attitude express (sic) in
your aforesaid letter of December 7, 1961, we are
Digested by: Sharmine M. Odchigue willing to waive provision that said treaties may be
Fieldmens Ins. Co. v. Asian Surety and Ins. Co. cancelled on December 31st of any year, and will
[34 SCRA 36] consider them cancelled at the end of three (3)
months from December 7, 1961, by which time we
Facts: shall be able to render the final accounting you
desire.
Between April 11, 1960 and January 9, 1961 the
Asian Surety & Insurance Company, Inc. (ceding FIELDMEN'S, filed a petition for declaratory relief with
company) and the Fieldmen's insurance Company, Inc. the Court of First Instance of Manila to seek a
(reinsuring company) entered into seven (7) declaration that all the reinsurance contracts entered
reinsurance agreements or treaties 1 under the general into between them had terminated as of December 31,
terms of which the former, undertook to cede to the 1961 and to obtain an order directing ASIAN to render
latter, a specified portion of the amount of insurance final accounting of the transactions between them with
underwritten by ASIAN upon payment to FIELDMEN'S respect to said reinsurance treaties as of the cut-off
of a proportionate share of the gross rate of the date.
premium applicable with respect to each cession after
deducting a commission. Said agreements or treaties Trial Court declared six 3 of the seven 4 insurance
were to, take effect from certain specific dates and agreements in question cancelled as of December 31,
were to be in force until cancelled by either party 1961 and upheld ASIAN'S position that all cessions of
upon previous notice of at least three (3) months reinsurance made by it to FIELDMEN'S prior to the
by registered mail to the other party, the cancellation cancellation of the reinsurance treaties continued in full
to take effect as of the 31st of December of the year in force and effect until expiry dates
which the notice was given.
Court of Appeals affirmed decision of RTC
On September 19, 1961( 1 st letter) FIELDMEN'S, by
means of registered mail, served notice to ASIAN to be Issue:
relieved from all participation in its various treaties
with the latter effective December 31, 1961. This Whether or not said cancellation had the effect of
communication, although admittedly received by terminating also the liability of FIELDMEN'S as
ASIAN on September 25, 1961, did not elicit any reply reinsurer with respect to policies or cessions issued
from ASIAN. prior to the termination of the principal reinsurance
contracts or treaties.
On December 7, 1961 (2nd letter) FIELDMEN'S sent
another letter to ASIAN reiterated its position that it Ruling: NO.
would consider itself "no longer at risk for any
reinsurance and/or cession" given by ASIAN which Of the six reinsurance contracts under consideration
might be in force on December 31, 1961. two contain provisions, which clearly and expressly
recognize the continuing effectivity of policies ceded
Not having received any formal reply from ASIAN, under them for reinsurance notwithstanding the
FIELDMEN'S sent anew a letter on February 17, 1962 cancellation of the contracts themselves.
(3rd letter) reminding ASIAN of the December 7 letter
regarding the cancellation of all the reinsurance Insofar as the two reinsurance agreements with the
treaties and cessions as of December 31, 1961. At the express stipulations aforequoted are concerned there
same time FIELDMEN'S requested ASIAN to submit its is clearly no merit in FIELDMEN'S claim that their
final accounting of all cessions made to the former for cancellation carried with it ipso facto the
the preceding months when the reinsurance termination of all reinsurance cessions
agreements were in force. thereunder. Such cessions continued to be in force
until their respective dates of expiration. Since it was
Meanwhile one of the risks reinsured with FIELDMENS under one of said agreements, namely, the Facultative
under Cession No. 61-87, Policy No. RI-1236, issued in Obligatory Reinsurance Treaty-Fire, that the

EH 403 2010-2011 2011-2012


Page 61
INSURANCE LAW CASE DIGESTS
reinsurance cession corresponding to the GSIS policy parties within thirty (30) days after having been
had been made, FIELDMEN'S cannot avoid liability required so to do by the other party in writing, and
which arose by reason of the burning of the insured in case of disagreement between the arbitrators, to
property. the decision of the umpire to be appointed by them
in writing before entering on the reference. Each
With respect to the other four agreements, it would party shall submit its case with all particulars
seem that the petition for declaratory relief is moot, within thirty days after their appointment. The seat
and that no useful purpose would be served by of arbitration shall be in Manila, Philippines, and
defining the respective rights and obligations of the the expenses of arbitration shall be borne in equal
parties thereunder. The said agreements have been proportion by the parties. The decision of the
cancelled, and it does not appear that any claim by or arbitrators or umpire, as the case may be, shall be
liability in favor of the insured has actually arisen final and binding on both the Company and the
under any of the reinsurance cessions made prior to Reinsurer. The arbitrators and umpire shall not be
such cancellation. bound by the strict rules of evidence and by
judicial formalities in making the award.
WHEREFORE, the decision appealed from is affirmed
insofar as it refers to the Facultative-Obligatory, The court rendered its decision in favor of plaintiff,
Reinsurance Treaty and the Personal Accident hence this appeal.
Reinsurance Treaty are concerned, and modified with
respect to the others by declaring the issues Issue:
concerning them as moot and academic
1. WON the trial court erred in failing to rule that
plaintiff-appellee has no causes of action against it,
Digested by: Kristine Oja the matter not having been referred to the decision
Equitable Ins. & Casualty Co. v. Rural Insurance of two arbitrators or umpire, which, it is claimed, is
the condition precedent agreed upon in Article VIII
Facts: of the Reinsurance Agreement
2. WON the trial court erred in failing to rule that in a
On November 11, 1957, plaintiff(Equitable) and facultative obligation the right to choose an
defendant (Rural Insurance) entered into a reciprocal alternative remedy lies only with the debtor, who
facultative reinsurance agreement. Pursuant to said in this case is the herein defendant-appellant", and
agreement, plaintiff reinsured for P2,000.00 with in support thereof, cites Article 1206 of the new
defendant the stock covered by fire insurance Policy Civil Code.
No. 5880 and also for P2,000.00 the stock covered by
fire insurance Policy No. 6026. Held:

Stocks covered by Insurance Policy Nos. 5880 and As to the first issue, the court finds no merit in this
6026 were subsequently burned and the share of the contention. Under the abovequoted provision of the
loss assumed by defendant as per reinsurance Reinsurance Agreement, it would seem clear that the
agreement was computed at P2,024.87 and P1,334.80 requirement of submitting for decision to two
respectively. arbitrators or an umpire the matter of losses by fire or
the liability of the parties thereto arises only if and
Notwithstanding repeated demands, defendant refused when the same is disputed by one of the parties. It
and failed to pay plaintiff, and that for defendant's does not appear in the instant case that appellant did
failure to pay its share of the losses assumed by it, dispute appellee's claims. Consequently, appellant may
plaintiff has been compelled to institute an action in not invoke said provision in avoidance of its liability to
court. appellee.

The defendant moved for the dismissal of the case It is true that paragraph (Article VIII) of said
contending that the complaint states no cause of Reciprocal Facultative Reinsurance Agreement required
action, the matter not having been referred to the that 'in the event of any question arising as to the
decision of two arbitrators or umpire, which, it is meaning of, or any way connected with or relating to
claimed, is the condition precedent agreed upon in this Agreement, whether before or after its
Article VIII of the Reinsurance Agreement entered into termination, the parties shall endeavor to arrive at a
between the parties, to wit: . satisfactory compromise by amicable settlement rather
ARTICLE VIII than by court action'; and that the dispute should be
In the event of any question arising as to the referred to the decision of two arbitrators and umpire,
meaning of, or any way connected with or relating as provided, therein. However, in this particular case,
to this Agreement, whether before or after its there is absolutely no dispute between the two parties,
termination, the parties shall endeavor to arrive at because in the stipulation of facts, the defendant has
a satisfactory compromise by amicable settlement admitted that plaintiff has paid its liability to the
rather than by court action. The dispute shall be insured as per its fire insurance policies specified in the
referred to the decision of two arbitrators, of whom two causes of action of the complaint. Defendant has,
one shall be appointed in writing by each of the likewise, admitted its liability as reinsurer under the

EH 403 2010-2011 2011-2012


Page 62
INSURANCE LAW CASE DIGESTS
Reciprocal Facultative Reinsurance Agreement (Annex petitioner Ivor Robert Dayton Gibson claims to be
"A" to the complaint) to pay to the plaintiff its himself.
proportional shares, the amounts of which are not
disputed. Indeed, according to the complaint as Petitioner Ivor Robert Dayton Gibson filed a motion to
admitted by the defendant, statements of account as intervene as defendant, which motion was denied by
to the amounts of its share as reinsurer and, for all the lower court.
that appears, said defendant has never questioned the
correctness of said amounts. It is, likewise, admitted Issue:
by the defendant in the stipulation of facts, that
because of its failure to pay said amounts, the plaintiff, whether the lower court committed, reversible error in
on April 11, 1959, complained to the Assistant refusing the intervention of petitioner Ivor Robert
Insurance Commissioner, for official intervention, but Dayton Gibson in the suit between Lepanto and
said defendant has continued to ignore plaintiff's Malayan
demands for reimbursement under the reinsurance
policies. Ruling:

As to the second issue, the court finds no connection We rule that the respondent Judge committed no error
whatsoever between Article 1206 of the Civil Code and of law in denying petitioner's Motion to Intervene. And
the agreement subject of this action, except the word neither has he abused his discretion in his denial of
"facultative" used in both. The term "facultative" is petitioner's Motion for Intervention.
used in reinsurance contracts, and it is so used in this
particular case, merely to define the right of the We agree with the holding of the respondent Court
reinsurer to accept or not to accept participation in the that since movant Ivor Robert Dayton Gibson appears
risk insured. But once the share is accepted, as it was to be only one of several re-insurers of the risks and
in the case at bar, the obligation is absolute and the liabilities assumed by Malayan Insurance Company,
liability assumed thereunder can be discharged by one Inc., it is highly probable that other re-insurers may
and only way payment of the share of the losses. likewise intervene. If petitioner is allowed to intervene,
There is no alternative nor substitute prestation. We hold that there is good and sufficient basis for the
Court a quo to declare that the trial between Lepanto
Judgment appealed from was affirmed, with costs and Malayan would be definitely disrupted and would
against the defendant-appellant. certainly unduly delay the proceedings between the
parties especially at the stage where Lepanto had
already rested its case and that the issues would also
Digested by: Gayle Opsima be compounded as more parties and more matters will
IVOR ROBERT DAYTON GIBSON, petitioner, vs. have to be litigated. In other words, the Court's
HON. PEDRO A. REVILLA, in his official capacity discretion is justified and reasonable.
as Presiding Judge of Branch XII, Court of First
Instance of Rizal, and LEPANTO CONSOLIDATED We also hold that respondent Judge committed no
MINING COMPANY, respondents reversible error in further sustaining the fourth ground
[G.R. No. L-41432 1979 Jul 30] of Lepanto's Opposition to the Motion to Intervene that
the rights, if any, of petitioner are not prejudiced by
Facts: the present suit and will be fully protected in a
separate action against him and his co-insurers by
Lepanto Consolidated Mining Company filed a Malayan.
complaint against Malayan Insurance Company,
Inc.The civil suit thus instituted by Lepanto against Petitioner's contention that he has to pay once
Malayan was founded on the fact that Malayan issued a Malayan is finally adjudged to pay Lepanto because of
Marine Open Policy covering all shipments of copper, the very nature of a contract of reinsurance and
gold and silver concentrates in bulk from Poro, San considering that the re-insurer is obliged 'to pay as
Fernando, La Union to Tacoma, Washington or to other may be paid thereon' (referring to the original
places in the United States. policies), although this is subject to other stipulations
and conditions of the reinsurance contract, is without
Thereafter, Malayan obtained reinsurance abroad merit. The general rule in the law of reinsurance is that
through Sedgwick, Collins & Co., Limited, a London the re-insurer is entitled to avail itself of every defense
insurance brokerage. The Memorandum of Insurance which the re-insured (which is Malayan) might urge in
issued by Sedgwick to Malayan listed three groups of an action by the person originally insured (which is
underwriters or reinsurers Lloyds 62.808%, Lepanto). Specifically, the rule is stated thus
Companies (I.L.U.) 34.705%, Other Companies
2.487%. "Sec. 1238. In an action on a contract of reinsurance,
as a general rule the reinsurer is entitled to avail itself
At the top of the list of underwriting members of of every defense which the reinsured might urge in an
Lloyds is Syndicate No. 448, assuming 2.48% of the action by the person originally insured; . . ."
risk assumed by the reinsurer, which syndicate number

EH 403 2010-2011 2011-2012


Page 63
INSURANCE LAW CASE DIGESTS
The same rule is stated otherwise in 44 Am. Jur. 2d, made within 3years from july 10, 1969 per
Sec. 1862, p. 793, as follows: collateral agreement of the parties.
- Wellington in its brief raises the issue that Artex
"Moreover, where an action is brought against the deemed to have agreed to look SOLELY to the
reinsurer by the reinsured, the former may assert any reinsurers for indemnity in case of loss since their
defense that the latter might have made in an action paid up capital stock is only P500,000 and that
on the policy of original insurance." (Eagle Ins. Co. vs. they have to secure such reinsurance coverage the
Lafayette Ins. Co., 91 Ind. 443) over P24M fire insurance coverage of the policy
issued by Wellington to Artex.
As to the effect of the clause "to pay as may be paid
thereon" contained in petitioner's re-insurance Issue:
contract, Arnould, on the Law of Marine Insurance and
Average, 13th Ed., Vol. 1, Section 327, p. 315, states WON reinsurance contract of the parties makes the
the rule, thus: insured to look SOLELY to the reinsurers for indemnity
in case of loss
"It has been decided that this clause does not preclude
the reinsurer from insisting upon proper proof that a Ruling:
loss strictly within the terms of the original policy has
taken place." NO, the insured who is not directly a party or privy to
the reinsurance contract between Wellington and
"This clause does not enable the original underwriter to Alexander and Alexander Inc., cannot demand
recover from his re-insurer to an extent beyond the enforcement of such insurance contracts.
subscription of the latter." The Contracts take effect only between the parties,
their assigns and heirs as provide by Art 1311 of our
WHEREFORE, IN VIEW OF THE FOREGOING, the civil code. Further it provides that a contract with
petition is hereby dismissed. No costs. stipulations pour autrui or in favor of a third person not
a party to the contract, the parties must have CLEARLY
and DELIBERATELY conferred favor upon a third
Digested by: Jr Abul person.
Artex Development Co VS Wellington Insurance
- The SC also stated that assuming that Artex
FACTS: directly sue the reinsurers for payment this does
- Wellington insurance insured for P24,346,509 the not in any way affect or cancel out Wellingtons
building stocks and machinery of plaintiff Artex direct contractual liability to Artex.
against loss or damage by fire or lightning upon
august 2, 1963 with an additional sum of The SC dispose the case by affirming the prayer of
P833,034. Artex.
- Another insurance against business interruption
(use and occupancy)for P5,200,000.
- On September 22, 1963 the building, and Digested by: Jr Abul
machineries were burned and a notice of loss and Avon Insurance vs CA
damage was given to Wellington.
- Insurance adjusters computed the loss for the fire FACTS:
as P10,106,544.40 and Wellington paid only
6,481,870.07, leaving a balance of 3,624,683.43 - It all started with Yupangco Cotton Mills engaged
- The computed business interruption loss was P3M to secure with Worldwide Security and Insurance
but Wellington paid only P1,864,134.08 leaving a Co. Inc., several of its properties totaling P200
balance of P1,748,460 (computation based on Million
case) - These contracts were covered by reinsurance
- Artex through counsel Norberto Quisumbing made treaties between Worldwide Surety and Insurance,
a manifestation that only about P397,ooo is the and several foreign reinsurance companies
remaining balance and liability which was the including the petitioners through CJ Boatrwright
subject of reinsurance with Alexander and acting as agent of Worldwide Surety and Insurance
Alexander Inc, of New York, Artex acknowledging - A Fire then razed the properties insured on
here the receipt of P3,600,000 as FINAL and FULL December 1969 and May 2, 1981
SETTLEMENT of all claims against Welllington - A Deed of Assignment made by Worldwide Surety
- Artex further prays to the court to affirm the lower and Insurance acknowledged a remaining balance
courts decision of liquidation and prayed for of P19,444,447.75 still due and assigned to
modification of the amount of liability to be fixed to Yupangco all reinsurance proceeds still collectible
P397,813.00 plus 12% interest per annum thereof from all the foreign reinsurance companies.
for the late payment until april 10, 1969 and - Yupangco then filed a collection suit on the above
attorneys fees of 15% of the recovery, expenses petitioners
of litigation, no writ of execution however to be - The service of summons were made through the
office of the Insurance Commissioner but since the

EH 403 2010-2011 2011-2012


Page 64
INSURANCE LAW CASE DIGESTS
international reinsurers question the jurisdiction the jurisdiction of the courts, 19 otherwise, a
the trial court the case has not proceeded to trial foreign corporation illegally doing business here
on the merits because of its refusal or neglect to obtain the
- The reinsurer is questioning also the service of required license and authority to do business may
summons through extraterritorial service under successfully though unfairly plead such neglect or
Sect 17 Rule 14 of the Rules of Court nor through illegal act so as to avoid service and thereby
the Insurance Commissioner under Sec 14 impugn the jurisdiction of the local courts.
- Yupangco also contends that since the reinsurers - Voluntary appearance before the lower court to
question the jurisdiction of the court they are question the jurisdiction is not equivalent to
deemed to have submitted to the jurisdiction of the submission to jurisdiction
court.
The SC disposed the case in favor of the international
ISSUE: insurers (petitioners) declaring that the lower court
has not acquired and cannot acquire jurisdiction over
- WON the international reinsurers are doing them and was ordered to desist from maintaining
business in the Philippines. further proceeding against them.

- WON the Philippine court has jurisdiction over


these international reinsurers who are not doing III. CLASSES OF INSURANCE
business in the Philippines A. Marine Insurance (Sections 99-166)
i. Definition (Section 99)
RULING: ii. Insurable Interest (Sections 100-106)
iii. Concealment (Sections 107-110)
NO, international reinsurers are not doing business in iv. Representations (Sections 111-112)
the Philippines and the Philippine court has not v. Implied Warranties (Sections 113-120)
acquired jurisdiction over them. vi. The Voyage and Deviation (Sections 121-
- The reinsurance treaties between the petitioners 126)
and Worldwide Surety and Insurance were made vii. Loss (Sections 127-137)
through an international insurance broker and NOT viii. Abandonment (Sections 138-155)
through any entity or means remotely connected ix. Measure of Indemnity (Sections 156-166)
with the Philippines B. Fire Insurance (Sections 167-173)
- Reinsurance company is not doing business in a C. Casualty Insurance (Section 174)
certain state even if the property or lives which are i. Compulsory Motor Vehicle Liability
insured by the original insurer company are Insurance
located in that state. D. Suretyship (Sections 175-178)
- Reinsurance Contract is generally separate and E. Life Insurance (Sections 179-183)
distinct arrangement from the original contract of
insurance.
- Doing business in the Philippines must be judged
in the light of its peculiar circumstances upon its
peculiar facts and upon the language of the statute
applicable.
o True test: whether the foreign corporation is
continuing the body or substance of the
business or enterprise for which it was
organized
- If there exist a domestic agent of the foreign
corporation it can be served with summons
through that agent without proving that such
corporation is doing business in the phils or not.
o NO allegation or demonstration of the
existence of petitioners domestic agent but
avers simply that they are doing business not
only abroad but in the Phils
o Petitioners had not performed any act which
would give the general public the impression
that it had been engaging or intends to engage
in its ordinary and usual business undertaking
in the country.
- The purpose of the law in requiring that foreign
corporations doing business in the country be
licensed to do so, is to subject the foreign
corporations doing business in the Philippines to

EH 403 2010-2011 2011-2012


Page 65

You might also like